\documentclass[12pt, a4paper]{report} \PassOptionsToPackage{dvipsnames}{xcolor} \usepackage{tikz} \usepackage{amsmath} \usepackage{amsfonts} \usepackage{enumitem} \usepackage{amssymb} \usepackage{marvosym} \usepackage{mathtools} \usepackage[colorlinks=true, linkcolor=magenta]{hyperref} \usepackage{cancel} \usepackage[ngerman]{babel} \usepackage{harpoon} \usetikzlibrary{tikzmark,calc,arrows,arrows.meta,angles,math,decorations.markings} \usepackage{pgfplots} \usepackage{framed} \usepackage[hyperref,amsmath,amsthm,thmmarks,thref,framed]{ntheorem} \usepackage{tcolorbox} \usepackage{geometry} \geometry{a4paper, top=35mm, left=25mm, right=25mm, bottom=30mm} \pgfplotsset{compat=1.17} \title{Lineare Algebra 2} \date{Sommersemester 2022} \author{Philipp Grohs \\ \small \LaTeX-Satz: Anton Mosich} \newcounter{textbox} \def\tl{\stepcounter{textbox}\tikzmarknode{a\thetextbox}{\strut}} \def\br{\tikzmarknode{b\thetextbox}{\strut}\begin{tikzpicture}[overlay, remember picture] \draw ($(a\thetextbox.north west)+(-0.4\arraycolsep,0ex)$) rectangle ($(b\thetextbox.south east)+(0.2\arraycolsep,0ex)$);\end{tikzpicture}} % https://tex.stackexchange.com/questions/481978/how-to-write-the-block-matrix-in-latex \newcommand*{\vect}[1]{\overrightharp{\ensuremath{#1}}} \newcommand\R{\ensuremath{\mathbb{R}}} \newcommand\C{\ensuremath{\mathbb{C}}} \newcommand\K{\ensuremath{\mathbb{K}}} \newcommand\mapsfrom{\rotatebox{180}{$\mapsto$}} \definecolor{pastellblau}{HTML}{5BCFFA} \definecolor{pastellrosa}{HTML}{F5ABB9} \definecolor{weiss}{HTML}{FFFFFF} \theoremstyle{break} \theoremseparator{:\smallskip} \theoremindent=1em \theoremheaderfont{\kern-1em\normalfont\bfseries} \theorembodyfont{\normalfont} \theoreminframepreskip{0em} \theoreminframepostskip{0em} \newtcbox{theoremBox}{colback=pastellrosa!17,colframe=pastellrosa!87,boxsep=0pt,left=7pt,right=7pt,top=7pt,bottom=7pt} \def\theoremframecommand{\theoremBox} \newshadedtheorem{theo}{Theorem}[section] \newshadedtheorem{satz}[theo]{Satz} \theoremstyle{nonumberbreak} \newshadedtheorem{nonumbersatz}{Satz} \theoremstyle{break} \newshadedtheorem{lemma}[theo]{Lemma} \newshadedtheorem{korollar}[theo]{Korollar} \newshadedtheorem{folgerung}[theo]{Folgerung} \newtcbox{definBox}{colback=pastellblau!17,colframe=pastellblau!94,boxsep=0pt,left=7pt,right=7pt,top=7pt,bottom=7pt} \def\theoremframecommand{\definBox} \newshadedtheorem{defin}[theo]{Definition} \DeclareMathOperator{\sgn}{sgn} \DeclareMathOperator{\rg}{rg} \DeclareMathOperator{\spec}{spec} \DeclareMathOperator{\spur}{sp} \DeclareMathOperator{\Hom}{Hom} \DeclareMathOperator{\adj}{adj} \DeclareMathOperator{\id}{id} \DeclareMathOperator{\diag}{diag} \DeclareMathOperator{\eig}{Eig} \DeclareMathOperator{\nxn}{n \times n} \DeclareMathOperator{\im}{im} \DeclareMathOperator{\GL}{GL} \newcommand\homkv{\Hom_\K(V, V)} \newcommand\homk{\Hom_\K} \newcommand\linspan[1]{\left\langle #1 \right\rangle} \newcommand\inner[2]{\left\langle #1, #2 \right\rangle} \newcommand\norm[1]{\left\lVert #1 \right\rVert} \newcommand\ontop[2]{\genfrac{}{}{0pt}{0}{#1}{#2}} \newcommand\abs[1]{\left\lvert #1 \right\rvert} \newif\ifhideproofs %\hideproofstrue \ifhideproofs \usepackage{environ} \NewEnviron{hide}{} \let\proof\hide \let\endproof\endhide \fi \begin{document} \tikzset{% -||-/.style={decoration={markings, mark=at position 0.5 with {\draw[thick, -] (-.2,-.2) -- (0, .2);\draw[thick, -] (0, -.2) -- (.2, .2);}}, postaction={decorate}}, } \begin{titlepage} \begin{tikzpicture}[remember picture, overlay] % Trans pride flag \foreach[count=\i] \col in {pastellblau,pastellrosa,weiss,pastellrosa,pastellblau} \node (back names) [shape=rectangle, fill=\col, minimum width=\paperwidth / 5, anchor=south west, minimum height=\paperheight] at ([xshift=(\i - 1)*(\paperwidth / 5)]current page.south west) {}; % The Loss % Panel 1 \draw[line width=.5mm, {Stealth[scale=1.3]}-] ([xshift = 85pt, yshift = -60pt]current page.north west) -- +(0, -.35\paperheight); % Panel 2 \draw[line width=.5mm, {Stealth[scale=1.3]}-, -||-] ([xshift = -85pt, yshift = -.1\paperheight - 60pt] current page.north east) -- +(0, -.25\paperheight); \draw[line width=.5mm, {Stealth[scale=1.3]}-, -||-] ([xshift = -185pt, yshift = -60pt]current page.north east) -- +(0, -.35\paperheight); % Panel 3 \draw[line width=.5mm, -{Stealth[scale=1.3]}, -||-] ([xshift = 40pt, yshift = 60pt] current page.south west) -- +(0, .35\paperheight); \draw[line width=.5mm, -{Stealth[scale=1.3]}, -||-] ([xshift = 175pt, yshift = 60pt] current page.south west) -- +(0, .35\paperheight); % Panel 4 \draw[line width=.5mm, -{Stealth[scale=1.3]}] ([xshift = -175pt, yshift = 60pt] current page.south east) -- +(0, .35\paperheight); \draw[line width=.5mm, {Stealth[scale=1.3]}-] ([yshift = 120pt, xshift = -25pt] current page.south east) -- +(-.38\paperwidth,0); \draw[very thick] ([xshift = -175+40pt, yshift = 120]current page.south east) arc [radius=40pt, start angle=0, end angle=90]; % Title, Author & Date \node at ([yshift = -.45\paperheight]current page.north) {\Huge{ \textbf{Lineare Algebra 2} }}; \node at ([yshift = -.52\paperheight]current page.north) {\Large{Philipp Grohs}}; \node at ([yshift = -.55\paperheight]current page.north) {\large{\LaTeX-Satz: Anton Mosich}}; \node at ([yshift = -.60\paperheight]current page.north) {\large{Sommersemester 2022}}; \end{tikzpicture} \end{titlepage} \tableofcontents \chapter{Determinanten} \section{Permutationen} \begin{defin} Sei $n \in \mathbb{N} \setminus \{0\}, [n] := \{1, 2, \dots, n\}$. \\ Eine bijektive Abbildung $\pi:[n]\to[n]$ heißt \underline{Permutation} von $[n]$. Wir definieren die \underline{symmetrische Gruppe} $S_n := \{\pi\text{ Permutation von }[n]\}$ mit der Hintereinanderausführung als Gruppenoperation. \end{defin} \subsubsection{Bemerkung} \begin{itemize} \item $(S_n, \circ)$ ist eine Gruppe. \item $\pi\in S_n$ ist eindeutig durch das Tupel $(\pi(1), \dots, \pi(n))$ definiert. \item Fixpunkte $(\pi(i)=i)$ werden oft weggelassen. \end{itemize} \begin{defin} $\pi\in S_n$ heißt \underline{Transposition} wenn es $i, j\in [n]$ gibt mit \[\pi(k) = \begin{cases} k & k\notin\{i, j\}\\ i & k = j\\ j & k=i \end{cases}\] Wir schreiben $\pi = (ij)$. \end{defin} \begin{satz} \label{theo:1.1.3} Es gilt $\abs{ S_n } = n!$. \end{satz} \begin{proof} Vollständige Induktion \begin{itemize} \item[$n=1$:] $S_1 = \{\id\}\implies\abs{ S_1} = 1 = 1!$ \item[$n-1\to n$:] Angenommen $\abs{ S_{n-1} } = (n-1)!$. Dann gilt $\abs{\{\pi \in S_n: \pi(n) = n \}} = (n-1)!$. Sei allgemein $i \in [n]$. Dann gilt $\pi(n)=i \iff (in)\circ\pi(n)=n$. Also gilt \begin{align*} & \abs{\{\pi\in S_n: \pi(n)=i\}} = \abs{\{(in)\circ\pi: \pi(n)=n\}} \\ & = \abs{\{\pi: \pi(n)=n\}} = (n-1)! \end{align*} Weiters gilt \begin{align*} & S_n = \bigcup_{i\in[n]}^\bullet\{\pi\in S_n: \pi(n)=i\} \implies \\ & \abs{S_n}= \sum_{i\in[n]}\abs{\{\pi \in S_n: \pi(n) = i\}} = n\cdot(n-1)! = n! \end{align*} \end{itemize} \end{proof} \begin{satz} \label{theo:1.1.4} Für $n\in \mathbb{N}_{\ge2}$ ist jedes $\pi \in S_n$ das Produkt von (endlich vielen) Transpositionen. \end{satz} \begin{proof} Vollständige Induktion \begin{itemize} \item[$n=2$:] $S_2 = \{\id, (2 1)\}$ \item[$n-1\to n$:] Sei $\pi \in S_n$. Dann gilt (siehe Beweis von Satz \ref{theo:1.1.3}) mit $i=\pi(n)$, dass \[\underbrace{(i n)\pi}_{\pi_i}(n) = n\] Sei $\pi_i = (\underbrace{\pi_i(1) \dots \pi_i(n-1)}_{\in S_{n-1}} n) \underset{\text{Induktions VS}}{\implies} \pi_i = (i_1 j_1) \dots (i_k j_k)$.\\ Außerdem gilt $\pi = (i n)\pi_i$, also $\pi = (i n)(i_1 j_1) \dots (i_k j_k)$ \end{itemize} \end{proof} \subsubsection{Bemerkung} \begin{itemize} \item Produktdarstellung ist nicht eindeutig, zum Beispiel:\\ $(3 1 2) = (2 1)(3 1) = (3 1)(3 2)$ \item $f\in \mathbb{Z}[X_1, \dots, X_n], \pi \in S_n$ \\ $\pi f(X_1, \dots, X_n) := f(X_{\pi(1)}, \dots, X_{\pi(n)})$ \end{itemize} \subsubsection{Beispiel} $\pi = (2 3 1), f(X_1, X_2, X_3) = X_1-X_2+X_1X_3 \implies \pi f(X_1, X_2, X_3) = X_2 - X_3 + X_2X_1$ \begin{lemma} \label{theo:1.1.5} Sei \[ f(X_1, \dots, X_n) = \prod_{\substack{i, j\in[n]\\ i < j}} (X_j-X_i)\in \mathbb{Z}[X_1, \dots, X_n] \] Dann gilt \begin{enumerate}[label=\alph*)] \item Zu jedem $\pi \in S_n$ existiert eine eindeutig Zahl $s(\pi) \in \{-1, 1\}$ mit $\pi f = s(\pi)f$. \item Für $\pi$ eine Transposition gilt $s(\pi) = -1$. \end{enumerate} \end{lemma} \begin{proof} \begin{enumerate}[label=\alph*)] \item \begin{align*} \pi f(X_1, \dots, X_n) & = \prod_{i\pi(j)\}}} \prod_{i\pi(j)\}}} \end{align*} \item $\pi = (i j), i \pi(y)$\\ Für alle anderen Paare gilt $x3 \to $ Gaußalgorithmus \begin{defin} Sei $A\in\K^{n\times n}$ und $i, j\in[n]$. Sei $M_{ij}\in\K^{n\times n}$ die Matrix, welche durch Ersetzung der j-ten Spalte durch den i-ten Einheitsvektor $e_j$ entsteht.\\ $A_{ij}:=\det(M_{ij})$ heißt \underline{Kofaktor} (zum Indexpaar $(i, j)$). \begin{equation*} \bordermatrix{ &&&&j \cr &a_{11}&\dots &a_{1i-1}&0&a_{1i+1}&\dots&a_{1n} \cr &\vdots&\ddots&\vdots&\vdots&\vdots&\ddots&\vdots \cr i&a_{ji}&\dots&a_{ji-1}&1&a_{ji+1}&\dots&a_{jn}\cr &\vdots&\ddots&\vdots&\vdots&\vdots&\ddots&\vdots \cr &a_{n1}&\dots &a_{ni-1}&0&a_{ni+1}&\dots&a_{nn} } \genfrac{}{}{0pt}{0}{= M_{ij}}{=(a_{\_1}, \dots, \underbrace{e_i}_{j}, \dots, a_{\_n})} \end{equation*} \end{defin} \subsubsection{Bemerkung} Es gilt \begin{equation}\label{eq:1.4.4.1} A_{ij}=\begin{vmatrix} a_{11} & \dots & a_{1i-1} & 0 & a_{1i+1} & \dots & a_{1n} \\ \vdots & \ddots & \vdots & \vdots & \vdots & \ddots & \vdots \\ a_{ji} & \dots & a_{ji-1} & 1 & a_{ji+1} & \dots & a_{jn} \\ \vdots & \ddots & \vdots & \vdots & \vdots & \ddots & \vdots \\ a_{n1} & \dots & a_{ni-1} & 0 & a_{ni+1} & \dots & a_{nn} \end{vmatrix} \end{equation} da obige Matrix aus $M_{ij}$ durch Spaltenadditionen hervorgeht. \begin{lemma} Sei $\tilde{A_{ij}}\in\K^{(n-1)\times(n-1)}$ die Matrix, welche aus A durch Streichung der i-ten Spalte und j-ten Zeile hervorgeht und $D_{ij}:=\det(\tilde{A_{ij}})$. Dann gilt \[A_{ij}=(-1)^{i+j}D_{ij}\] \end{lemma} \begin{proof} Transformiere durch ($i-1$) Spaltenvertauschungen und ($j-1$) Zeilenvertauschungen die Matrix \ref{eq:1.4.4.1} auf \[ B_{ij} = \begin{pmatrix} 1 & 0 & \dots & 0 \\ 0 & & & \\ \vdots & & \tilde{A_{ij}} & \\ 0 & & & \end{pmatrix} \] Es gilt $\abs{B_{ij}}=D_{ij}$ und $\abs{B_{ij}}=(-1)^{(i-1)+j(-1)}A_{ij}$, woraus die Behauptung folgt. \end{proof} \begin{satz} [Entwicklungssatz von Laplace] Sei $A\in\K^{n\times n}$ und $i, j\in[n]$. Dann gilt \begin{enumerate}[label=\alph*)] \item $\det(A) = \sum\limits_{l=1}^na_{il}A_{il} = \sum\limits_{l=1}^n(-1)^{l+i}a_{il}D_{il}$ \item $\det(A) = \sum\limits_{l=1}^na_{lj}A_{lj} = \sum\limits_{l=1}^n(-1)^{l+j}a_{lj}D_{lj}$ \end{enumerate} \end{satz} \begin{proof} b) \[\begin{aligned} \det(A) & = \det(a_{\_1}, \dots, a_{\_n})= \\ & =\det(a_{\_1}, \dots, \underbrace{\sum_{l=1}^na_{lj}e_l}_{=a_{\_j}}, \dots, a_{\_n})= \\ & =\sum_{l=1}^n a_{lj}\det(a_{\_1}, \dots, \underbrace{e_l}_{j}, \dots, a_{\_n}) = \\ & = \sum_{l=1}^n a_{lj}A_{lj} \end{aligned} \] a) analog (angewendet auf $A^T$). \end{proof} \begin{satz} [Cramer'sche Regel] Sei $\adj(A)=(A_{ji})_{i, j\in[n]}$. Dann gilt \[A\cdot \adj(A) = \det(A)\cdot I_n\] \end{satz} \begin{proof} Sei $B=A\cdot\adj(A)\implies$ \[\begin{aligned} b_{ij} & = \sum_{k=1}^n a_{ik} A_{jk} \\ & = \sum_{k=1}^n a_{ik} \det(a_{\_1}, \dots, \underbrace{e_j}_{k}, \dots, a_{\_n}) \\ & = \sum_{k=1}^n a_{ik} \bordermatrix{ & & & k & & \\ & a_{11} & \dots & a_{1k} & \dots & a_{1n} \\ & \vdots & \ddots & \vdots & \ddots & \vdots \\ j & 0 & \dots & 1 & \dots & 0 \\ & \vdots & \ddots & \vdots & \ddots & \vdots \\ & a_{n1} & \dots & a_{nk} & \dots & a_{nn} \\ } \\ & = \det\left(\bordermatrix{ & \\ & a_{1\_} \\ & \vdots \\ j \to & a_{i\_} \\ & \vdots \\ & a_{n\_}}\right) \\ & = \begin{cases}0& i\neq j \\ \det(A) & i=j\end{cases} \end{aligned}\] \end{proof} \begin{folgerung} Sei $A\in\K^{n\times n}$ invertierbar. Sei $x\in\K^n$ die eindeutige Lösung des linearen Gleichungssystems $Ax=b$. Dann gilt \[ x_i= \det(A)^{-1} \det(a_{\_1}, \dots, \underbrace{b}_{i}, \dots, a_{\_n}) \] \end{folgerung} \begin{proof} \[\begin{aligned} & A^{-1}=\frac{1}{\det(A)}(A_{ji}) \\ & \implies \det(A)x_i=\sum_{j=1}^n A_{ji}b_j & = \sum_{j=1}^n b_j \det(a_{\_1}, \dots, \underbrace{e_j}_{i}, \dots, a_{\_n}) \\ & & =\det(a_{\_1}, \dots, \underbrace{b}_{i}, \dots, a_{\_n}) \end{aligned}\] \end{proof} \subsubsection{Blockmatrizen} \begin{defin} $A\in\K^{n\times n}$ heißt \underline{obere Blockmatrix} wenn $\exists p\in \{1, \dots, n-1\}$ mit $a_{ij}=0$ für $p+1\le i\le n, 1\le j\le p$, d.h. \begin{equation} \label{blockmatrix} A=\bordermatrix{ \ &\overbrace{}^{p} & \overbrace{}^{n-p} \cr p\{\ & P & D \cr % } TODO geschwungene Klammern besser machen n-p\{\ &0&Q} % } \end{equation} Analog sind \underline{untere Blockmatrizen} definiert. \end{defin} \begin{satz} \label{theo:1.4.10} Sei $A$ obere Blockmatrix wie in \ref{blockmatrix}. Dann gilt $\det(A)= \det(P) \det(Q)$ \end{satz} \begin{proof} Sei $A = \begin{pmatrix} P & D \\ 0 & Q \end{pmatrix}$.\\ Wende elementare Zeilenumformungen der ersten $p$ Zeilen an, sodass $P$ obere Dreiecksform hat (mit $s$ Zeilenvertauschungen) und elementare Zeilenumformungen der letzten $n-p$ Zeilen sodass $Q$ obere Dreiecksform hat (mit $t$ Zeilenvertauschungen). Bezeichne das Ergebnis mit $A'= \begin{pmatrix} P' & D \\ 0 & Q' \end{pmatrix}$, wobei $P', Q'$ obere Dreiecksform haben.\\ Es folgt, dass $A', P', Q'$ obere Dreiecksform hat. Da die Determinante oberer Dreiecksmatrizen das Produkt der Diagonalelemente ist, gilt $\det(A')=\det(P')\det(Q')$.\\ Weiters gilt $\det(A')=(-1)^{s+t} \det(A)$ (insgesamt $s+t$ Vertauschungen) und $\det(P')= (-1)^s \det(P), \det(Q') = (-1)^t \det(Q)$. Daraus folgt die Behauptung. \end{proof} \chapter{Eigenwerte und Eigenvektoren} \section{Diagonalisierbarkeit} \begin{defin} $D\in \K^{n\times n}$ heißt \underline{Diagonalmatrix} wenn $\forall i\neq j: d_{ij}=0$. Wir schreiben auch \[ \diag(\lambda_1, \dots, \lambda_n):=\begin{pmatrix} \lambda_1 & 0 & \dots & 0 \\ 0 & \lambda_2 & \dots & 0 \\ \vdots & \vdots & \ddots & \vdots \\ 0 & 0 & \dots & \lambda_n \end{pmatrix} \] \end{defin} \subsubsection{Bemerkung} \begin{itemize} \item $A\in \K^{n\times m} \implies \diag(\lambda_1, \dots, \lambda_n)A = \begin{pmatrix} \lambda_1 a_{1\_} \\ \vdots \\ \lambda_n a_{n\_}\end{pmatrix}$ \item $\diag(\lambda_1, \dots, \lambda_n)^k = \diag(\lambda_1^k, \dots, \lambda_n^k)$ \end{itemize} \begin{defin} \begin{enumerate}[label=\alph*)] \item $\alpha \in \homkv, \dim(V)<\infty$ heißt \underline{diagonalisierbar} (bzgl. $B$) wenn eine geordnete Basis $B$ existiert mit ${}_B M(\alpha)_B$ Diagonalmatrix \item $A\in\K^{n\times n}$ heißt diagonalisierbar wenn eine invertierbare Matrix $P\in\K^{n\times n}$ existiert mit $P^{-1}AP$ Diagonalmatrix. \end{enumerate} \end{defin} \begin{lemma} Sei $V$ ein $\K$-Vektorraum mit $\dim(V)=n<\infty$. \\ Dann gilt für $\alpha\in\homkv$ und $C$ Basis: \[\alpha \text{ diagonalisierbar} \iff {}_C M(\alpha)_C \text{ diagonalisierbar}\] \end{lemma} \begin{proof} \begin{itemize} \item[$\implies$:] Sei $\alpha$ diagonalisierbar und $B$ eine Basis mit $_B M(\alpha)_B$ Diagonalmatrix. Dann gilt \begin{align*} {}_B M(\alpha)_B & = {}_B M(\id)_C \cdot {}_C M(\alpha)_C \cdot {}_C M(\id)_B \\ & = {}_C M(\id)_{B^{-1}} \cdot {}_C M(\alpha)_C \cdot {}_C M(\id)_B \end{align*} Also ist ${}_C M(\alpha)_C$ diagonalisierbar. \item[$\impliedby$:] Sei ${}_C M(\alpha)_C$ diagonalisierbar und $P$ invertierbar mit $P^{-1}\cdot {}_C M(\alpha)_C \cdot P$ Diagonalmatrix. Sei $B$ Basis mit $P={}_C M(\id)_B$. Dann gilt ${}_B M(\alpha)_B$ ist Diagonalmatrix. \end{itemize} \end{proof} \begin{lemma} \label{theo:2.1.4} \begin{enumerate}[label=\alph*)] \item $\alpha \in \homkv$ ist diagonalisierbar genau wenn es eine Basis $B=(b_1, \dots, b_n)$ und $\lambda_1, \dots, \lambda_n\in\K$ gibt mit $\forall i=1, \dots, n:\alpha(b_i)=\lambda_i b_i$. \item $A\in\K^{n\times n}$ ist diagonalisierbar genau wenn es eine geordnete Basis $B= (b_1, \dots, b_n)$ von $\K^n$ gibt mit $\forall i=1, \dots, n: A b_i = \lambda_i b_i$. \end{enumerate} \end{lemma} \begin{proof} \begin{enumerate} [label=\alph*)] \item die Bedingung ist äquivalent zu ${}_B M(\alpha)_B$ diagonalisierbar. \item Spezialfall von a). \end{enumerate} \end{proof} \section{Eigenwerte und Eigenvektoren} \begin{defin} \label{theo:2.2.1} \begin{enumerate}[label=\alph*)] \item Sei $\alpha \in \homkv$. $\lambda\in\K$ heißt \underline{Eigenwert} von $\alpha$ wenn es einen Vektor $v\in V\setminus\{0\}$ gibt mit $\alpha(v)=\lambda v$. $v$ heißt \underline{Eigenvektor} zu $\lambda$.\\ Die Menge aller Eigenwerte von $\alpha$ heißt \underline{Spektrum} von $\alpha; \spec(\alpha)$ \item Sei $A \in \K^{n\times n}$. $\lambda\in\K$ heißt \underline{Eigenwert} von $A$ wenn es $v\in \K^n\setminus\{0\}$ gibt mit $A v = \lambda v$. $v$ heißt \underline{Eigenvektor} zu $\lambda$.\\ Die Menge aller Eigenwerte von $A$ heißt \underline{Spektrum} von $A; \spec(A)$ \end{enumerate} \end{defin} \begin{lemma} \label{theo:2.2.2} \begin{enumerate}[label=\alph*)] \item $\alpha \in \homkv$ diagonalisierbar $\iff \exists$ Basis aus Eigenvektoren. \item $A \in \K^{n\times n}$ diagonalisierbar $\iff \exists$ Basis aus Eigenvektoren. \end{enumerate} \end{lemma} \begin{proof} Folgt direkt aus Lemma \ref{theo:2.1.4} und Definition \ref{theo:2.2.1} \end{proof} \begin{defin} \begin{enumerate}[label=\alph*)] \item Sei $\alpha \in \homkv$ und $\lambda \in \spec(\alpha)$. Dann heißt $\eig_\alpha(\lambda):=\{v\in V: \alpha(v) = \lambda v \}$ der zugehörige \underline{Eigenraum}. \item Sei $A \in \K^{n\times n}$ und $\lambda \in \spec(A)$. Dann heißt $\eig_A(\lambda):=\{v\in \K^n: A v = \lambda v \}$ der zugehörige \underline{Eigenraum}. \end{enumerate} \end{defin} \begin{lemma} Sei $\alpha \in \homkv / A\in\K^{n\times n}$ und $\lambda \in \spec(\alpha)/\lambda\in\spec(A)$.\\ Dann ist $\eig_\alpha(\lambda)/\eig_A(\lambda)$ ein Unterraum von $V/\K$. \end{lemma} \begin{proof} Nur für $\alpha\in\homkv$ \begin{itemize} \item $ 0 = \alpha(0) = \lambda \cdot 0 \implies 0 \in \eig_\alpha(\lambda) $ \item $v, w\in \eig_\alpha(\lambda) \implies \alpha(v+w) = \alpha(v) + \alpha(w) = \lambda v + \lambda w = \lambda(v + w) \implies v + w \in \eig_\alpha(V)$ \item $\mu \in \K, v \in \eig_\alpha(\lambda) \implies \alpha(\mu v) = \mu \cdot \alpha(v) = \mu \cdot \lambda \cdot v = \lambda \cdot (\mu \cdot v) \implies \mu \cdot v \in \eig_\alpha(\lambda)$ \end{itemize} \end{proof} \begin{satz} Sei $\alpha \in \homkv$ und $B$ Basis. Dann gilt \begin{align*} & \spec(\alpha) = \spec({}_B M(\alpha)_B) \\ & {}_B\Phi(\eig_\alpha(\lambda)) = \eig_{{}_B M(\alpha)_B}(\lambda) \end{align*} \end{satz} \begin{proof} Sei $\lambda \in \spec(\alpha)$ und $v\in\eig_\alpha(\lambda)$. Dann gilt \[ \alpha(v) = \lambda v \iff {}_B M(\alpha)_B \cdot {}_B v = \lambda \cdot {}_B v \] \end{proof} \begin{defin} \begin{enumerate}[label=\alph*)] \item Sei $\alpha \in \homkv, \dim(V)<\infty$ und $B$ Basis. Dann heißt die Funktion \[ \chi_\alpha:\begin{cases}\K \to \K \\ \lambda \mapsto \det({}_B M(\alpha)_B - \lambda \cdot I_n)\end{cases} \] \underline{charakteristisches Polynom} von $\alpha$. \item Sei $A \in \K^{n\times n}$. Dann heißt die Funktion \[ \chi_A:\begin{cases}\K \to \K \\ \lambda \mapsto \det(A - \lambda \cdot I_n)\end{cases} \] \underline{charakteristisches Polynom} von $A$. \end{enumerate} \end{defin} \subsubsection{Bemerkung} $\genfrac{}{}{0pt}{0}{\chi_\alpha}{\chi_A}$ ist Polynom vom Grad $\le\genfrac{}{}{0pt}{0}{\dim(V)}{n}$, da \begin{align*} & \chi_A(\lambda)= \sum_{\pi \in S_n} \tilde{a}_{1\pi(1)}^{(\lambda)} \cdots \tilde{a}_{n\pi(n)}^{(\lambda)} \text{ mit} \\ & \tilde{a}_{ij}^{(\lambda)} = \begin{cases} a_{ij} & i\neq j \\ a_{ij}-\lambda & i=j \end{cases} \dots \text{ Polynom von Grad $0$ oder $1$} \end{align*} \begin{lemma} \label{theo:2.2.7} \begin{enumerate}[label=\alph*)] \item $\chi_\alpha$ ist unabhängig von der Wahl der Basis. \item $\chi_A = \chi_B$ wenn $A, B$ ähnlich (das heißt $\exists P \in \K^{n \times n}: B = P^{-1}AP$) \end{enumerate} \end{lemma} \begin{proof} \begin{enumerate}[label=\alph*)] \item Sei C weitere Basis.\\ Dann gilt $\underbrace{{}_C M(\alpha)_C}_{B} = \underbrace{{}_C M(\id)_B}_{P^{-1}} \underbrace{{}_B M(\alpha)_B}_{A} \underbrace{{}_B M(\id)_C}_{P}$. \\ Man kann also alles auf b) zurückführen. \item \[\begin{aligned} \chi_A(\lambda) & = \det(A-\lambda I) \\ & = \det(P)^{-1} \det(A - \lambda I) \det(P) \\ & = \det(P^{-1}) \det(A - \lambda I) \det(P) \\ & = \det(P^{-1}(A - \lambda I)P) \\ & = \det(P^{-1}AP-\lambda I) \\ & = \det(B - \lambda I) \\ & = \chi_B(\lambda) \end{aligned}\] \end{enumerate} \end{proof} \begin{lemma} \begin{enumerate}[label=\alph*)] \item Sei $\alpha\in\homkv$. Dann gilt \[\spec(\alpha) = \{\lambda \in \K: \chi_\alpha(\lambda)=0\}\] \item Sei $A\in \K^{\nxn}$. Dann gilt \[\spec(A) = \{\lambda \in \K: \chi_A(\lambda)=0\}\] \end{enumerate} \end{lemma} \begin{proof} Nur b) \[\begin{aligned} \lambda \in \spec(A) & \iff \exists v\in V \setminus \{0\}: A v = \lambda v \\ & \iff \exists v \in V \setminus \{0\}: (A - \lambda I) v = 0 \\ & \iff \ker(A - \lambda I) \neq \{0\} \\ & \iff A - \lambda I \text{ nicht injektiv} \\ & \iff \det(A - \lambda I) = 0 \end{aligned}\] \end{proof} \subsubsection{Beispiele} \begin{alignat*}{3} & A = \begin{pmatrix}\bar3 & \bar4 \\ \bar1 & \bar1 \end{pmatrix} \in \mathbb{Z}_5^{2\times2} & & \\ & \chi_A(\lambda) = \begin{vmatrix} \bar3 - \lambda & \bar4 \\ \bar1 & \bar1 - \lambda \end{vmatrix} & & = (\bar3 - \lambda)(\bar1 - \lambda) - \bar4 \\ & & & = \bar3 - \bar4 \lambda + \lambda^2 - \bar4 \\ & & & = \bar4 - \bar4 \lambda + \lambda^2 = (\bar2 - \lambda)^2 \\ & \implies \spec(A) = \{2\} \\ & \eig_{\bar2}(A) = ? \\ & v \in \eig_{\bar2}(A) \iff \mathrlap{(A - \bar2 I)v = 0} \\ & \iff \mathrlap{\left(\begin{array}{c c | c} \bar3 - \bar2 & \bar4 & \bar0 \\ \bar1 & \bar1 - \bar2 & \bar0 \end{array}\right)} \\ & \left(\begin{array}{c c | c} \bar1 & \bar4 & \bar0 \\ \bar1 & \bar4 & \bar0 \end{array}\right) \\ & \left(\begin{array}{c c | c} \bar1 & \bar4 & \bar0 \\ \bar0 & \bar0 & \bar0 \end{array}\right) \\ & \implies \eig_{\bar2}(A) = \linspan{\begin{pmatrix}\bar1 \\ \bar1\end{pmatrix}} \\ & \implies A \mathrlap{\text{ nicht diagonalisierbar [Lemma \ref{theo:2.1.4} (b)]}} \end{alignat*} \begin{lemma} Sei $A \in \C^{n\times n}$ mit reellen Einträgen. Dann gilt: \begin{enumerate}[label=\alph*)] \item $\lambda \in \spec(A) \implies \overline{\lambda} \in \spec(A)$ \item $v \in \eig_\lambda(A) \implies \overline{v} \in \eig_{\overline{\lambda}}(A)$ \end{enumerate} \end{lemma} \begin{proof} \begin{enumerate}[label=\alph*)] \item Klarerweise ist $\chi_A(\lambda)$ ein Polynom mit reellen Koeffizienten, also $\chi_A(\lambda)=a_0+a_1 \lambda + \cdots + a_n \lambda^n, a_0, \dots, a_n\in\R$\\ Sei $\chi_A(\lambda)=0 \implies 0 = \overline0 = a_0 + a_1 \overline\lambda + \cdots + a_n \overline{\lambda} ^ n = \chi_A(\overline\lambda)$ \item $v\in\eig_\lambda(A) \implies A v = \lambda v \implies \overline{A V} = \overline{\lambda v} \implies A \overline{v} = \overline\lambda \overline{v}$ \end{enumerate} \end{proof} \begin{lemma} \label{theo:2.2.10} Eigenvektoren zu unterschiedlichen Eigenwerten sind linear unabhängig. \end{lemma} \begin{proof} Seien $v_i \in \eig_{\lambda_i}(A), i=1, \dots, r, \lambda_i \neq \lambda_j \text{ für } i\neq j$. Induktion nach $r$ \begin{itemize} \item[$r=1$:] $v_1$ ist linear unabhängig. \item[$r-1\mapsto r$:] \begin{equation}\label{eq:2.2.10.1} \mu_1 v_1 + \cdots + \mu_1 v_1 = 0 \end{equation} \[ \implies A(\mu_1 v_1 + \cdots + \mu_r v_r) = 0 \] \begin{equation}\label{eq:2.2.10.2} \implies \lambda_1\mu_1 v_1 + \cdots \lambda_r \mu_r v_r = 0 \end{equation} Weiters folgt durch Multiplikation von \ref{eq:2.2.10.1} mit $\lambda_r$, dass \begin{equation}\label{eq:2.2.10.3} \lambda_r \mu_1 v_1 + \cdots + \lambda_r \mu_r v_r = 0 \end{equation} \[ \begin{aligned} \text{\ref{eq:2.2.10.3}} - \text{\ref{eq:2.2.10.2}} & \implies \underbrace{(\lambda_r - \lambda_1)}_{\neq0} \mu_1 v_1 + \cdots + \underbrace{(\lambda_r - \lambda_{r-1})}_{\neq0} \mu_{r-1} v_{r-1} = 0 \\ & \implies v_1, \dots, v_{r-1} \text{ linear abhängig. \Lightning} \end{aligned} \] \end{itemize} \end{proof} \begin{lemma} Sei $\alpha \in \homkv, \dim(V)=n \text{ oder } A \in \K^{\nxn}$ mit $n$ verschiedenen Eigenvektoren. Dann ist $\alpha/A$ diagonalisierbar. \end{lemma} \begin{proof} Wegen Lemma \ref{theo:2.2.10} gibt es Basis von Eigenvektoren. Daher ist $\alpha/A$ diagonalisierbar wegen Lemma \ref{theo:2.2.2}. \end{proof} \begin{defin} Sei $\spec(A) = \{\lambda_1, \dots, \lambda_r \}$ und $(\lambda_1 - \lambda)^{k_1} \cdots (\lambda_r - \lambda)^{k_r} p \in\K[X]$ mit $p$ nicht durch Linearfaktoren teilbar (also keine Nullstellen in $\K$).\\ $k_i$ heißt \underline{algebraische Vielfachheit} des Eigenwerts $\lambda_i$. Wir schreiben $k_i = m_a(\lambda_i)$.\\ $\dim(\eig_A(\lambda_i))$ heißt \underline{geometrische Vielfachheit} des Eigenwerts $\lambda_i$. Wir schreiben $\dim(\eig_A(\lambda_i)) = m_g(\lambda_i)$ \end{defin} \subsubsection{Beispiel} \begin{itemize} \item $\chi_A(\lambda) = \lambda^4 - 2 \lambda^3 + 2 \lambda^2 - 2\lambda + 1 \in \R[X]$\\ $\implies \chi_A(\lambda) = (1 - X)^2 \underbrace{(1 + \lambda^2)}_{p(\lambda)}$ \\ $\implies m_a(1) = 2$ \item Für $\K=\C$ zerfällt jedes Polynom in Linearfaktoren, also ist $p$ immer konstant. \end{itemize} \begin{satz} Sei $\mu\in\spec(A)/\spec(\alpha)$. Dann gilt \[ 1\le m_g(\mu) \le m_a(\mu) \] \end{satz} \begin{proof} Klarerweise gilt $1\le m_g(\mu)$ da $\mu$ Eigenwert ist. Sei $r:= m_g(\mu)$ und $b_1, \dots, b_r$ Basis von $\eig_\alpha(\mu)$. Sei $B=(b_1, \dots, b_n)$ Basis. Dann ist \[ {}_B M(\alpha)_B = \bordermatrix{ & & & & r & & \cr & \mu & 0 & 0 & 0 & * & \dots & * \cr & 0 & \mu & 0 & 0 & * & \dots & * \cr & \vdots & & \ddots & \vdots & \vdots & & \vdots \cr r & 0 & 0 & 0 & \mu & * & \dots & * \cr & \vdots & \vdots & \vdots & \vdots & \vdots & & \vdots \cr & 0 & 0 & 0 & 0 & * & \dots & * } \text{, also} \] \begin{align*} \chi_\alpha(\lambda) & = \abs {\begin{array}{c | c} \begin{smallmatrix} \mu - \lambda & & \\ & \ddots & \\ & & \mu - \lambda \end{smallmatrix} & A \\ \hline \\ 0 & B \end{array}} \underbrace{=}_ {\text{Satz \ref{theo:1.4.10}}} \det \begin{pmatrix} \mu - \lambda & & 0 \\ & \ddots & \\ 0 & & \mu - \lambda \end{pmatrix} \cdot \det(B) \\ & = (\mu - \lambda)^r \det(B) \\ & \implies r \le m_a(\mu) \end{align*} \end{proof} \begin{lemma} Seien $A, B$ ähnlich und $\mu \in \spec(A) (=\spec(B) \text{ nach Lemma \ref{theo:2.2.7}})$. Dann stimmen die geometrischen Vielfachheiten überein, das heißt $\dim(\eig_\mu(A)) = \dim(\eig_\mu(B))$. \end{lemma} \begin{proof} Sei $B = P^{-1} A P$. Dann gilt \begin{align*} \eig_{\mu}(B) & = \ker(B - \mu I) = \ker(B - \mu P^{-1} P) \\ & = \ker(P^{-1} (A - \mu I) P) \\ & \underbrace{\implies}_{\mathclap{\text{ Für ähnliche Matrizen stimmen die Dimensionen der Kerne überein }}} \dim(\eig_\mu(B)) = \dim\eig_\mu(A) \end{align*} \end{proof} \begin{satz} $A/\alpha$ diagonalisierbar $\iff$ \begin{enumerate}[label=\roman*)] \item $\chi_{A/\alpha}$ zerfällt in Linearfaktoren, das heißt \[ \chi_{A/\alpha}(\lambda)= (\lambda_1 - \lambda)^{k_1} \cdots (\lambda_r - \lambda)^{k_r}, \sum k_i = n \] \item algebraische und geometrische Vielfachheiten stimmen überein, das \\ heißt $m_a(\lambda_i) = m_g(\lambda_i), i=1, \dots, r$ \end{enumerate} \end{satz} \begin{proof} \begin{itemize} \item[$\impliedby$:] Aus i), ii) folgt, dass \begin{equation}\label{eq:2.2.15.1} \sum_{i=1}^r \underbrace{\dim(\eig_\alpha(\lambda_i))}_{=m_g(\lambda_i)=:d_i} = n \end{equation} Sei $b_i^1, \dots, b_i^{d_i}$ Basis von $\eig_\alpha(\lambda_i)$. Wir zeigen, dass $B=\{b_i^1, \dots, b_i^{d_i}: i=1, \dots, r\}$ Basis ist. \begin{enumerate}[label=\arabic*)] \item $\abs{ B } = n$ folgt aus \ref{eq:2.2.15.1} \item Ang. $\sum\limits_{i=1}^r (\underbrace{\mu_i^1 b_i^1 + \cdots + \mu_i^{d_i} b_i^{d_i}}_{v_i}) = 0$ \\ $\underbrace{\implies}_ {\mathclap{\substack{v_i \text{Eigenwerte zu} \\ \text{verschiedenen Eigenvektoren} \\ + \text{Lemma \ref{theo:2.2.10}}}}} v_i = 0 \forall i=1, \dots, r \underbrace{\implies}_ {\mathclap{\substack{b_i^1, \dots, b_i^{d_i} \\ \text{Basis von } \eig_\alpha(\lambda_i)}}} \mu_i^1, \dots, \mu_i^{d_i} = 0 \forall i=1, \dots, r$ \\ $ \implies B $ ist Basis aus Eigenvektoren $\underbrace{\implies}_{\mathclap{\text{Lemma \ref{theo:2.2.2}}}} \alpha $ diagonalisierbar. \end{enumerate} \item[$\implies$:] Sei $\alpha$ diagonalisierbar. \[\begin{aligned} & \implies \exists \text{ Basis } \{b_1, \dots, b_n\} \text{ aus Eigenvektoren} \\ & \implies {}_B M(\alpha)_B = \diag(\lambda_1, \dots, \lambda_n) \\ & \implies \chi_B(\lambda) = (\lambda_1 - \lambda) \cdots (\lambda_n - \lambda) \end{aligned}\] \end{itemize} \end{proof} \subsubsection{Diagonalisieren} \begin{enumerate}[label=\arabic*)] \item Zerlegung in Linearfaktoren \[ \chi_A(\lambda) = (\lambda_1 - \lambda)^{m_a(\lambda_1)} \cdots (\lambda_r - \lambda)^{m_a(\lambda_r)} \] \item Bestimme Basis $B_i$ der Eigenräume \[ \eig_A(\lambda_i) = \ker(A - \lambda_i I) \] \item Ordne Basis $B= \bigcup\limits_{i=1}^n B_i$ zu $B= (b_1, \dots, b_n)$ \item Mit $S = (b_1, \dots, b_n)$ gilt dann \[ \diag(\underbrace{\lambda_1, \dots, \lambda_n}_{ \mathclap{\substack{\text{Eigenwerte werden nach} \\ \text{Vielfachheit gezählt!} \\ \lambda_i \text{ ist Eigenwert von } b_i \text{!}}} }) = S^{-1} A S \] \end{enumerate} \subsubsection{Beispiel} $A = \begin{pmatrix} 1 & 2 & 2 \\ 2 & -2 & 1 \\ 2 & 1 & -2 \end{pmatrix}$ \begin{enumerate}[label=\arabic*)] \item \begin{align*} \chi_A(\lambda) = & \begin{vmatrix} 1 -\lambda & 2 & 2 \\ 2 & -2 -\lambda & 1 \\ 2 & 1 & -2 -\lambda \end{vmatrix} \\ \underbrace{=}_{\mathclap{\substack{\text{Entwicklung} \\ \text{nach 1. Zeile}}}} & (1-\lambda) \begin{vmatrix} -2 -\lambda & 1 \\ 1 & -2 -\lambda \end{vmatrix} + (-2) \begin{vmatrix} 2 & 1 \\ 2 & -2-\lambda \end{vmatrix} \\ & + 2 \begin{vmatrix} 2 & -2 - \lambda \\ 2 & 1 \end{vmatrix} \\ = & \dots= -\lambda^3 - 3 \lambda^2 + 9\lambda + 27 = (3-\lambda)(-3-\lambda)^2 \end{align*} \item $\lambda = 3$ \begin{align*} & \left( \begin{array}{c c c | c} 1-3 & 2 & 2 & 0 \\ 2 & -2-3 & 1 & 0 \\ 2 & 1 & -2-3 & 0 \end{array} \right) = \left( \begin{array}{c c c | c} -2 & 2 & 2 & 0 \\ 2 & -5 & 1 & 0 \\ 2 & 1 & -5 & 0 \end{array} \right) \\ & \sim \left( \begin{array}{c c c | c} -1 & 1 & 1 & 0 \\ 0 & -3 & 3 & 0 \\ 0 & 3 & -3 & 0 \end{array} \right) \sim \left( \begin{array}{c c c | c} 1 & -1 & -1 & 0 \\ 0 & 1 & -1 & 0 \\ 0 & 0 & 0 & 0 \end{array} \right) \sim \left( \begin{array}{c c c | c} 1 & 0 & -2 & 0 \\ 0 & 1 & -1 & 0 \\ 0 & 0 & 0 & 0 \end{array} \right) \\ & \implies \eig_A(3) = \linspan{\begin{pmatrix} 2 \\ 1 \\ 1 \end{pmatrix}} \end{align*} $\lambda = -3$ \begin{align*} & \left( \begin{array}{c c c | c} 1+3 & 2 & 2 & 0 \\ 2 & -2+3 & 1 & 0 \\ 2 & 1 & -2+3 & 0 \end{array} \right) = \left( \begin{array}{c c c | c} 4 & 2 & 2 & 0 \\ 2 & 1 & 1 & 0 \\ 2 & 1 & 1 & 0 \end{array} \right) \\ & \sim \left( \begin{array}{c c c | c} 2 & 1 & 1 & 0 \\ 0 & 0 & 0 & 0 \\ 0 & 0 & 0 & 0 \end{array} \right) \\ & \implies \eig_A(-3) = \linspan {\begin{pmatrix} -1 \\ 0 \\ 2 \end{pmatrix}, \begin{pmatrix} -1 \\ 2 \\ 0 \end{pmatrix}} \end{align*} \item \begin{align*} & S = \begin{pmatrix} 2 & -1 & -1 \\ 1 & 0 & 2 \\ 1 & 2 & 0 \end{pmatrix} \\ & \implies S^{-1} A S = \begin{pmatrix} 3 & 0 & 0 \\ 0 & -3 & 0 \\ 0 & 0 & -3 \end{pmatrix} \end{align*} \end{enumerate} \begin{lemma} \label{theo:2.2.16} Sei $A\in\K^{\nxn}$ und $\underbrace{\spur(A)}_{\mathclap{\color{red}\text{\dq Spur von $A$ \dq}}} := \sum\limits_{i=1}^n a_{ii}$ \[\chi_A(\lambda) = (-1)^n\lambda^n + (-1)^{n-1} \spur(A) \lambda^{n-1} + \cdots + \det(A)\] \end{lemma} \begin{proof} $\chi_A(\lambda) = \sum\limits_{\pi \in S_n} \sgn(\pi) \prod\limits_{i=1}^n \tilde{a}_{i\pi(i)}$ mit $\tilde{a}_{ij} = \begin{cases} a_{ij} & i\neq j \\ a_{ij} - \lambda & i=j\end{cases}$. \\ Wenn $\pi\neq \id$ gilt $\deg\left(\prod\limits_{i=1}^n \tilde{a}_{i\pi(i)}\right)\le n-2$, da mindestens zwei Elemente vertauscht werden. Die Koeffizienten von Grad $n, n-1$ kann man also aus $\prod\limits_{i=1}^n \tilde{a}_{ii} = \prod\limits_{i=1}^n (\tilde{a}_{ii} - \lambda)$ ablesen. Daraus folgt die Behauptung für die höchsten beiden Koeffizienten. Weiters gilt $\chi_A(0)=\det(A)$, was die Aussage für den konstanten Koeffizienten zeigt. \end{proof} \[ \sigma_j := (-1)^j \sum\limits_{\substack{S\subset [n] \\ \abs{ S } = n-j}} \prod\limits_{s \in S} \lambda_s \] \begin{korollar} \begin{enumerate}[label=\alph*)] \item $A\sim B \implies \spur(A)=\spur(B)$ \item A diagonalisierbar $\implies \spur(A)=\lambda_1 + \cdots + \lambda_n$ mit $\lambda_1, \dots, \lambda_n$ Eigenwerte von $A$, nach Vielfachheit gezählt. \item A diagonalisierbar $\implies \det(A)=\lambda_1 \cdot \dots \cdot \lambda_n$ mit $\lambda_1, \dots, \lambda_n$ Eigenwerte von $A$, nach Vielfachheit gezählt. \end{enumerate} \end{korollar} \begin{proof} Folgt daraus, dass das charakteristische Polynom (und damit seine Koeffizienten) unter Ähnlichkeit invariant sind (Lemma \ref{theo:2.2.7}) und Lemma \ref{theo:2.2.16} \end{proof} \begin{satz} [Cayley-Hamilton] \dq$\chi_A(A) = 0$\dq, das heißt sei $A\in \K^{\nxn}$ mit charakteristischem Polynom $\chi_A(\lambda)=c_n \lambda^n + c_{n-1} \lambda^{n-1} + \cdots + c_0$. Dann gilt \[ \chi_A(A):=c_n A^n + c_{n-1} A ^{n-1} + \cdots c_0 I = 0 = \begin{pmatrix}0 &\dots &0 \\ \vdots& \ddots &\vdots \\ 0 & \dots & 0\end{pmatrix} \in \K^{\nxn} \] \end{satz} \begin{proof} Sei $B := A^T - \lambda I = \begin{pmatrix} a_{11} - \lambda & a_{21} & \dots & a_{n1} \\ a_{12} & a_{22} - \lambda & \dots & a_{n2} \\ \vdots & \ddots & \ddots & \vdots \\ a_{1n} & a_{2n} & \dots & a_{nn} - \lambda \end{pmatrix} = (a_{ji} - \delta_{ij} \lambda)_{ij}$ und $C:= \adj(B)$, sodass \begin{equation} CB = \det(B) I_n = \chi_A = I_n [\chi_A = \chi_{A^T} \label{eq:2.2.18.1} \end{equation} \ref{eq:2.2.18.1} heißt komponentenweise, dass \begin{flalign} & \sum_{i=1}^{n} \underbrace{c_{ki}}_{\mathrlap{\text{Polynome, in die $A$ eingesetzt werden kann}}} \underbrace{b_{ij}} = \delta_{ij} \cdot \underbrace{\chi_A} \forall k, j \in [n] \nonumber \\ = & \sum_{i=1}^{n}c_{ki}(A) b_{ij}(A) = \delta_{jk}\chi_A (A) \label{eq:2.2.18.2} \end{flalign} Wegen $b_{ij}(A) = a_{ji} I_N - \delta_{ij}A$ gilt weiters \begin{equation} \forall i \in [n]: \sum_{j=1}^{n} b_{ij}(A) e_j = \left(\sum_{j=1}^{n} a_{ji} e_j\right) - A e_i = 0 \label{eq:2.2.18.3} \end{equation} Es folgt $\forall k \in [n]$ \begin{align*} \chi_A (A) e_k & = \sum_{j=1}^{n} \delta_{jk} \chi(A) e_j & \\ & \underbrace{=}_{\mathclap{\text{\ref{eq:2.2.18.2}}}} \sum_{j=1}^{n} \sum_{i=1}^{n} c_{ki}(A) b_{ij}(A) e_j & \\ & = \sum_{i=1}^{n} c_{ki}(A) \left(\sum_{j=1}^{n} b_{ij(A) e_j}\right) & \\ & \underbrace{=}_{\mathclap{\text{\ref{eq:2.2.18.3}}}} 0 & \\ & \implies \chi_A(A) = 0 \end{align*} \end{proof} \subsubsection{Berechnung der Koeffizienten von $\chi_A$} Sei $f(\lambda) \underbrace{=}_{\text{(*)}} \prod\limits_{j=1}^{n}(\lambda_j - \lambda) = \underbrace{c_n\lambda^n}_{=(-1)^n} + c_{n-1}\lambda ^{n-1} + \cdots + c_0$ Wie können wir $c_j$ effizient bestimmen? \begin{itemize} \item [Bemerkung 1:] $\displaystyle { c_j = (-1)^{j} \sum_{\substack{S\subseteq [n] \\ \abs{ S } = n-j}} \prod_{s \in S} \lambda_s =: \sigma_{n-j}^n (\lambda_1, \dots, \lambda_n)}$ \\ Dies folgt aus (*) durch Ausmultiplizieren \\ Sei nun weiters $p_j^n(\lambda_1, \dots, \lambda_n) := \sum\limits_{i=1}^{n}\lambda_i^j$ \item [Bemerkung 2:] $\sigma_j^n, p_j^n$ sind symmetrisch, das heißt \[ \begin{aligned} & \sigma_j^n(\lambda_{\pi(1)}, \dots, \lambda_{\pi(n)}) = \sigma_{j}^n (\lambda_1, \dots, \lambda_n) \\ & p_j^n(\lambda_{\pi(1)}, \dots, \lambda_{\pi(n)}) = p_{j}^n (\lambda_1, \dots, \lambda_n) \end{aligned} \text{ für } \pi \in S_n \] \end{itemize} \begin{lemma} [Newtonidentität] \label{theo:2.2.19} Es gilt für $k\le n$ \[k\sigma_k^n+\sum_{j=0}^{k-1}\sigma_j^n p_{k-j}^n=0\] \end{lemma} \begin{proof} Induktion. \begin{itemize} \item [$k=n$:] Wegen \begin{equation*} 0= \sum_{i=1}^{n} = \sum_{i=1}^{n} \sum_{j=0}^n c_j \lambda_i^j = \sum_{j=0}^n c_j p_j^n = \sum_{j=0}^n \sigma_{n-j}^n p_j^n = \sum_{j=0}^n \sigma_j^n p_{n-j}^n \end{equation*} folgt $\sigma_n^n p_0^n + \sum\limits_{j=0}^n \sigma_j^n p_{n-j}^n = 0$ was mit $p_0^n = n$ die gewünschte Aussage liefert. \item [$k1$:] \begin{align*} & \alpha(c_i) = \alpha(\tilde{b}_i) = \alpha\left(\sum_{j=2}^n \mu_{ij} b_j\right) = \sum_{j=2}^n \mu_{ij} \alpha(b_j) \\ & = \sum_{j=2}^n\mu_{ij}(a_{1j} b_1 + \beta(b_j)) = \underbrace{\left(\sum_{j=2}^n \mu_{ij} a_{1j}\right)}_ {\displaystyle\sigma_i} + \sum_{j=2}^n \mu_{ij}\beta(b_j) \\ & = \sigma_i b_1+ \beta(\sum_{j=2}^n \mu_{ij} b_j) = \sigma_i b_1 + \beta(\tilde{b}_i) \\ & \underbrace{\in}_{\text{\ref{eq:2.2.22.2}}} \linspan{ b_1,\tilde{b}_2,\dots,\tilde{b}_i} = \linspan{ c_1, \dots, c_i } \end{align*} \end{itemize} \end{itemize} \end{itemize} \end{proof} \section{Jordan Normalform} \begin{defin} Eine $m\times m$ Matrix \[J_m(\lambda) := \begin{pmatrix} \lambda & 1 & 0 & \dots & 0 \\ 0 & \ddots & \ddots & \ddots & \vdots \\ \vdots & \ddots & \ddots & \ddots & 0 \\ \vdots & \ddots & \ddots & \ddots & 1 \\ 0 & \dots & \dots & 0 & \lambda \end{pmatrix}\] heißt \underline{Jordanblock} der Dimension $m$ zum Eigenwert $\lambda$.\\ Eine Matrix $A \in \K^{\nxn}$, die als Blockdiagonalmatrix aus Jordanblöcken besteht, heißt \underline{Jordanmatrix}. \\ $A \in \K^{\nxn}$ besitzt eine \underline{Jordan-Normalform} wenn $P\in\K^{\nxn}$ invertierbar existiert, sodass $P^{-1}AP$ Jordanmatrix ist.\\ $\alpha \in \homkv$ besitzt eine \underline{Jordan-Normalform} wenn eine Basis $B$ von $V$ existiert, sodass $ {}_{B} M(\alpha)_{B} $ Jordanmatrix ist.\\ B heißt \underline{Jordanbasis} zu $A/\alpha$. \end{defin} \subsubsection{Beispiel} \begin{itemize} \item Jede Diagonalmatrix ist Jordanmatrix \item $\begin{pmatrix}1\end{pmatrix}, \begin{pmatrix}1 & 1 \\ 0 & 1\end{pmatrix}, \begin{pmatrix}0 & 1 \\ 0 & 0\end{pmatrix}, \begin{pmatrix}1 & 1 & 0 \\ 0 & 1 & 0 \\ 0 & 0 & 2\end{pmatrix}, \xcancel{\begin{pmatrix} 1 & 1 \\ 0 & 2 \end{pmatrix}}$ \item \( \begin{pmatrix} \tl3\br \\ & \tl2 & 1 \\ & & 2\br\end{pmatrix} , \begin{pmatrix} \tl0 & 1 \\ 0 & 0\br \\ & & \tl-1\br \end{pmatrix}\) \end{itemize} Wir wollen zeigen, dass $\alpha/A$ genau dann eine Jordan-Normalform besitzt, wenn $\alpha/A$ triangulierbar ist. \subsubsection{Bemerkung} \begin{itemize} \item $\chi_{J_m(\lambda)}(\mu) = (\lambda - \mu)^m \implies \spec(J_m(\lambda)) = \{\lambda\}$ \\ $J_m(\lambda) - \lambda I = \begin{pmatrix} 0 & 1 & 0 & \dots & 0 \\ 0 & \ddots & \ddots & \ddots & \vdots \\ \vdots & \ddots & \ddots & \ddots & \vdots \\ \vdots & & \ddots & \ddots & 1 \\ 0 & \dots & \dots & 0 & 0 \end{pmatrix}$\\ $\implies \dim(\eig_{J_m(\lambda)}(\lambda)) = \dim(\ker(J_m(\lambda) - \lambda I)) = 1$ \\ $\implies m_g(\lambda) = 1$ und $m_a(\lambda) = m$. \item $J_m(0)^m = 0$, das heißt $J_m(0)$ ist \underline{nilpotent}. \begin{align*} & J_m(0)(e_i): \begin{cases} e_{i-1} & i \in \{2, \dots, m\} \\ 0 & \text{sonst}\end{cases} \\ & J_m(0)^l(e_i): \begin{cases}e_{i-l} & i \in \{l+1, \dots, m\} \\ 0 & \text{sonst}\end{cases} \end{align*} \end{itemize} \begin{defin} $\alpha \in \homkv$ oder $A\in \K^{\nxn}$ heißt \underline{nilpotent} (mit Index $m$) falls $\alpha^m = 0 / A^m = 0$ und $\forall l \in [m-1]: \alpha^l \neq 0 / A^l \neq 0$. \end{defin} \begin{lemma} \label{theo:2.3.3} Sei $\alpha \in \homkv, \dim(V)=n$ nilpotent mit Index $m$. Dann existiert eine Basis $B$ mit \begin{equation*} {}_B M(\alpha)_B = \begin{pmatrix} 0 & \delta_1 & & \\ & \ddots & \ddots & \\ & & \ddots & \delta_{n-1} \\ & & & 0 \end{pmatrix} \text{ und } \delta_i \in \{0, 1\} \forall i \in [n-1] \end{equation*} Das heißt ${}_B M(\alpha)_B$ ist blockdiagonal mit Jordanblöcken mit Eigenwerten $0$ \end{lemma} \begin{proof} Sei $V_i := \ker(\alpha^i)$. \\ Dies ergibt eine aufsteigende Kette von Unterräumen \begin{equation*} \underbrace{\{0\}}_{=V_0} \subseteq V_1 \subseteq \cdots \subseteq \underbrace{V_m}_{=V} \end{equation*} Wir bauen uns iterativ eine Basis für Komplemente $W_i$ mit $V_{i-1} \oplus W_i = V_i$. Sei also $B^{m-1}$ Basis von $V_{m-1}$. \\ $C^m = (c_1^m, \dots, c_{r_{m}})$ Basis von $W_m$ [das heißt $C^m$ ergänzt die Basis $B^{m-1}$ zu Basis von $V^m$]. \\ \underline{Behauptung} \begin{enumerate} [label=\arabic*)] \item $\alpha(C^m) \subseteq V_{m-1}$ \item $\alpha(C^m)$ linear unabhängig \item $\linspan{ \alpha(C^m) } \cap V_{m-2} = \{0\}$ \end{enumerate} \begin{proof}[Zwischenbeweis] \begin{itemize} \item[1)] folgt aus $\alpha(V_{i+1}) \subseteq \alpha(V_i)$ \item[3)] Sei $\sum\limits_{i}\mu_i \alpha(c_i^m) \in V_{m-2}$ \begin{align*} & \implies \alpha^{m-2} \left(\sum_{i}\mu_i \alpha(c_i^m)\right) = 0 \\ & \implies \alpha^{m-1} \left(\sum_{i} \mu_i \alpha(c_i^m)\right) = 0 \\ & \implies \sum \mu_i c_i^m \in V_{m-1} \\ & \underbrace{\implies}_{\mathclap{\substack{(c_i^m) \text{ liegen} \\ \text{im Komplement} \\ \text{von } V_{m-1}}}} \mu_i = 0, \forall i \implies \sum_{i} \mu_i \alpha(c_i^m) = 0 \end{align*} \item[2)] folgt aus 3) [da $0\in V_{m-2}$] \end{itemize} \end{proof} Es folgt, dass \[ \underbrace{V_{m-2} \oplus \linspan{ \alpha(C^m) } \oplus \overset{\linspan{ D^{m-1} }}{\linspan{C^{m-1}}}}_{V_{m-1}} \oplus \overset{\linspan{ D^m }}{\linspan{C^m}} = V \] Setze $D^m := C^m$ und definiere induktiv für $D^i \subseteq V_i$ die Menge $D^{i-1} := \alpha(D^i) \cup C^{i-1} \subseteq V_{i-1}$ sodass mit einer Basis $B^{i-2}$ von $V_{i-2}$ die Menge $B^{i-2} \cup D^{i-1}$ Basis von $V_{i-1}$ ist, also \[ V_{i-2} \oplus \underbrace{\linspan{ \alpha(D^i) } \oplus \linspan {C^{i-1}}}_ {\linspan{ D^i }} = V_{i-1} \text{$\leftarrow$ das geht nach obiger Behauptung} \] Nach Konstruktion ist $(D^1, \dots, D^m)$ Basis von $V$. Sie besteht aus folgenden Elementen: \begin{align*} \left. \begin{array}{lll} J_m(0) \to & \alpha^{m-1}(d_1^m), \dots, \alpha(d_1^m), & d_1^m \\ & & \vdots \\ J_m(0) \to & \alpha^{m-1}(d_{r_m}^m), \dots, \alpha(d_{r_m}^m), & d_{r_m}^m \end{array} \right\} \in V_m \\ \left. \begin{array}{lll} J_{m-1}(0) \to & \alpha^{m-1}(d_1^{m-1}), \dots, \alpha(d_1^{m-1}), & d_1^{m-1} \\ & & \vdots \\ J_{m-1}(0) \to & \alpha^{m-1}(d_{r_{m-1}}^{m-1}), \dots, \alpha(d_{r_{m-1}}^{m-1}), & d_{r_{m-1}}^{m-1} \end{array} \right\} \in V_{m-1} \\ \left. \begin{array}{lr} J_1(0) \to & d_1^1 \\ & \vdots \\ J_1(0) \to & d_{r_1}^1 \end{array} \right\} V_1 = \ker(\alpha) \end{align*} Wenn wir die Basiselemente von links nach rechts und von oben nach unten ordnen erhalten wir die gewünschte Gestalt. \end{proof} \subsubsection{Bemerkung} Angenommen \(\alpha - \lambda \id: V \to V\) nilpotent. Dann besitzt \(\alpha\) nach Lemma \ref{theo:2.3.3} Jordan-Normalform. \begin{defin} \label{theo:2.3.4} Sei \(V \K\)-Vektorraum, \(\dim(V) < \infty, \alpha \in \homkv\) und \(\lambda \in \spec(\alpha)\). Für \(l \in \mathbb{N}\) definiere \(V_{l, \lambda}:= \ker((\alpha - \lambda \id)^l)\) \end{defin} \subsubsection{Bemerkung} \begin{itemize} \item $\alpha - \lambda \id|_{V_{l, \lambda}} \in \homk(V_{l, \lambda}, V_{l, \lambda})$: \begin{align*} \text{zu Zeigen: } v\in V_{l, \lambda} & \implies \alpha(v) - \lambda v \in V_{l, \lambda}\text{, das heißt} \\ (\alpha - \lambda \id)^l v = 0 & \implies (\alpha - \lambda \id)^{l-1} (\alpha - \lambda \id) v = 0 \\ & \implies (\alpha - \lambda \id)(v) \in V_{l, \lambda} & \square \end{align*} \item Nach Lemma \ref{theo:2.3.3} gibt es also Basis von $V_{l, \lambda}$ bezüglich derer $\alpha - \lambda \id |_{V_{l, \lambda}}: V_{l, \lambda} \to V_{l, \lambda}$ Jordan-Normalform hat \end{itemize} \begin{lemma} \label{theo:2.3.5} Sei $V \K$-Vektorraum, $\dim(V) < \infty, \alpha \in \homkv$. Für $l\in\mathbb{N}$ sei $V_l := \ker(\alpha^l)$. Dann gilt $\alpha(V_l) \subseteq V_{l-1} \subseteq V_l$ für alle $l\in \mathbb{N}$ und es existiert genau ein $k\in \mathbb{N}_0$ mit \[ \{0\} = V_0 \subseteq V_1 \subseteq \cdots \subseteq V_k = V_{k+1} \text{ und } V_{l+1} = V_l, \forall l \ge k \] \end{lemma} \begin{proof} Da $\dim(V) < \infty$ muss es ein kleinstes $k$ mit $V_{k+1} = V_{k}$ geben. Angenommen $\exists l\ge k$ mit $V_{l+1} \neq V_l$. Sei $0\neq v\in V_{l+1} \setminus V_l$ $\implies 0 = \alpha^{l+1}(v) = \alpha^{k+1}(\alpha^{l-k}(v))$ und $0\neq \alpha^l(v) = \alpha^k (\alpha^{l-k}(v)) \implies 0\neq \alpha^{l-k}(v) \in V_{k+1}\setminus V_k$ \Lightning \end{proof} \begin{defin} Sei $V_{l, \lambda}$ wie in Definition \ref{theo:2.3.4} und $k$ wie in Lemma \ref{theo:2.3.5} Dann heißt \[ \widetilde{\eig_\alpha(\lambda)} := V_{k, \lambda} = V_{k+1, \lambda} \] \underline{verallgemeinerter Eigenraum} oder \underline{Hauptraum} von $\alpha$ zum Eigenwert $\lambda$. $v \in V_{l, \lambda} \setminus V_{l-1, \lambda}$ für $1 \le l \le k$ heißt \underline{verallgemeinerter Eigenvektor} der Ordnung $l$. \end{defin} \subsubsection{Idee} \begin{itemize} \item $\alpha|_{\widetilde{\eig_\alpha(\lambda)}}: \widetilde{\eig_\alpha(\lambda)} \to \widetilde{\eig_\alpha(\lambda)}$ hat Jordan-Normalform. Zerlege \begin{equation} \label{eq:2.3.6.1} V:= \widetilde{\eig_\alpha(\lambda_1)} \oplus \cdots \oplus \widetilde{\eig_\alpha(\lambda_r)} \end{equation} dann besitzt ganz $\alpha: V\to V$ Jordan-Normalform \item Sei $V= V_1 \oplus \cdots \oplus V_r$ und $\alpha \in \homkv$. Falls $\alpha(V_i) \subseteq V_i$ für alle $i \in [r]$, dann schreiben wir $\alpha = \alpha_1 \oplus \cdots \oplus \alpha_r$ mit $\alpha_i = \alpha|_{V_i} \forall i \in [r]$. Für $v= v_1 + \cdots + v_r, v_i \in V_i, \forall i \in [r]$ gilt also $\alpha(v) = \alpha_1(v_1) + \cdots + \alpha_r(v_r)$. Sei $B_i = \{b_1^i, \dots, b_{d_i}^i\}$ Basis von $V_i$ und $B = (B_1, \dots, B_r)$. Dann hat ${}_B M(\alpha)_B$ Blockdiagonalgestalt mit Blöcken ${}_{B_i} M(\alpha_i)_{B_i}$, das heißt \[ {}_B M(\alpha)_B = \begin{pmatrix} \overbrace{{}_{B_1} M(\alpha_1)_{B_1}}^{\in \K^{d_1 \times d_1}} & & 0 \\ & \ddots & \\ 0 & & \underbrace{{}_{B_r} M(\alpha_r)_{B_r}}_{\in\K^{d_r \times d_r}} \end{pmatrix} \] Insbesondere gilt $\chi_\alpha = \chi_{\alpha_1} \cdot \dots \cdots \chi_{\alpha_r}$ \item Da wir schon wissen, dass $\alpha|_{\widetilde{\eig_\alpha(\lambda_i)}}$ Jordan-Normalform hat folgt \\ Jordan-Normalform für $\alpha$ wenn \ref{eq:2.3.6.1} gezeigt werden kann. \end{itemize} \begin{satz} \label{theo:2.3.7} Sei $V \K$-Vektorraum mit $\dim(V) < \infty$ und $\alpha \in \homkv$ sodass $\chi_\alpha(\lambda) = (\lambda_1 - \lambda) \cdots (\lambda_r - \lambda)$ in Linearfaktoren zerfällt. Dann gilt $V = \widetilde{\eig_\alpha(\lambda_1)} \oplus \cdots \oplus \widetilde{\eig_\alpha(\lambda_r)}$ und insbesondere $\alpha = \alpha_1 \oplus \cdots \oplus \alpha_r$ mit $\alpha_i := \alpha|_{\widetilde{\eig_\alpha(\lambda_i)}} \in \homk(\widetilde{\eig_\alpha(\lambda_i)}, \widetilde{\eig_\alpha(\lambda_i)})$ \end{satz} \begin{proof} Induktion nach $\dim(V)$. \begin{itemize} \item[$n=1$:] \checkmark \item[$n-1 \mapsto n$:] Da $\chi_A$ in Linearfaktoren zerfällt besitzt es eine Nullstelle $\lambda \in \spec(\alpha)$. \begin{enumerate}[label=Fall \arabic*:] \item $\widetilde{\eig_\alpha(\lambda)} = V$ \\ \underline{Behauptung:} $\spec(\alpha) = \{\lambda\}$ \begin{proof}[Zwischenbeweis] Angenommen $\lambda' \neq \lambda$ und $\lambda' \in \spec(\alpha)$ und $v\in \eig_\alpha(\lambda')$. \\ $\implies (\alpha - \lambda \id) (v) = \alpha(v) - \lambda'v + (\lambda' - \lambda) v = (\lambda' - \lambda)(v)$ \\ $\implies (\alpha - \lambda \id)^l (v) \neq 0,\forall l \in \mathbb{N}$\Lightning \\ Daraus folgt das gewünschte Resultat \end{proof} \item $\widetilde{\eig_\alpha(\lambda)} \neq V$. Sei $k$ minimal mit $\ker(\alpha - \lambda - \id)^k = \widetilde{\eig_\alpha(\lambda)}$ [Lemma \ref{theo:2.3.5}] Setze $V_1 := \widetilde{\eig_\alpha(\lambda)}, V_2 := \im(\alpha - \lambda \id)^k$. \\ \underline{Behauptung:} \begin{enumerate}[label=(\roman*)] \item $\alpha(V_i) \subseteq V_i, i \in \{1, 2\}$ \item $V = V_1 \oplus V_2$ \end{enumerate} \begin{proof}[Zwischenbeweis] \begin{enumerate}[label=(\roman*)] \item Wir zeigen $(\alpha - \lambda \id)(V_i) \subseteq V_i$. \begin{itemize} \item[$i=1$:] Sei $v\in V_1 = \ker(\alpha - \lambda \id)^k$. Dann gilt klarerweise $(\alpha - \lambda \id)(v) \in \ker(\alpha - \lambda \id)^k \checkmark$ \item[$i=2$:] Sei $v \in \im(\alpha - \lambda \id)^k$, also $v = (\alpha - \lambda \id)^k (w)$ $\implies (\alpha - \lambda \id)(v) = (\alpha - \lambda \id)^k (\alpha - \lambda \id)(w) \in \im(\alpha - \lambda \id)^k \checkmark$ \end{itemize} \item Es gilt $\dim(V) = \dim(V_1) + \dim(V_2)$ nach der Dimensionsformel. Es genügt also zu zeigen, dass $V_1 \cap V_2 = \{0\}$. \\ Sei $v\in V_1 \cap V_2$ \begin{align*} & \underbrace{\implies}_{v\in V_2} \exists w\in V: v = (\alpha - \lambda \id)^k(w) \\ & \underbrace{\implies}_{v\in V_1} (\alpha - \lambda \id)^{2k}(w) = 0 \\ & \implies w \in V_{2k, \lambda} \setminus V_{k, \lambda} \underbrace{\implies}_{\mathclap{\text{Lemma \ref{theo:2.3.5}}}} w = \{0\}\checkmark \end{align*} \end{enumerate} \end{proof} \end{enumerate} Es folgt $V = \underbrace{\widetilde{\eig(\lambda)}}_{V_1} \oplus V_2, \dim(V_2) < n$ und \\ $\alpha = \alpha_1 \oplus \alpha_2, \alpha_i := \alpha|_{V_i}, i\in\{1, 2\}$. Es folgt $\chi_\alpha = \chi_{\alpha_1} \cdot \chi_{\alpha_2}$, also zerfällt $\chi_{\alpha_2}$ in Linearfaktoren. Daher können wir die Induktionsvorraussetzung anwenden, was das gewünschte Resultat lierfert. \end{itemize} \end{proof} \begin{satz} Sei $V \K$-Vektorraum, $\dim(V) < \infty$ und $\alpha \in \homkv$ sodass $\chi_A$ in Linearfaktoren zerfällt. Dann besitzt $\alpha$ Jordan-Normalform. \end{satz} \begin{proof} Zerlege nach Satz \ref{theo:2.3.7} $V = \widetilde{\eig_\alpha(\lambda_1)} \oplus \cdots \oplus \widetilde{\eig_\alpha(\lambda_r)}$ und \\ $\alpha = \alpha \oplus \cdots \oplus \alpha_r$. Da $\widetilde{\eig_\alpha(\lambda_i)} = \ker(\alpha - \lambda_i \id)^{k_i}$ ist $\alpha_i - \lambda_i \id := \alpha|_{\widetilde{\eig_\alpha(\lambda_i)}} - \lambda \id|_{\widetilde{\eig_\alpha(\lambda_i)}} $ nilpotent. Nach Lemma \ref{theo:2.3.3} gibt es eine Basis $B_i$ von $\widetilde{\eig_\alpha(\lambda_i)}$ sodass ${}_{B_i} M(\alpha_i)_{B_i}$ Jordan-Normalform hat. Es folgt mit $B= (B_1, \dots, B_r)$ dass \\ ${}_B M(\alpha)_B = \begin{pmatrix} {}_{B_1} M(\alpha_1)_{B_1} & & \\ & \ddots & \\ & & {}_{B_r}M(\alpha_r)_{B_r} \end{pmatrix}$ Jordanmatrix ist. \end{proof} \subsubsection{Berechnung der Jordan-Normalform} \begin{enumerate} \item Berechne $\spec(\alpha) = \{ \lambda_1, \dots, \lambda_r \}$ \item \begin{enumerate}[label=\alph*)] \item Haupträume berechnen: Finde $k$ minimal mit \[ \ker(\alpha - \lambda \id)^{k+1} = \ker(\alpha - \lambda \id)^k =: V_\lambda \] \item Für $1 \le l \le k$ bestimme $B_l = \{ b_1^l, \dots, b_{r_l}^l\}$, sodass $(B_1, \dots, B_l)$ Basis von $\ker(\alpha - \lambda \id)^l$. \end{enumerate} \item \begin{enumerate}[label=\alph*)] \item Setze zunächst $v_i^k = b_i^k, i = 1, \dots, r_k$. $D_k := (v_1^k, \dots, v_{r_k}^k)$ \\ Setze $v_i^{k-1} := (\alpha - \lambda \id)(v_i^k) \in \linspan{ B_{k-1} }, i = 1, \dots, r_k$ \\ Ergänze gegebenenfalls $(v_1^{k-1}, \dots, v_{r_k}^{k-1}, v_{r_{k+1}}^{k-1}, \dots, v_{r_{k-1}}^{k-1})=:D_{k-1}$, sodass \\ $\linspan{ D_{k-1} } = \linspan B_{k-1}$ \item Führe 3a) iterativ aus. \\ Setze $v_i^{l-1} := (\alpha - \lambda \id)(v_i^l), i = 1, \dots, r_l$ \\ Ergänze gegebenenfalls $v_1^{l-1}, \dots, v_{r_l}^{l-1}, v_{r_{l+1}}^{l-1}, \dots, v_{r_{l-1}}^{l-1} =:D_{l-1}$, sodass $\linspan{ D_{l-1} } = \linspan{B_{l-1}}$ \end{enumerate} \item Sei $B_\lambda = (D_1, \dots, D_k) \implies {}_{B_\lambda} M(\alpha|_{v_\lambda})_{B_\lambda}$ hat Jordan-Normalform mit Eigenwert $\lambda$. \item Setze $B = (B_{\lambda_1}, \dots, B_{\lambda_r}) \implies {}_B M(\alpha)_B$ hat Jordan-Normalform. \end{enumerate} \subsubsection{Beispiel} \begin{align*} & A= \begin{pmatrix} 1 & 0 & 2 & 3 & 4 \\ 0 & 1 & 0 & -2 & -3 \\ 0 & 0 & 1 & 0 & 2 \\ 0 & 0 & 0 & 1 & -1 \\ 0 & 0 & 0 & 0 & 1 \end{pmatrix} , \chi_A(\lambda) = (\lambda - 1)^5 \\ & (A - 1\cdot I) = \begin{pmatrix} 0 & 0 & 2 & 3 & 4 \\ 0 & 0 & 0 & -2 & -3 \\ 0 & 0 & 0 & 0 & 2 \\ 0 & 0 & 0 & 0 & -1 \\ 0 & 0 & 0 & 0 & 0 \end{pmatrix} \implies \ker(A - I) = \linspan{ ( \underbrace{e_1, e_2}_{B_1} ) } \\ & (A-I)^2 = \begin{pmatrix} 0 & 0 & 0 & 0 & 1 \\ 0 & 0 & 0 & 0 & 2 \\ 0 & 0 & 0 & 0 & 0 \\ 0 & 0 & 0 & 0 & 0 \\ 0 & 0 & 0 & 0 & 0 \end{pmatrix} \implies \ker((A-I)^2) = \linspan{ (\underbrace{e_1, e_2}_{B_1}, \underbrace{e_3, e_4}_{B_2}) } \\ & (A-I)^3 = 0 \implies \ker((A-I)^3) = \linspan{(\underbrace{e_1, e_2}_{B_1}, \underbrace{e_3, e_4}_{B_2}, \underbrace{e_5}_{B_3}) } \\ & B_1 = (e_1, e_2), B_2 = (e_3, e_4), B_3 = (e_5) \end{align*} \begin{align*} \begin{rcases} v_1^3 = e_5 \end{rcases} D_3 \\ \begin{rcases} v_1^2 = (A-1I)(v_1^3) = \begin{pmatrix}-4 \\ -3 \\ 2 \\ -1 \\ 0\end{pmatrix} \\ v_2^2 = e_4 \end{rcases} D_2 \\ \begin{rcases} v_1^1 = (A - I)(v_1^2) = \begin{pmatrix}1 \\ 2 \\ 0 \\ 0 \\ 0 \end{pmatrix} \\ v_2^1 = (A - I)(v_2^2) = \begin{pmatrix}3 \\ -2 \\ 0 \\ 0 \\ 0 \end{pmatrix} \end{rcases} D_1 \end{align*} \begin{align*} (\overset{\mathrlap{\rotatebox{30}{\scriptsize$\in\ker(A-I)$}}}{v_1^1} \underset{\mathclap{\substack{\rotatebox{180}{$\curvearrowright$} \\ A-I}}}{,} v_1^2 \underset{\mathclap{\substack{\rotatebox{180}{$\curvearrowright$} \\ A-I}}}{,} v_1^3, \overset{\mathrlap{\rotatebox{30}{\scriptsize$\in\ker(A-I)$}}}{v_1^1} \underset{\mathclap{\substack{\rotatebox{180}{$\curvearrowright$} \\ A-I}}}{,} v_2^2) = B, {}_B M(A)_B = \begin{pmatrix} 1 & 1 & 0 & 0 & 0 \\ 0 & 1 & 1 & 0 & 0 \\ 0 & 0 & 1 & 0 & 0 \\ 0 & 0 & 0 & 1 & 1 \\ 0 & 0 & 0 & 0 & 1 \end{pmatrix} \\ P = \begin{pmatrix} 1 & 4 & 0 & 3 & 0 \\ 2 & -3 & 0 & -2 & 0 \\ 0 & 2 & 0 & 0 & 0 \\ 0 & 1 & 0 & 0 & 1 \\ 0 & 0 & 1 & 0 & 0 \end{pmatrix} \implies P^{-1} A P = \begin{pmatrix} \tl1 & 1 & 0 & 0 & 0 \\ 0 & 1 & 1 & 0 & 0 \\ 0 & 0 & 1\br & 0 & 0 \\ 0 & 0 & 0 & \tl1 & 1 \\ 0 & 0 & 0 & 0 & 1\br \end{pmatrix} \end{align*} \chapter{Euklidische und Unitäre Vektorräume} \subsubsection{Motivation} Wir wollen Geometrie betreiben und Längen beziehungsweise Winkel messen können. \subsubsection{Länge} \begin{tikzpicture}[scale=4] \draw [-latex, very thick] (0, 0) -- (1.3, 1); \draw [dashed] (0, 0) -- (1.3, 0) -- (1.3, 1); \node [below] at (0.65, 0) {$x_2 - x_1$}; \node [right] at (1.3, 0.5) {$y_2 - y_1$}; \node [below left] at (0, 0) {$(x_1, y_1)$}; \node [above right] at (1.3, 1) {$(x_2, y_2)$}; \draw (1.1, 0) -- (1.1, 0.2) -- (1.3, 0.2); \draw [fill] (0, 0) circle [radius=0.02]; \end{tikzpicture} \( \R^2: P_1 = (x_1, y_1), P_2 = (x_2, y_2) \) \\ \( d(P_1, P_2) = \sqrt{(x_2 - x_1)^2 + (y_2 - y_1)^2} = \abs{ \vect{P_1P_2} } \) \subsubsection{Winkel} \begin{tikzpicture}[scale=0.7] \coordinate (a) at (0, 0); \coordinate (b) at (5, 6); \coordinate (c) at (8, 4); \draw [fill] (a) circle [radius=0.07]; \draw [very thick, ->] (a) -- (b); \draw [very thick, ->] (a) -- (c); \node [below left] at (a) {$p$}; \node [right] at (b) {$v_2$}; \node [right] at (c) {$v_2$}; \draw pic [draw, thick, angle radius=3cm, pic text=$\alpha$] {angle=c--a--b}; \end{tikzpicture} \\ $v_1 = (u_1, w_1), v_2 = (u_2, w_2), v= (u, w)$ \\ $\cos(\alpha) = \dfrac{u_1 u_2 + w_1 w_2}{\abs{w_1} \abs{w_2}}$ $\abs{ v } = \sqrt{u^2 + w^2}$ \\ $v_1 \cdot v_2 = u_1 u_2 + w_1 w_2$ skalares Produkt \\ $\implies d(P_1, P_2) = \sqrt{\vect{P_1 P_2} \cdot \vect{P_1 P_2}}, \cos(\sphericalangle{v_1 v_2}) = \dfrac{v_1 v_2}{\abs{v_1}\abs{v_2}}$ \section[Skalarprodukte und Hermitesche Formen]{Skalarprodukte und Hermitesche \\ Formen} Zunächst sei \( \K = \R \) \begin{defin} Sei $V$ ein $\R$-Vektorraum und $\beta: V \times V \to \R$. $\beta$ heißt \begin{itemize} \item \underline{bilinear} (Bilinearform) wenn $\forall u, v, w\in V, \lambda \in \R$: \begin{align*} \beta(u+v, w) = \beta(u, w) + \beta(v, w), \\ \beta(u, v+w)=\beta(u, v) + \beta(u, w), \\ \beta(\lambda u, v) = \lambda \beta(u, v) = \beta(u, \lambda v) \end{align*} \item \underline{symmetrisch} wenn $\forall u, v \in V: \beta(u, v) = \beta(v, u)$ \item \underline{positiv definit} wenn $\forall v \in V\setminus\{0\}: \beta(v, v) > 0$ \item \underline{skalares Produkt} wenn $\beta$ symmetrisch, positiv definit (spd) und bilinear ist. \end{itemize} \end{defin} \subsubsection{Bemerkung} $v=0\in V \implies 0 \cdot v = v \implies \beta(v, v) = \beta(0 \cdot v, v) = 0 \cdot \beta(v, v) = 0$ \subsubsection{Beispiele} \begin{itemize} \item Sei $V = \R^n$ und $v= (v_1, \dots, v_n), w= (w_1, \dots, w_n)$ \\ $\beta_1(v, w) = \sum\limits_{i=1}^n v_i w_i= v^Tw$ ist symmetrische positiv definite Bilinearform. \item Sei $\dim(V) = n$ und $B=(b_1, \dots, b_n)$ Basis \\ Sei für $v, w \in V: \begin{cases} {}_B \Phi(v) = (v_1, \dots, v_n) \\ {}_B \Phi(w) = (w_1, \dots, w_n)\end{cases}$ \\ $\beta_2(v, w) = \sum\limits_{i=1}^n v_i w_i = \beta_1({}_B \Phi(v), {}_B \Phi(w))$ ist spd. \item $V= \R^2, v= \begin{pmatrix}v_1\\v_2\end{pmatrix}, w= \begin{pmatrix}w_1\\w_2\end{pmatrix}, A = \begin{pmatrix} 4 & -2 \\ -2 & 3 \end{pmatrix}$ \\ $\beta_3(v, w) = v^T A w \in \R$ \\ symmetrisch, weil \[ \beta_3(v, w) = \beta_3(v, w)^T = (v^T A w)^T = w^T A^T v = w^T A v = \beta_3(w, v) \checkmark \] $\beta_3(u, v) = 4v_1w_1 - 2v_1w_2 - 2v_2w_1 + 3v_2 w_2 \implies \beta(v, v) = (2 v_1 - v_2)^2 + 2 v_2^2 = 0 \implies v_2 = 0 \implies (2v_1)^2 = 0 \implies v_1 = 0$ \item Sei $a, b \in \R, a < b, V = \{f:[a, b] \to \R: f \text{ stetig}\}$ \\ Sei $h \in V: h(t) > 0 \forall t \in [a, b]$ \begin{align*} & \beta_4(f, g) = \int_a^b f(t) g(t) h(t) dt \text{ bilinear, symmetrisch} \\ & \beta_4(f, f) = \int_a^b \abs{ f(t) } ^2 h(t) dt = 0 \implies f= 0 \end{align*} \end{itemize} \begin{defin} Ein Vektorraum mit skalarem Produkt heißt \underline{Euklidischer Raum}.\\ Man schreibt oft $u \cdot v, \inner uv$ anstatt $\beta(u, v)$. \end{defin} Nun sei $\K = \C$ \begin{defin} Sei $V$ ein \C-Vektorraum und $\beta: V \times V \to \C$. $\beta$ heißt \underline{hermitesche Form} wenn für alle $u, v, w \in V, \lambda \in \C$: \begin{enumerate}[label=\roman*)] \item $\beta(u + v, w) = \beta(u, w) + \beta(v, w)$ \item $\beta(\lambda u, v) = \lambda \beta(u, v)$ \item $\beta(u, v) = \overline{\beta(u, v)}$ \end{enumerate} \end{defin} \begin{lemma} \label{theo:3.1.4} Sei $\beta$ hermitesche Form \begin{enumerate}[label=\alph*)] \item $\beta(u, v + w) = \beta(u, v) + \beta(u, w)$ \item $\beta(u, \lambda v) = \overline{\lambda} \beta(u, v)$ \item $\beta(u, u) \in \R$ \end{enumerate} \end{lemma} \begin{proof} \begin{enumerate}[label=\alph*)] \item $\beta(u, v+w) \overset{\text{iii}}{=} \overline{\beta(v+ w, u)} \overset{\text{i}}{=} \overline{\beta(v, u)} + \overline{\beta(w, u)} \overset{\text{iii}}{=}\beta(u, v) + \beta(u, w) \checkmark$ \item $\beta(u, \lambda v) \overset{\text{iii}}{=} \overline{\beta(\lambda v, u)} \overset{\text{ii}}{=} \overline{\lambda}\cdot\overline{\beta(v, u)} \overset{\text{iii}}{=} \overline{\lambda} \beta(u, v)$ \item $z = \overline{z} \iff z \in \R$ \\ $\beta(u, u) \overset{\text{iii}}{=} \overline{\beta(u, u)} \implies \beta(u, u) \in \R$ \end{enumerate} \end{proof} \begin{defin} Sei $\beta$ hermitesche Form. \begin{itemize} \item $\beta$ heißt \underline{positiv definit} wenn $\forall v \in V\setminus\{0\}: \underbrace{\beta(v, v)}_{\in\R} > 0$ \item Eine positiv definite hermitesche Form heißt \underline{skalares Produkt} \item Ein komplexer Vektorraum mit einem skalaren Produkt heißt \underline{unitärer} \underline{Raum}. \end{itemize} \end{defin} \subsubsection{Beispiel} $V = \C^n, u = (u_1, \dots, u_n), v = (v_1, \dots, v_n)$ \\ $u \cdot v = \sum\limits_{i=1}^n u_i \overline{v_i}$ ist skalares Produkt \par Wir zeigen nun, dass jeder euklidische Vektorraum in einen unitären Vektorraum eingebettet werden kann. \newpage \begin{defin} Sei $V$ ein \R-Vektorraum. \begin{align*} & V_\C := \{ (u, v): u, v\in V \} \text{ [Schreibe $(u, v) =: u + \overset{\mathclap{\substack{i^2 = -1 \\ |}}}{i} \cdot v$]} \\ & (u_1, v_1) + (u_2, v_2) := (u_1 + u_2, v_1 + v_2) \text{ Addition} \\ & \lambda = (\gamma + i \delta) \in \C, \lambda \cdot (u, v) = (\gamma u - \delta v, \delta u + \gamma v) \text{ skalare Multiplikation} \\ & \lambda(u + iv) = (\gamma + i \delta) (u + iv) = \gamma u + i \gamma v + i \delta u - \delta v \\ & \; \; =(\gamma u - \delta v) + i (\gamma v + \delta u) \\ & \implies (V_\C, +, \cdot) \text{ ist \C-Vektorraum} \end{align*} $V_\C$ heißt die \underline{komplexe Erweiterung} von V. \end{defin} \[ \text{Einbettung: }\iota: \begin{cases}V \to V_\C \\ v \mapsto (v, 0) = v + i\cdot 0 \end{cases} \] \begin{lemma} \begin{enumerate}[label=\alph*)] \item $V$ ist durch die Einbettung $v \overset{\iota_V}{\mapsto} (v, 0)$ \dq in $V_\C$ enthalten\dq, das heißt $\iota_V$ ist injektiv. \item Seien $V, W$ \R-Vektorräume, $\alpha \in \Hom_\R(V, W)$. Dann existiert eine eindeutige komplexe Erweiterung $\alpha_\C \in \Hom_\C(V_\C, W_\C)$ mit \[ \forall v \in V: \alpha_\C(\iota_V(v)) = \iota_W(\alpha(v)) \] \end{enumerate} \end{lemma} \begin{proof} \begin{enumerate}[label=\alph*)] \item $\iota_V$ ist linear \checkmark \\ $\iota_V(v) = (0, 0) \implies v = 0$ (injektiv) \item Sei $\alpha_\C$ so eine Fortsetzung \\ $\alpha_\C(u + iv) = \alpha_\C(u) + i \alpha_\C(v) = \alpha(u) + i\alpha(v)$ \\ $\alpha_\C((u, v)) = (\alpha(u), \alpha(v))$ Dadurch ist $\alpha_\C$ eindeutig bestimmt! \end{enumerate} \end{proof} \begin{defin} $\alpha_\C$ heißt die komplexe Forsetzung von $\alpha$ \end{defin} Auch skalare Produkte können eindeutig fortgesetzt werden. \begin{satz} Sei $(V, \beta)$ euklidischer \R-Vektorraum. Dann existiert genau eine hermitesche Form $\beta_\C$ auf $V_\C$, welche $\beta$ fortsetzt: \[ \forall v, w \in V: \beta_\C(\iota_V(v), \iota_V(w)) = \beta(v, w) \] \end{satz} \begin{proof} Ein solches $\beta_\C$ muss erfüllen, dass \begin{align*} \beta_\C(u_1 + i v_1, u_2 + i v_2) & = \beta_\C(u_1, u_2 + i v_2) + i \beta_\C(v_1, u_2+iv_2) \\ & = \beta_\C(u_1, u_2) + i \beta_\C(v_1, u_2) \underset{\mathclap{\substack{| \\\text{\ref{theo:3.1.4} b)}}}}{-} i \beta_\C(u_1, v_2) + \beta_\C(v_1, v_2) \\ & = \beta(u_1, u_2) + \beta(v_1, v_2) + i(\beta(v_1, u_2) - \beta(u_1, v_2)) \end{align*} und dadurch ist $\beta_\C$ eindeutig bestimmt. \end{proof} \begin{satz} [Cauchy-Schwarz] \label{theo:3.1.10} Für $u, v$ in einem euklidischen/unitären Vektorraum $V$ gilt \[ \abs{ \inner uv } ^2 \le \inner uu \inner vv \] Gleichheit gilt genau wenn $u, v$ linear abhängig sind. \end{satz} \begin{proof} $v=0 \checkmark$ \\ $v \neq 0 \implies \inner vv >0$ \\ Sei $\lambda \in \C \implies$ \begin{align*} 0 & \le \inner{u - \lambda v}{ u - \lambda v } \\ & = \inner u {u - \lambda v} - \lambda \inner v {u-\lambda v} \\ & = \inner uu - \overline{\lambda} \inner uv - \lambda \inner vu + \underbrace{\lambda \overline{\lambda}}_{=\abs{\lambda}^2} \inner vv \end{align*} Sei $\lambda := \dfrac{\inner uv}{\inner vv}, \overline{\lambda} = \dfrac{\overline{\inner uv }}{\overline{\inner vv }} =\dfrac{\inner vu}{\inner vv}$, so folgt \begin{align*} 0 & \le \inner uu - \frac{\inner vu \inner uv }{\inner vv } - \frac{\inner uv \inner vu }{\inner vv } + \frac{\cancel{\inner vv } \inner uv \inner vu } {\inner{v}{v}^{\cancel{2}}} \\ & = \inner uu - \frac{\abs{ \inner uv }^2}{\inner vv } \\ & \implies 0 \le \inner uu \inner vv - \abs{ \inner uv } ^2 \\ & \implies \inner uu \inner vv \ge \abs{ \inner uv }^2. \end{align*} Gleichheit gilt, wenn $\inner{u - \lambda v}{u - \lambda v} = 0$, also $u, v$ linear abhängig. \end{proof} \begin{defin} Man nennt \begin{itemize} \item $\norm v := \sqrt{\inner vv }$ die \underline{Länge} oder die \underline{Norm} von $v \in V$. \item $\cos(\sphericalangle v w) := \dfrac{\inner vw }{\norm v \norm w }$ der Kosinus des \underline{Winkels} zwischen $v, w \in V$. \\ (Wegen Satz \ref{theo:3.1.10} ist $\cos(\sphericalangle v w) \le 1$ und damit auch $\sphericalangle v w$ wohldefiniert!) \item $v\in V$ heißt \underline{normiert} wenn $\norm v = 1$ \end{itemize} \end{defin} \begin{satz} $\norm \cdot $ ist eine \underline{Norm}, das heißt \begin{enumerate}[label=\alph*)] \item $\norm v \ge 0$ \item $\norm v = 0 \implies v = 0$ \item $\norm {\lambda v} = \abs{ \lambda } \norm v $ \item $\norm {v + w} \le \norm v + \norm w $ (Dreiecksungleichung) \end{enumerate} \end{satz} \begin{proof} \begin{enumerate}[label=\alph*)] \item $\norm v = (\underbrace{\inner vv }_{\in [0, \infty)})^\frac12 \ge 0$ \item $\norm v = 0 \implies \norm{v}^2 = 0 \implies \inner vv = 0 \implies v = 0$ \item $\norm{\lambda v}^2 = \inner{\lambda v}{\lambda v}= \norm{\lambda}^2 \inner vv = \abs{ \lambda }^2 \norm{v}^2$ \item \begin{align*} \norm{u + v}^2 & = \inner{u + v}{u +v} \\ & = \inner u{u+v} + \inner v{u+v} = \inner uu + \inner uv + \inner vu + \inner vv \\ & = \inner uu + \inner uv + \overline{\inner uv } + \inner vv \\ & = \inner uu + 2 \Re(\inner uv ) + \inner vv \\ & \le \inner uu + 2 \abs{ \inner uv } + \inner vv \\ & \le \inner uu + 2 \norm u \norm v + \inner vv \\ & = \norm{u}^2 + 2 \norm u \norm v + \norm{v}^2 = (\norm u + \norm v )^2 \end{align*} \end{enumerate} \end{proof} \begin{defin} Sei $V = (v_1, \dots, v_k)$ mit $\forall i \in [k]: v_i \neq 0$. \begin{itemize} \item $v, w$ heißen \underline{orthogonal}, wenn $\inner vw = 0$ [schreibe auch $v \bot w$] \item $V$ heißt \underline{Orthogonalsystem} (OS), wenn $\forall i, j \in [k], i\neq j: v_i \bot v_j$ \item $V$ heißt \underline{Orthonormalsystem} (ONS), wenn $V$ ein Orthogonalsystem ist und $\forall i \in [k]: \norm{v_i}= 1$ \item $V$ heißt \underline{Orthogonalbasis} (OB), wenn $V$ ein Orthogonalsystem und eine Basis ist. \item $V$ heißt \underline{Orthonormalbasis} (ONB), wenn $V$ ein Orthonormalsystem und eine Basis ist. \end{itemize} \end{defin} \begin{satz} Sei $(v_1, \dots, v_k)$ ein Orthogonalsystem. Dann ist $(v_1, \dots, v_k)$ linear \\unabhängig. \end{satz} \begin{proof} Angenommen $\lambda_1 v_1 + \dots + \lambda_k v_k = 0$ \[ \forall i \in [k]: 0 = \lambda_1 \underbrace{\inner {v_1}{v_i}}_{=0} + \dots + \lambda_i \underbrace{\inner {v_i}{v_i}}_{\neq0} + \dots + \lambda_k \underbrace{\inner {v_k}{v_i}}_{=0} = \lambda_i \underbrace{\norm{v_i}^2}_{\neq 0} \] $\implies \lambda_i = 0$ \end{proof} \begin{satz} \label{theo:3.1.15} Sei $B=(b_1, \dots, b_n)$ Orthonormalbasis von $V, n\in \mathbb{N}\cup \{\infty\}$. Dann gilt für alle $v, w \in V$ und $(\lambda_1, \dots, \lambda_n) = {}_B \Phi(v), (\mu_1, \dots, \mu_n) = {}_B\Phi(w)$: \[ \inner vw = \sum_{i=1}^n \lambda_i \overline{\mu_i} \] Weiters gilt $\lambda_i = \inner{v}{b_i}, b_i^*(v) = \inner v {b_i}$ \end{satz} \begin{proof} \begin{align*} & \inner{b_i}{b_j} = \delta_{ij} \\ & \begin{rcases}v = \sum_{i=1}^n \lambda_i b_i \\ w = \sum_{i=1}^n \mu_i b_i\end{rcases} \implies \inner vw = \sum_{i, j = 1}^n \inner{\lambda_i b_i}{\mu_j b_j} = \sum_{i, j = 1}^n \lambda_i \overline{\mu_j} \inner{b_i}{b_j} = \sum_{i=1}^n \lambda_i \overline{\mu_i} \\ & {}_B \Phi(b_i) = (0, \dots, \overset{i}{1}, \dots, 0) \implies \inner v{b_i} = \sum_{j=1}^n \lambda_j \delta_{ij} = \lambda_i \end{align*} \end{proof} \begin{satz} [Gram-Schmidt Orthonormalisierungsverfahren] \label{theo:3.1.16} Sei $(a_1, a_2, \dots) \subseteq V$ linear unabhängig. Dann existiert genau ein Orthonormalsystem $(b_1, b_2, \dots)$ mit \begin{enumerate}[label=\roman*)] \item $\forall k: \linspan{ a_1, \dots, a_k } = \linspan{b_1, \dots, b_k} =: U_k$ \item Die Basistransformationsmatrix $M_k$ zwischen der Basen $(a_1, \dots, a_k)$ und $(b_1, \dots, b_k)$ von $U_k$ hat positive Determinante. \end{enumerate} \end{satz} \begin{proof} $b_1, b_2, \dots$ werden induktiv definiert. \begin{itemize} \item $b_1 = \frac{a_1}{\norm{a_1}}, M_1 = \begin{pmatrix}\frac{1}{\norm{a_1}}\end{pmatrix}$ \\ Eindeutigkeit: Sei $\tilde b_1$ mit i), ii) $\implies \tilde b_1 = c \cdot a_1, 1 = \norm{\tilde b_1} = \norm{c \cdot a_1} = \abs{ c } \norm{a_1}$ \\ $ \implies \abs{ c } = \dfrac{1}{\norm{a_1}} \implies \tilde M_k =(c)$ \item $(b_1, \dots, b_n)$ schon konstruiert mit i), ii) \\ Sei $c_{n+1} := a_{n+1} - \sum\limits_{j=1}^n \inner{a_{n+1}}{b_j} b_j$ \begin{align*} & \forall i \in [n]: \inner{c_{n+1}}{b_i} = \inner{a_{n+1}}{b_i} - \sum\limits_{j=1}^n \inner{a_{n+1}}{b_j} \underbrace{\inner{b_j}{b_i}}_{\delta_{ij}} \\ & = \inner{a_{n+1}}{b_i} - \inner{a_{n+1}}{b_i} = 0 \implies c_{n+1} \bot \linspan{ b_1, \dots, b_n } \end{align*} $b_{n+1} = \dfrac{c_{n+1}}{\norm{c_{n+1}}} \implies (b_1, \dots, b_{n+1})$ Orthonormalsystem mit \\ $\linspan{ b_1, \dots, b_n } = \linspan{a_1, \dots, a_n}$ \begin{align*} & b_1 = \mu_{11} a_1 \\ & b_2 = \mu_{21} a_1 + \mu_{22} a_2 \\ & b_3 = \mu_{31} a_1 + \mu_{32} a_2 + \mu_{33} a_3 \\ & \vdots \\ & b_n = \mu_{n1} a_1 + \dots + \mu_{nn} a_n \\ & b_{n+1} = \mu_{n+1 1} a_1 + \dots + \mu_{n+1 n} a_n + \dfrac{1}{\norm{c_{n+1}}} a_{n+1} \\ & \implies \det(\mu_{ij}) = \det(M_n) \cdot \dfrac{1}{\norm{c_{n+1}}} > 0 \end{align*} Eindeutigkeit: Sei $\tilde b_{n+1}$ ein weiterer Vektor mit i), ii) \begin{align*} & \implies \tilde b_{n+1} = \mu_1 b_1 + \dots + \mu_n b_n + \mu b_{n+1} \\ & \forall i \in [n]: 0 = \inner{\tilde b_{n+1}}{b_i} = \mu_i \implies \tilde b_{n+1} = \mu b_{n+1} \\ & 1 = \norm{\tilde b_{n+1}} = \abs{\mu} \norm{b_{n+1}} = \abs{\mu} \implies \abs{\mu} = 1 \\ & \det(\tilde M_{n+1}) = \det(M_n) \cdot \mu > 0 \implies \mu = 1 \land \tilde b_{n+1} = b_{n+1} \end{align*} \end{itemize} \end{proof} \begingroup \allowdisplaybreaks \subsubsection{Veranschaulichung im $\R^2$} \begin{tikzpicture}[scale=3.79] \tikzmath{ \a1 = 3; \a2 = 1; \a3 = 2; \a4 = 2; \norma1 = sqrt((\a1 * \a1 + \a2 * \a2)); \normeda1 = \a1 / \norma1; \normeda2 = \a2 / \norma1; \innerprod = (\normeda1 * \a3) + (\normeda2 * \a4); \c1 = \a3 - (\innerprod * \normeda1); \c2 = \a4 - (\innerprod * \normeda2); \t1 = (\innerprod * \normeda1); \t2 = (\innerprod * \normeda2); \normc = sqrt((\c1 * \c1 + \c2 * \c2)); \b3 = \c1 / \normc; \b4 = \c2 / \normc; } \draw [->] (0, 0) --node[above]{$a_1$} (\a1, \a2); \draw [->] (0, 0) --node[above]{$a_2$} (\a3, \a4); \draw [->, blue, thick] (0, 0) --node[below right]{$\inner{a_2}{b_1}b_1$} (\t1, \t2); \draw [->, violet, very thick] (0, 0) --node[below right]{$\frac{a_1}{\norm{a_1}}=:b_1$} (\normeda1, \normeda2); \draw [->, magenta, thick] (0, 0) --node[left]{$c_2:=a_2 - \inner{a_2}{b_1}b_1$} (\c1, \c2); \draw [->, teal, very thick] (0, 0) --node[right]{$\frac{c_2}{\norm{c_2}}=:b_2$} (\b3, \b4); \end{tikzpicture} \subsubsection{Beispiel} $V = \R^4, a_1 = \begin{pmatrix} 4 \\ 2 \\ -2 \\ -1 \end{pmatrix}, a_2 = \begin{pmatrix} 2 \\ 2 \\ -4 \\ -5 \end{pmatrix}, a_3 = \begin{pmatrix} 0 \\ 8 \\ -2 \\ -5 \end{pmatrix}$ \begin{align*} & b_1 = \frac{1}{\norm{a_1}} a_1 ,\; \norm{a_1} = (4^2 + 2^2 + 2^2 + 1^2)^{\frac 12} = \sqrt{25} = 5 \\ & = \frac 15 \begin{pmatrix} 4 \\ 2 \\ -2 \\ -1 \end{pmatrix} ,\; \inner{a_2}{b_1} = \frac 15 \begin{pmatrix} 2 \\ 2 \\ -4 \\ -5 \end{pmatrix} \cdot \begin{pmatrix} 4 \\ 2 \\ -2 \\ -1 \end{pmatrix} = \frac 15 (8 + 4 + 8 + 5)^\frac 12 = \frac{25}5 \\ & c_2 = a_2 - \underbrace{\inner{a_2}{b_1}}_5 b_1 = \begin{pmatrix} 2 \\ 2 \\ -4 \\ -5 \end{pmatrix} - \begin{pmatrix} 4 \\ 2 \\ -2 \\ -1 \end{pmatrix} = \begin{pmatrix} -2 \\ 0 \\ -2 \\ -4 \end{pmatrix} \\ & \norm c_2 = (4 + 4 + 16) = \sqrt{24} \\ & \implies b_2 = \frac{1}{\sqrt{24}} \begin{pmatrix} -2 \\ 0 \\ -2 \\ -4 \end{pmatrix} \\ & c_3 = a_3 - \inner{a_3}{b_1} b_1 - \inner{a_3}{b_2} b_2 = \dots = \begin{pmatrix} -2 \\ 6 \\ 2 \\ 0\end{pmatrix} \\ & \norm{c_3} = (4 + 36 + 4)^\frac 12 = \sqrt{44} \\ & \implies b_3 = \frac 1{\sqrt{44}} \begin{pmatrix} -2 \\ 6 \\ 2 \\ 0 \end{pmatrix} \end{align*} \endgroup \begin{satz} \label{theo:3.1.17} Sei $V$ ein euklidischer/unitärer Vektorraum mit höchstens abzählbarer Dimension. Dann kann jedes Orthonormalsystem zu einer Orthonormalbasis von $V$ ergänzt werden. \end{satz} \begin{proof} Sei $(b_1, \dots, b_k)$ ein Orthonormalsystem, $(b_1, \dots, b_k, a_{k+1}, \dots)$ eine Basis. Satz \ref{theo:3.1.16} $\implies \exists b_{k+1}, b_{k+2}, \dots$ mit $(b_1, \dots, b_k, b_{k+1}, \dots)$ Orthonormalbasis. \end{proof} \begin{defin} \begin{itemize} \item $M, N \subseteq V$ heißen \underline{orthogonal} wenn $\forall v \in M, w \in N: \underset{\inner vw = 0}{v \bot w}$ \\ Wir schreiben $M \bot N$ \\ $[M = {v} \implies v \bot N]$ \item Für $M \subseteq V$ heißt \[ M^\bot := \{ v\in V: v \bot M \} = \{ v \in V: \forall w \in M: \inner vw = 0 \} \] \underline{orthogonales Komplement} von M \end{itemize} \end{defin} \subsubsection{Bemerkung} $M^\bot$ ist immer Unterraum von $V$, selbst wenn $M$ kein Unterraum ist. \begin{satz} Sei $U$ $r$-dimensionaler Unterraum von $n$-dimensionalem euklidischen/\\ unitären Vektorraum $V$. Dann gilt: \begin{enumerate}[label=\alph*)] \item $\dim(U^\bot) = n - r$ \item $(U^\bot)^{{}^\bot} = U$ \item $V = U \oplus U^\bot$ \end{enumerate} \end{satz} \begin{proof} \begin{enumerate}[label=\alph*)] \item $(b_1, \dots, b_r)$ Orthonormalbasis von $U$.\\ $(b_1, \dots, b_r, b_{r+1}, \dots, b_n)$ Orthonormalbasis von $V$. \newline [die existiert laut Satz \ref{theo:3.1.17}] \\ Behauptung: $U^\bot = \linspan{ b_{r+1}, \dots, b_n }$ \\ Beweis: $\subseteq$: Sei $v\in U^\bot, v = \sum_{i=1}^n \lambda_i b_i$ \[ \forall i \in [r]: 0 = \inner v{b_i} = \inner{\sum_{j=1}^n \lambda_j b_j}{b_i} = \sum_{j=1}^n \lambda_j \underbrace{\inner{b_j}{b_i}}_{\delta_{ij}} = \lambda_i \] $\supseteq: v\in \linspan{ b_{r+1}, \dots, b_n } \overset{!}{\implies} v \in U^\bot$ \\ $\implies \sum_{j=r+1}^n \lambda_j b_j, u = \sum_{i=1}^r \mu_i b_i \in U$ \\ $\implies \inner vu = \sum_{j=r+1}^n \lambda_j \sum_{i=1}^r \underbrace{\inner{b_j}{b_i}}_{=0}=0$ \\ $\implies$ a) \item $U \subseteq (U^\bot)^{{}^\bot}$ \\ Sei $v \in U \implies \forall w \in U^\bot: \inner wv = 0 \implies v \in (U^\bot)^{{}^\bot}$ \\ $(U^\bot)^{{}^\bot} \subseteq U: \dim((U^\bot)^{{}^\bot}) \overset{\text{a)}}{=} n - \dim(U^\bot) \overset{\text{a)}}{=} n - (n-r) = r = \dim(U)$ \item $U\oplus U^\bot$: Sei $w\in U \cap U^\bot \implies \inner ww = 0 \implies w = 0$ \end{enumerate} \end{proof} \subsubsection{Bemerkung} $(U^\bot)^{{}^\bot}$ gilt im Allgemeinen nicht, wenn $\dim(V) = \infty$. \subsubsection{Beispiel} $V = \{ f: [0, 1] \to \R, \text{ stetig} \}, \inner fg = \int_0^1 f(t) g(t) dt$\\ $U = \{ p \in V: p \text{ ist Polynom}\}$\\ $(p_1, p_2, \dots)$ ist eine Orthonormalbasis von $U$.\\ Wir zeigen: $U^\bot = \{0\} \implies (U^\bot)^{{}^\bot} =V\neq U$ \begin{nonumbersatz}[Weierstraß] $\forall f \in V, \varepsilon > 0 \; \exists p \in U: \norm{f-p}_\infty \le \varepsilon$\\ Beweis wird hier nicht geführt. \end{nonumbersatz} \par Sei $f \in V \setminus \{0\}, a := \norm f^2 = \inner ff, b = \norm f _\infty$. Sei $p \in U: \norm{f-p}_\infty < \frac a {2b}$ \\ Behauptung: $\inner fp > 0$ \begin{align*} \inner fp = \int_0^1 f(t)p(t) dt & = \int_0^1 f(t)[f(t) - (f(t) - p(t))]dt \\ & = \int_0^1 f(t)f(t)dt - \int_0^1 f(t)(f(t) - p(t)) dt \\ & = a - \int_0^1 f(t)[f(t)-p(t)] dt \\ \int_0^1 \underbrace{f(t)}_{\substack{\le \norm b_\infty \\ \le b}} [\underbrace{f(t) - p(t)}_{\substack{\le \norm{f - p}_\infty \\\le \frac a {2b}}}] & \le \int_0^1 b \cdot \frac a {2b}dt - \int_0^1 \frac a2 dt = \frac a2 \\ \forall f \in V: \exists p \in U: \int_0^1 f(t)p(t)dt & \neq 0 \implies U^\bot = \{\} \implies U \subsetneq (U^\bot)^{{}^\bot} \end{align*} \section[Adjungierte Abbildungen und normale Endomorphismen] {Adjungierte Abbildungen und normale \\Endomorphismen} \begin{defin} Seien $V, W$ euklidische/unitäre Vektorräume, $\alpha \in \homk(V, W), \K \in \{\R, \C\}$. \\ $\alpha^* \in \homk(W, V)$ heißt \underline{zu $\alpha$ adjungiert}, falls \[ \forall v \in V: \forall w \in W: \inner{\alpha(v)}{w}_W = \inner{v}{\alpha^*(w)}_V \] $V=W$: Gilt $\genfrac{}{}{0pt}{0}{\alpha = -\alpha^*}{\alpha = \alpha^*}$, so heißt $\alpha$ $\genfrac{}{}{0pt}{0}{\text{\underline{anti-selbstadjungiert}}} {\text{\underline{selbstadjungiert}}}$. \end{defin} \subsubsection{Bemerkung} \begin{itemize} \item $\alpha^*$ muss nicht existieren! \\ Beispiel: $U, V$ wie vorher. $\alpha \in \Hom_\R(U, V), \alpha(p) = p \forall p \in U$ \\ Angenommen $\exists \alpha^* \in \Hom_\R(V, U), e(t) = e^t \implies e \in V$ \\ $\alpha^* (e) = a_1 p_1 + \dots + a_m p_m$ \\ $f := e - (a_1 p_1 + \dots + a_m p_m) = e- \alpha^*(e) \neq 0$ \\ Behauptung: $f \in U^\bot (\implies f = 0$ \Lightning) \item $i \in \{m+1, m+2, \dots \}$ %Eigentlich nicht so wirklich ein Punkt?! \begin{align*} & \inner{e}{p_i} = \inner{e}{\alpha(p_i)} = \inner{\alpha^*(e)}{p_i} = \inner{a_1 p_1 + \dots + a_m p_m}{p_i} = 0 \\ & \implies \forall i = m+1, m+2, \dots: \inner{f}{p_i} = 0 \end{align*} Außerdem: $i\in [m]: \inner{e}{p_i} = \inner{\alpha^*}{p_i}$ \newline $\implies \inner{f}{p_i} = \inner{e - \alpha^*(e)}{p_i} = \inner{\alpha^*(e) - \alpha^*(e)}{p_i} = 0$ \\ $\implies \inner{f}{p_i} \forall i = 1, 2, \dots \implies f \in U^\bot \implies f= \{0\}$\Lightning \item Wenn $\alpha^*$ existiert, dann ist $\alpha^*$ eindeutig. \end{itemize} \begin{lemma} Sei $\alpha \in \homk(V, W), \dim(V) < \infty$ Dann existiert $\alpha^*$: \\ Mit $\{e_1, \dots, e_n\}$ Orthonormalbasis von $\underbrace{V}_{ \mathclap{\text{Existenz gegeben wegen Satz \ref{theo:3.1.17}}}}$ gilt: \[ \alpha^*(w) := \sum_{i=1}^n \inner{w}{\alpha(e_i)} e_i \] \end{lemma} \begin{proof} Zu Zeigen: $\forall v \in V, w \in W: \inner{\alpha(v)}{w} = \inner{v}{\alpha^*(w)}$.\\ O.B.d.A.: $v = e_j$ für $j \in [n]$ \begin{align*} \inner{v}{\alpha^*(w)} = \inner{e_j}{\sum_{i=1}^n \inner{w}{\alpha(e_i)}e_i} & = \sum_{i=1}^n \inner{e_j}{\inner{w}{\alpha(e_i)} e_i} \\ & = \sum_{i=1}^n \overline{\inner{w}{\alpha(e_i)}} \underbrace{\inner{e_j}{e_i}}_{\delta_ij} \\ & = \overline{\inner{w}{\alpha(e_j)}} = \inner{\alpha(e_j)}{w} = \inner{\alpha(v)}{w} \end{align*} \end{proof} \begin{defin} Sei $A \in \C^{m \times n}$. \begin{align*} & \overline{A} := (\overline{a}_{ij})_{i,j} & & \text{ zu $A$ \underline{konjugiert komplexe} Matrix} \\ & A^* = (\overline{A})^T & & \text{ zu $A$ \underline{adjungierte} Matrix} \end{align*} \end{defin} \begin{satz} \label{theo:3.2.4} Sei $\alpha \in \homk(V, W), \K \in \{\R, \C\}, \dim(V), \dim(W) < \infty$ \[ \begin{rcases} E = \{e_1, \dots, e_n\} \text{ ONB von V} \\ F = \{f_1, \dots, f_n\} \text{ ONB von W} \end{rcases} \implies {}_E M(\alpha^*)_F = ({}_F M(\alpha)_E)^* \] \end{satz} \begin{proof} \begin{align*} & A = {}_F M(\alpha)_E = (a_{ij})_{\substack{i=1,\dots,m \\j=1,\dots,n}}, \, B = {}_E M(\alpha^*)_F = (b_{ij})_{\substack{i=1,\dots,n \\j=1,\dots,m}} \\ & \alpha(e_j) = \sum_{i=1}^m a_{ij} f_i \\ & F \text{ ONB} \implies a_{ij} = \inner{\alpha(e_j)}{f_i} \\ & \alpha^*(f_j) = \sum_{i=1}^n b_{ij} e_i \implies b_{ij} = \inner{\alpha^*(f_j)}{e_i} \\ & \dots = \overline{\inner{e_i}{\alpha^*(f_j)}} = \overline{\inner{\alpha(e_i)}{f_j}} = \overline{a_{ji}} \\ & \implies B = A^* \end{align*} \end{proof} \begin{lemma} \begin{enumerate}[label=\alph*)] \item $(\alpha^*)^{{}^*} = \alpha$ \item $(\alpha + \beta)^* = \alpha^* + \beta^*$ \item $(\lambda \alpha)^* = \overline{\lambda}\alpha^*$ \item $(\beta \circ \alpha)^* = \alpha^* \circ \beta^*$ \item $\alpha \in \homkv, \dim(V) < \infty \implies \det(\alpha) = \overline{\det(\alpha^*)}$ \end{enumerate} \end{lemma} \begin{proof} \begin{enumerate}[label=\alph*)] \item $\inner{\alpha(v)}{w} = \inner{v}{\alpha^*(w)} = \overline{\inner{\alpha^*(w)}{v}} =$ \\ $\overline{\inner{w}{(\alpha^*)^{{}^*}(v)}} = \inner{(\alpha^*)^{{}^*}(v)}{w} \; \forall v \in V, w \in W$ \\ $\implies \inner{\alpha(v) - (\alpha^*)^{{}^*}(v)}{w} = 0 \; \forall v \in V, w \in W, \; w:= \alpha(v) - (\alpha^*)^{{}^*}$ \\ $\implies \inner{\alpha(v) - (\alpha^*)^{{}^*}}{\alpha(v) - (\alpha^*)^{{}^*}} = 0 \iff \norm{\alpha(v) - (\alpha^*)^{{}^*}} = 0 \implies \forall v \in V: \alpha(v) = (\alpha^*)^{{}^*}$ \item \begin{align*} \inner{(\alpha + \beta)}{w} & = \inner{v}{(\alpha + \beta)^*(w)} = \inner{\alpha(v) + \beta(v)}{w} \\ & = \inner{\alpha(v)}{w} + \inner{\beta(v)}{w} = \inner{v}{\alpha^*(w)} + \inner{v}{\beta^*(w)} \\ & = \inner{v}{\alpha^*(w) + \beta^*(w)} \end{align*} \item $ \inner{(\lambda \alpha)(v)}{w} = \inner{v}{(\lambda \alpha)^*(w)} $\\ $ = \lambda \inner{\alpha(v)}{w} = \lambda \inner{v}{\alpha^*(w)} = \inner{v}{\overline{\lambda} \alpha^*(w)}$ \item $ \inner{\beta \circ \alpha(v)}{w} = \inner{\alpha(v)}{\beta^*(w)} = \inner{v}{\alpha^* \circ \beta^*(w)}$ \\ $ = \inner{v}{(\beta \circ \alpha)^*(w)} $ \item Sei $E$ Orthonormalbasis, $A = {}_E M(\alpha)_E = (a_{ij}) \in \C^{\nxn}$ \[ \overline{\det(\alpha)} = \sum_{\pi \in S_n} \sgn(\pi) \overline{a}_{1\pi(1)} \cdots \overline{a}_{n\pi(n)} = \det(\overline{A}) = \det(\overline A^T) = \det(A^*) \] $ =\det({}_E M(\alpha^*)_E) = \det(\alpha^*) $ %Eigentlich noch in dem oberen environment \end{enumerate} \end{proof} \begin{defin} $\alpha \in \homkv$ mit $V$ euklidisch/unitär heißt \underline{normal}, wenn $\alpha^*$ existiert und \[ \alpha \circ \alpha^* = \alpha^* \circ \alpha \] \end{defin} \begin{satz} \label{theo:3.2.7} \[ \alpha \text{ normal} \iff \inner{\alpha(v)}{\alpha(w)} = \inner{\alpha^*(v)}{\alpha^*(w)} \] \end{satz} \begin{proof} \begin{itemize} \item[$\implies$:]$\inner{\alpha(v)}{\alpha(w)} = \inner{v}{\alpha^*(\alpha(w))} \underbrace{=}_{\alpha \text{ normal}} \inner{v}{\alpha(\alpha^*(w))}$ \\ $ \inner{\alpha^*(v)}{\alpha^*(w)} = \inner{v}{(\alpha^*)^{{}^*} (\alpha^*(v))} = \inner{v}{\alpha(\alpha^*(w))}$ \end{itemize} \end{proof} \begin{lemma} \[ \alpha \text{ normal } \implies \ker(\alpha) = \ker(\alpha^*) \] \end{lemma} \begin{proof} \begin{align*} & \norm{\alpha(v)}^2 = \inner{\alpha(v)}{\alpha(v)} = \inner{\alpha^*(v)}{\alpha^*(v)} = \norm{\alpha^*(v)}^2 \\ & v \in \ker(\alpha) \iff \alpha(v) = 0 \iff \norm{\alpha(v)} = 0 \iff \norm{\alpha^*(v)} = 0 \\ & \iff \alpha^*(v) = 0 \iff v \in \ker(\alpha^*) \end{align*} \end{proof} \begin{satz} \label{theo:3.2.9} $\alpha$ normal: \begin{enumerate}[label=\alph*)] \item $\alpha$ und $\alpha^*$ besitzen die selben Eigenvektoren. \item $v \in \eig_\alpha(\lambda)\implies v \in \eig_{\alpha^*}(\overline \lambda)$ \end{enumerate} \end{satz} \begin{proof} $v \in \eig_\alpha(\lambda)$ \begin{align*} \norm{\alpha(v) - \lambda v}^2 & = \inner{\alpha(v) - \lambda v}{\alpha(v) - \lambda v} \\ & = \inner{\alpha(v)}{\alpha(v)} - \lambda \inner{v}{\alpha(v)} - \overline\lambda \inner{\alpha(v)}{v} + \lambda \overline \lambda \inner vv \\ & \underbrace{=}_{\alpha\text{ normal}} \inner{\alpha^*(v)}{\alpha^*(v)} - \lambda \overline{\inner{\alpha(v)}{v}} -\overline{\lambda} \inner{v}{\alpha^*(v)} + \lambda \overline \lambda \inner vv \\ & = \inner{\alpha^*(v)}{\alpha^*(v)} - \lambda \overline{\inner{v}{\alpha^*(v)}} - \overline \lambda \inner{v}{\alpha^*(v)} + \lambda \overline \lambda \inner vv \\ & =\inner{\alpha^*(v)}{\alpha^*(v)} - \lambda \inner{\alpha^*(v)}{v} - \overline \lambda \inner{v}{\alpha^*(v)} + \lambda \overline\lambda \inner vv \\ & = \inner{\alpha^*(v) - \overline \lambda v}{\alpha^*(v) - \overline{\lambda} v} = \norm{\alpha^*(v) - \overline \lambda v}^2 \end{align*} \begin{align*} & v \in \eig_\alpha(\lambda) \iff \alpha(v) - \lambda v = 0 \iff \norm{\alpha(v) - \lambda v}^2 = 0 \\ & \iff \norm{\alpha^*(v) - \overline \lambda v}^2 = 0 \iff \alpha^*(v) - \overline \lambda v = 0 \iff v \in \eig_{\alpha^*}(\overline\lambda) \end{align*} \end{proof} \begin{satz} [Spektralsatz für normale Abbildungen] \label{theo:3.2.10} $\alpha \in \Hom_\C(V, V), V$ unitär mit $\dim(V) = n < \infty$. Dann gilt: \[ \exists \text{ Orthonormalbasis aus Eigenvektoren von } \alpha \iff \alpha \text{ normal} \] \end{satz} \begin{proof} \begin{itemize} \item[$\impliedby$:] $\alpha$ normal \begin{itemize} \item[$n=1$:] $\exists$ Eigenvektor $e_1 \in V \setminus\{0\}$ mit $\alpha(e_1) = \lambda e_1$.\\ o.B.d.A.: $\norm{e_1} = 1 \implies v$ ist Orthonormalbasis aus Eigenvektoren \item[$n-1 \to n$:] $\exists$ Eigenvektor $e_1 \in V \setminus\{0\}$ mit $\alpha(e_1) = \lambda e_1$.\\ o.B.d.A.: $\norm{e_1} = 1 \; U= \linspan{ e_1 } ^\bot$ \begin{itemize} \item $V = \linspan{ e_1 } \oplus U, \alpha(U) \overset{\text{!}}{\subseteq} U, \alpha(\linspan{e_1}) \overset{\checkmark}{\subseteq} \linspan{e_1 }$ \end{itemize} $\implies \alpha = \alpha|_{\linspan{ e_1 }} \oplus \alpha|_U$ \\ Sei $v \in U\implies 0 = \inner{v}{e_1} \;\;\; [e_1 \in \eig_\alpha(\lambda) \iff e_1 \in \eig_{\alpha^*}(\overline\lambda)]$ \begin{align*} \inner{\alpha(v)}{e_1} & = \inner{v}{\alpha^*(e_1)} = \inner{v}{\overline \lambda e_1} \\ & = \lambda \inner{v}{e_1} = 0 \\ \implies \alpha(v) \in U & \implies \alpha(U) \subseteq U \checkmark \\ & \implies \alpha|_U \in \Hom(U, U), \dim(U) = n-1 \\ \end{align*} \begin{align*} & \overset{\mathclap{\substack{\text{Induktionsvorraussetzung} \\|}}} {\implies} \exists \text{ ONB } (e_2, \dots, e_n) \text{ von $U$ aus Eigenvektoren von } \alpha \\ & \implies (e_1, \dots, e_n) \text{ ist ONB von $V$ aus Eigenvektoren von } \alpha \end{align*} \end{itemize} \item[$\implies$:] Sei $(e_1, \dots, e_n)$ Orthonormalbasis aus Eigenvektoren von $\alpha$. Seien \\ weiters $\lambda_1, \dots, \lambda_n \in \C$ die zugehörigen Eigenwerte. \[ \alpha:\begin{cases} V & \to V \\ v = \sum_{i=1}^n \mu_i e_i & \mapsto \sum_{i=1}^n \lambda_i \mu_i e_i\end{cases} \] Definiere \[ \beta:\begin{cases} V & \to V \\ v = \sum_{i=1}^n \mu_i e_i &\mapsto \sum_{i=1}^n \overline \lambda_i \mu_i e_i\end{cases} \implies \beta = \alpha^* \] \[ \begin{aligned} \alpha^*(\alpha(v)) & = \alpha^*( \sum_{i=1}^n \lambda_i \mu_i e_i ) = \sum_{i=1}^n \overline\lambda_i \lambda_i \mu_i e_i = \sum_{i=1}^n \abs{ \lambda_i }^2 \mu_i e_i \\ \alpha(\alpha^*(v)) & = \alpha( \sum_{i=1}^n \overline\lambda_i \mu_i e_i ) = \sum_{i=1}^n \lambda_i \overline\lambda_i \mu_i e_i = \sum_{i=1}^n \abs{ \lambda_i }^2 \mu_i e_i \end{aligned} \implies \alpha \text{ normal} \] \end{itemize} \end{proof} \subsubsection{Bemerkung} Im Reellen/Euklidischen Fall gilt dieser Satz genau dann, wenn $\alpha$ diagonalisierbar ist. \begin{satz} \label{theo:3.2.11} Sei $V$ ein unitärer Vektorraum mit $\dim(V) = n < \infty$ und $\alpha \in \Hom_\C(V, V)$ selbstadjungiert. (Das heißt $\alpha = \alpha^*$) \\ Dann gilt: \begin{enumerate}[label=\alph*)] \item Alle Eigenwerte von $\alpha$ sind reell. \item $V$ besitzt eine Orthonormalbasis aus Eigenvektoren von $\alpha$. \item Eigenvektoren zu unterschiedlichen Eigenwerten sind orthogonal. \end{enumerate} \end{satz} \begin{proof} $\alpha$ ist normal: $\alpha \circ \alpha^* = \alpha \circ \alpha = \alpha^* \circ \alpha$ \begin{enumerate}[label=\alph*)] \item Sei $\lambda$ Eigenwert von $\alpha$ mit Eigenvektor $v\in V\setminus\{0\}$\\ $\overset{\text{\ref{theo:3.2.7}}}{\implies} v$ ist Eigenvektor von $\alpha^*$ mit Eigenwert $\overline{\lambda}$ \\ $\implies \lambda v = \alpha(v) = \alpha^*(v) = \overline{\lambda} v \implies (\lambda - \overline{\lambda}) v = 0 \overset{v\neq0}{\implies} \lambda = \overline{\lambda}$ \\ $\implies \lambda \in \R$ \item Folgt direkt aus Satz \ref{theo:3.2.10} \& $\alpha$ normal. \item Sei $\alpha(v_1) = \lambda_1 v_1, \alpha(v_2)=\lambda_2 v_2, \lambda_1 \neq \lambda_2$ \begin{align*} & \lambda_1 \inner{v_1}{v_2} = \inner{\lambda_1 v_1}{v_2} = \inner{\alpha(v_1)}{v_2} = \inner{v_1}{\alpha^*(v_2)} = \inner{v_1}{\alpha(v_2)} \\ & = \inner{v_1}{\lambda_2 v_2} = \overline{\lambda_2} \inner{v_1}{v_2} = \lambda_2 \inner{v_1}{v_2} \\ & \implies \underbrace{(\lambda_1 - \lambda_2)}_{\neq 0} \inner{v_1}{v_2} = 0 \implies \inner{v_1}{v_2}=0 \end{align*} \end{enumerate} \end{proof} \begin{lemma} Sei $V$ ein euklidischer Vektorraum und $\alpha \in \Hom_\R(V, V)$ normal. Dann ist $\alpha_\C \in \Hom_\C (V_\C, V_\C)$ auch normal. \end{lemma} \begin{proof} Sei $\alpha \in \Hom(V, V), \alpha_\C$ die komplexe Erweiterung von $\alpha$. Seien weiters $v, v' \in V$ mit $v = u+iw, v' = u' +iw', u, w, u', w' \in \R$. \begin{equation} \label{eq:3.2.12.1} \begin{split} \inner{\alpha_\C(v)}{\alpha_\C(v')} & = \inner{\alpha(u) + i\alpha(w)}{\alpha(u') + i\alpha(w')} \\ & \begin{multlined}=\inner{\alpha(u)}{\alpha(u')} + i \inner{\alpha(w)}{\alpha(u')} - i \inner{\alpha(u)}{\alpha(w')} \\ + (-i)(-i)\inner{\alpha(w)}{\alpha(w')}\end{multlined} \\ & \begin{multlined}= \inner{\alpha^*(u)}{\alpha^*(u')} + i \inner{\alpha^*(w)}{\alpha^*(u')} - i \inner{\alpha^*(u)}{\alpha^*(w')} \\ + (-i)(-i)\inner{\alpha^*(w)}{\alpha^*(w')} \end{multlined} \\ & = \inner{\alpha^*(u)+i\alpha^*(w)}{\alpha^*(u')} + \inner{\alpha^*(u) + i\alpha^*(w)}{i\alpha^*(w')} \\ & = \inner{\alpha^*(u) + i\alpha^*(w)}{\alpha^*(u') + i\alpha^*(w')} \\ & = \inner{(\alpha^*)_\C(v)}{(\alpha^*)_\C(v')} \end{split} \end{equation} Bleibt zu Zeigen, dass $(\alpha^*)_\C = (\alpha_\C)^*$: \begin{align*} \inner{\alpha_\C(v)}{v'} & = \inner{\alpha(u) + i \alpha(w)}{u' + i w'} \\ & = \inner{\alpha(u)}{u'} + i \inner{\alpha(w)}{u'} -i \inner{\alpha(u)}{w'} + i (-i) \inner{\alpha(w)}{w'} \\ & = \inner{u}{\alpha^*(u')} + i \inner{w}{\alpha^*(u')} -i \inner{u}{\alpha^*(w')} +i (-i) \inner{w}{\alpha^*(w')} \\ & = \inner{u+iw}{\alpha^*(u')} + \inner{u+iw}{i\alpha^*(w')} \\ & = \inner{u + iw}{\alpha^*(u') + i \alpha^*(w')} = \inner{v}{(\alpha^*)_\C(v')} \end{align*} Das heißt $(\alpha^*)_\C$ ist tatsächlich die adjungierte Abbildung von $\alpha_\C$ und mit \ref{eq:3.2.12.1} folgt nach Satz \ref{theo:3.2.7}, dass $\alpha_\C$ normal ist. \end{proof} \begin{lemma} \label{theo:3.2.13} Sei $V$ ein euklidischer Vektorraum und $\alpha \in \Hom_\R(V, V)$ normal. Sei $v_\C \in V_\C$ $\underbrace{\text{normierter}}_{\text{d.h. }\norm{v_\C}_{V_\C}=1}$ Eigenvektor von $\alpha_\C$ zum Eigenwert $\lambda \in \C\setminus \R$. \\ Dann ist $\overline{v_\C}$ normierter Eigenvektor von $\alpha_\C$ zu Eigenwert $\overline{\lambda}$. Insbesondere sind $v_\C, \overline{v_\C}$ orthogonal. \end{lemma} \begin{proof} Sei $u + iw \in V_\C$ mit $u, w\in V$. $\alpha_\C(u + iw) = \alpha(u) + i\alpha(w)$ \\ $v_\C = u + iw$ ist normiert \begin{align*} \implies 1 = \inner{u+iw}{u+iw} & = \inner uu_V + i \inner wu_V - i \inner uw_V + \inner ww_V \\ & = \inner uu_V + i \inner wu_V - i \inner wu_V + \inner ww_V \\ & = \inner uu_V + \inner ww_V \\ \implies \inner{u - iw}{u - iw} & = \inner uu_V + \inner{-w}{-w}_V = \inner uu_V \inner ww_V = 1 \\ \implies \norm{\overline{v_\C}} = 1 \end{align*} $\lambda = \gamma + i \delta$ \[ \alpha_\C(v_\C) = \lambda v_\C \implies \alpha(u) + i \alpha(w) = (\gamma + i \delta) (u + iw) = (\gamma u - \delta w) + i(\delta u + \gamma w) \] \[ \begin{aligned} \alpha(u) = \gamma u + \delta w \\ \alpha(w) = \delta u + \gamma w \end{aligned} \implies \begin{aligned} \alpha_\C (\overline{v_\C}) = \alpha(u) + i \alpha(-w) & = (\gamma u + \delta w) + i (\delta u - \gamma w) \\ & = (\gamma - i \delta)(u - iw) = \overline{\lambda}\overline{v_\C} \end{aligned} \] $\implies \overline{v_\C}$ ist Eigenvektor von $\alpha_\C$ zum Eigenvektor $\overline\lambda$ $\overset{\text{\ref{theo:3.2.9}}}{\implies} \overline{v_\C}$ ist Eigenvektor von $\alpha^*_\C$ zum Eigenwert $\lambda$. \begin{align*} & \lambda \inner{v_\C}{\overline{v_\C}} = \inner{\alpha(v_\C)}{\overline{v_\C}} = \inner{v_\C}{\alpha^*_\C(\overline{v_\C})} = \inner {v_\C}{\lambda \overline{v_\C}} = \overline{\lambda} \inner{v_\C}{\overline{v_\C}} \\ & \implies (\underbrace{\lambda - \overline\lambda} _{\mathrlap{\neq 0\text{, weil } \lambda \in \C \setminus \R}}) \inner{v_\C}{\overline{v_\C}}= 0 \implies \inner{v_\C}{\overline{v_\C}} = 0 \end{align*} \end{proof} \begin{satz} \label{theo:3.2.14} Sei $V$ ein euklidischer Vektorraum mit $\dim(V) = n < \infty$ \[ \alpha \in \Hom_\R(V, V) \text{ normal} \iff \exists \text{ ONB } B = (e_1, \dots, e_n) \text{ von } V \text{ mit } \] {\setcounter{MaxMatrixCols}{20} \[ {}_B M(\alpha)_B = \begin{pmatrix} \lambda_1 \\ & \lambda_2 \\ & & \ddots \\ & & & \lambda_k \\ & & & & \tl \gamma_1 & \mathllap{-}\delta_1 \\ & & & & \delta_1 & \gamma_1 \br \\ & & & & & & \tl \gamma_2 & \mathllap{-}\delta_2 \\ & & & & & & \delta_2 & \gamma_2 \br \\ & & & & & & & & \ddots \\ & & & & & & & & & \tl \gamma_r & \mathllap{-}\delta_r \\ & & & & & & & & & \delta_r & \gamma_r \br \end{pmatrix} \] } wobei $\spec(\alpha_\C) = \{\underbrace{\lambda_1, \dots, \lambda_k}_{\in \R}, \underbrace{\lambda_{k+1}, \dots, \lambda_{k+r}}_{\in\C\setminus\R} \}$ und $\lambda_{k+j} = \gamma_j + i \delta_j$ \subsubsection{Bemerkung} Jedem Kästchen $\eta(\gamma, \delta)$ entspricht ein Paar $\lambda, \overline\lambda$ konjugiert komplexer Eigenwerte von $\alpha_\C$. $\gamma = \Re(\lambda), \delta = \Im(\lambda)$ \end{satz} \begin{proof} \leavevmode \begin{itemize} \item[$n=1$:] \checkmark \item[$n-1 \to n$:] Wenn $\alpha$ reellen Eigenwert besitzt, kann man wie im Beweis von Satz \ref{theo:3.2.10} vorgehen. Wenn nicht: Sei $v_\C \in V_\C$ ein Eigenvektor von $\alpha_\C$ zum Eigenwert $\lambda = \delta + i\gamma \in \C \setminus \R$.\\ Lemma \ref{theo:3.2.13}: $\overline{v_\C}$ ist Eigenvektor von $\alpha_\C$ zum Eigenwert $\overline\lambda$ und \\ $\inner{v_\C}{\overline{v_\C}} = 0$ \\ Setze \[ \begin{rcases} a = \frac{1}{\sqrt2}(v_\C + \overline{v_\C}) \in V & v_\C = u + iw \\ b = \frac{1}{i\sqrt2}(v_\C - \overline{v_\C}) \in V & \overline{v_\C} = u - iw \end{rcases} \implies \begin{aligned} v_\C + \overline{v_\C} = 2u \\ \frac 1i(v_\C - \overline{v_\C}) = 2w \end{aligned} \] \tl UE\br $\implies \norm a = \norm b = 1$ und $\inner ab = 0$ \\ Weiters \[ \begin{aligned} \alpha(a) & = \frac{1}{\sqrt{2}}(\alpha_\C(v_\C) + \alpha_\C(\overline {v_\C})) = \frac{1}{\sqrt{2}}(\lambda v_\C + \overline \lambda \overline {v_\C}) \\ & = \frac{1}{\sqrt{2}}((\delta + i \gamma)(u + iw) + (\delta - i \gamma)(u - iw)) \\ & = \frac{1}{\sqrt{2}}((\delta u - \gamma w) + i(\delta w + \gamma u) + (\delta u - \gamma w) - i(\delta w + \gamma u)) \\ & = \frac{1}{\sqrt{2}}(\delta 2 u - \gamma 2 w) = \delta\frac{2u}{\sqrt{2}} - \gamma \frac{2w}{\sqrt{2}} \\ & = \delta a - \gamma b \\ \alpha(b) & = \frac{1}{i\sqrt{2}}(\alpha_\C(v_\C) - \alpha_\C(\overline{v_\C})) = \frac{1}{i\sqrt{2}}(\lambda v_\C - \overline \lambda \overline{v_\C}) \\ & = \frac{1}{i\sqrt{2}}((\delta + i \gamma)(u + iw) - (\delta - i \gamma)(u - iw)) \\ & = \frac{1}{i\sqrt{2}}((\delta u - \gamma w) + i(\delta w + \gamma u) - ((\delta u - \gamma w) - i(\delta w + \gamma u))) \\ & = \frac{1}{i\sqrt{2}}(2i \delta w + 2i \gamma u) = \delta \frac{2iw}{i\sqrt{2}} + \gamma \frac{2iu}{i\sqrt{2}} = \delta \frac{2w}{\sqrt{2}} + \gamma \frac{2u}{\sqrt{2}} \\ & = \delta b + \gamma a \end{aligned} \] \item[$\impliedby$:] Da $B$ Orthonormalbasis ist, folgt aus Satz \ref{theo:3.2.4} und $\lambda_i, \gamma_i, \delta_i \in \R$, dass ${}_B M(\alpha^*)_B = {}_B M(\alpha)_B^* = \overline{{}_B M(\alpha)_B}^T = {}_B {M(\alpha)_B}^T$. Da ${}_B M(\alpha^*)_B$ eine Blockdiagonalmatrix ist, reicht es aus die multiplikative Kommutativität für die einzelnen Blöcke zu zeigen: \begin{align*} & \lambda_i \lambda_i = \lambda_i \lambda_i \checkmark \\ & \begin{pmatrix} \gamma & -\delta \\ \delta & \gamma \end{pmatrix} \begin{pmatrix} \gamma & \delta \\ -\delta & \gamma \end{pmatrix} = \begin{pmatrix} \gamma^2 + \delta^2 & 0 \\ 0 & \delta^2 +\gamma^2 \end{pmatrix} = \begin{pmatrix} \gamma & \delta \\ -\delta & \gamma \end{pmatrix} \begin{pmatrix} \gamma & -\delta \\ \delta & \gamma \end{pmatrix} \end{align*} \end{itemize} \end{proof} \begin{satz} Sei $V$ ein euklidischer/unitärer Vektorraum, $\alpha \in \homkv$ \\ anti-selbstadjungiert. Dann gilt \begin{enumerate}[label=\alph*)] \item $\lambda \in \spec(\alpha) \implies \Re(\lambda) = 0$ \item $\alpha_\C$ besitzt eine Orthonormalbasis aus Eigenvektoren. \item Ist $V$ euklidisch, so sind die Diagonalelemente der Matrix ${}_B M(\alpha)_B$ gleich $0$, wobei $B$ die Basis aus Satz \ref{theo:3.2.14} ist. \end{enumerate} \end{satz} \begin{proof} \begin{enumerate}[label=\alph*)] \item $\alpha$ ist normal $\implies$ wegen Satz \ref{theo:3.2.9} $v \in \eig_\alpha(\lambda)\implies v \in \eig_{\alpha^*}(\overline \lambda)$ Mit $0 \neq v \in \eig_{\alpha}(\lambda)$: \[ \alpha(v) = \lambda v = -\alpha^*(v) = -\overline \lambda v \implies \lambda = -\overline \lambda \implies \Re (\lambda) = 0 \] \item $\alpha$ ist normal, $\alpha^*= -\alpha$ \item Folgt aus dem Satz~\ref{theo:3.2.14}, sowie a). \end{enumerate} \end{proof} \section{Orthogonale und unitäre Abbildungen} \begin{defin} Seien $V, W$ beide euklidische/unitäre Vektorräume, $\alpha \in \Hom(V, W)$ heißt\\ \underline{orthogonal}/% \underline{unitär} wenn \[ \forall v, w \in V: \inner{\alpha(v)}{\alpha(w)}_W = \inner vw_V \] \end{defin} \subsubsection{Bemerkung} Das sind genau die Längen- und Winkelerhaltenden Abbildungen. \begin{satz} \label{theo:3.3.2} Seien $V, W$ euklidische/unitäre Vektorräume und $\alpha \in \Hom(V, W)$. Dann sind äquivalent: \begin{enumerate}[label=\alph*)] \item $\alpha$ ist orthogonal/unitär \item $\forall v \in V: \norm v_V = 1 \implies \norm{\alpha(v)}_W = 1$ \item $\forall v \in V: \norm v_V = \norm{\alpha(v)}_W$ \item $( e_1, \dots, e_n ) \subseteq V \text{ ONS } \implies ( \alpha(e_1), \dots, \alpha(e_n) ) \subseteq W \text{ ONS.}$ \end{enumerate} \end{satz} \begin{proof} \leavevmode \begin{itemize} \item[a) $\implies$ b):] \checkmark \item[b) $\implies$ c):] Es gilt für $v \in V\setminus \{0\}:$ $\norm{\frac{v}{\norm v}} = 1 \implies \norm{\frac{\alpha(v)}{\norm v}} = 1$\\ $\implies \norm{\alpha(v)} = \norm v$ \item[c) $\implies$ d):] $\inner vw = \frac 14(\norm{v+w}^2 - \norm{v-w}^2 +i\norm{v+iw}^2 - i\norm{v-iw}^2)$ \item[d) $\implies$ a):] Sei $v, w \in V$. \begin{enumerate}[label=\arabic*. Fall:] \item $v = 0 \implies \inner{\alpha(v)}{\alpha(w)} = \inner{0}{\alpha(w)} = 0 \checkmark$ \item $w = \lambda v \implies \inner{\alpha(w)}{\alpha(v)} = \lambda \inner{\alpha(v)}{\alpha(v)} = \lambda \norm{\alpha(v)}^2$. \\ Sei $l := \frac{v}{\norm v} \overset{\text{d)}}{\implies} \alpha(l)$ ist ONS $\implies \norm{\alpha(l)} = 1 \implies \norm{\alpha(v)} = \norm v$. \\ Es folgt $\inner{\alpha(v)}{\alpha(w)} = \inner vw \checkmark$. \item $v, w$ linear unabhängig. Sei $(e_1, e_2)$ ONS mit $\linspan{\{e_1, e_2\}} = \linspan{\{ v, w \}}$. (Gram-Schmidt liefert Existenz) \begin{align} \implies & (\alpha(e_1), \alpha(e_2)) \text{ ist ONS} \nonumber \\ \nonumber & v = \mu_1 e_1 + \mu_2 e_2 \\ \nonumber & w = \tau_1 e_1 + \tau_2 e_2 \\ \nonumber \implies & \alpha(v) = \mu_1 \alpha(e_1) + \mu_2 \alpha(e_2) \\ & \alpha(w) = \tau_1 \alpha(e_1) + \tau_2 \alpha(e_2) \label{eq:3.3.2.1} \end{align} \[ \underset{\text{\ref{theo:3.1.15}}}{\implies} \inner vw = \mu_1 \overline{\tau_1} + \mu_2 \overline{\tau_2} \underset{\mathclap{\substack{| \\ (\alpha(e_1), \alpha(e_2)) \text{ ONS \& \ref{eq:3.3.2.1}}}}}{=} \inner{\alpha(v)}{\alpha(w)} \] \end{enumerate} \end{itemize} Beweis für $\inner vw = \frac 14(\norm{v+w}^2 - \norm{v-w}^2 +i\norm{v+iw}^2 - i\norm{v-iw}^2)$: \begin{align*} & \frac 14(\norm{v+w}^2 - \norm{v-w}^2 +i\norm{v+iw}^2 - i\norm{v-iw}^2) \\ & \begin{multlined}= \frac 14 (\inner{v+w}{v+w} - \inner{v-w}{v-w}) + \frac i4 (\inner{v+iw}{v+iw} \\- \inner{v-iw}{v-iw})\end{multlined} \\ & \begin{multlined}= \frac 14 (\cancel{\inner vv} + \inner vw + \inner wv + \cancel{\inner ww} - \cancel{\inner vv} + \inner vw + \inner wv - \cancel{\inner ww}) \\ + \frac i4 (\cancel{\inner vv} + \inner v{iw} + \inner {iw}v + \cancel{\inner {iw}{iw}}\\ - \cancel{\inner vv} - \inner v{-iw} - \inner {-iw}v - \cancel{\inner {-iw}{-iw}}) \end{multlined} \\ & = \frac 14 (2 \inner vw + 2 \inner wv) + \frac i4 (-i \inner vw + i \inner wv -i \inner vw + i \inner wv) \\ & = \frac 14 (2 \inner vw + 2 \inner wv) + \frac 14 (\inner vw - \inner wv + \inner vw - \inner wv) \\ & = \frac 14 (2 \inner vw + 2 \inner wv + 2 \inner vw - 2 \inner wv) \\ & = \frac 14 4 \inner vw = \inner vw \end{align*} \end{proof} \begin{korollar} \label{theo:3.3.3} \begin{enumerate}[label=\alph*)] \item $\alpha$ orthogonal $\implies \alpha_\C$ unitär \item $\alpha$ orthogonal/unitär $\implies \alpha$ injektiv. \end{enumerate} \end{korollar} \begin{proof} \begin{enumerate}[label=\alph*)] \item Folgt direkt aus Satz~\ref{theo:3.3.2}: \begin{align*} \underset{\substack{\rotatebox{90}{=} \\u+iv}}{\norm{v_\C}} = 1 & \iff \norm u^2 + \norm v^2 = 1 \\ & \implies \norm{\alpha_\C(v_\C)} = \norm{\alpha(u)}^2 + \norm{\alpha(v)}^2 = 1 \end{align*} \item $\alpha(v) = 0 \implies \norm{\alpha(v)} = 0 \implies \norm v = 0 \implies v = 0$ \end{enumerate} \end{proof} \begin{defin} \begin{itemize} \item $A \in \R^{\nxn}$ heißt \underline{orthogonal} wenn $A^{-1} = A^T$. \item $A \in \C^{\nxn}$ heißt \underline{unitär} wenn $A^{-1} = A^* = \overline{A}^T$. \item $O(n, \R) := \{ A \in \R^{\nxn}: \det(A)\neq 0 \land A^{-1} = A^T \}$ \item $U(n, \C) := \{ A \in \C^{\nxn}: \det(A)\neq 0 \land A^{-1} = A^* \}$ \end{itemize} \end{defin} \subsubsection{Beispiele} \[ \begin{pmatrix} 0 & 1 & 0 \\ 1 & 0 & 0 \\ 0 & 0 & 1\end{pmatrix}, \; \begin{pmatrix} \sin\varphi & \cos \varphi \\ -\cos\varphi & \sin \varphi \end{pmatrix}, \; \frac 13 \begin{pmatrix} 2 & 1 & 2 \\ -2 & 2 & 1 \\ 1 & 2 & -2\end{pmatrix}\text{orthogonal} \] \[ \frac{1}{\sqrt{2}}\begin{pmatrix} 1 & i \\ -i & -1 \end{pmatrix} \text{unitär} \] \begin{satz} Es sind äquivalent für $A \in \K^{\nxn}, \K \in \{\R, \C\}$: \begin{enumerate}[label=\alph*)] \item $A$ ist orthogonal/unitär. \item $(a_{1\_}, a_{2\_}, \dots, a_{n\_})$ bilden ONS in $\K^n$. \item $(a_{\_1}, a_{\_2}, \dots, a_{\_n})$ bilden ONS in $\K^n$. \end{enumerate} \end{satz} \begin{proof} \leavevmode \begin{itemize} \item[a) $\iff$ b):] heißt, dass $\inner{a_{i\_}}{a_{j\_}}_{\K^n} = \delta_{ij}$.\\ Gleichzeitig gilt $(\inner{a_{i\_}}{a_{j\_}}_{\K^n})_{i,j=1,\dots,n} = A A^*$ \\ Also ist b) gleichbedeutend mit $A A^* = I$, also $A^{-1} = A^*$. \item[a) $\iff$ c):] genauso, nur mit $A^* A$ \end{itemize} \end{proof} \begin{satz} Sei $V$ ein euklidischer/unitärer Vektorraum mit $\dim(V)<\infty$ und $\alpha\in\Hom(V, V)$. Dann gilt \[ \alpha\text{ ist orthogonal/unitär} \iff \alpha^{-1}=\alpha^* \] \end{satz} \begin{proof} \begin{itemize} \item[$\implies$:] Seien $v,w \in V$. $\alpha^{-1}$ existiert wegen Korollar~\ref{theo:3.3.3}b) \begin{align*} \inner{v}{\alpha^*(w) - \alpha^{-1}} & = \inner{v}{\alpha^*(w)} - \inner{v}{\alpha^{-1}(w)} \\ & = \inner{\alpha(v)}{w} - \inner{v}{\alpha^{-1}(w)} \\ & = \inner{\alpha(v)}{w} - \inner{\alpha(v)}{\alpha(\alpha^{-1}(w))} \\ & = \inner{\alpha(v)}{w} - \inner{\alpha(v)}{w} = 0 \end{align*} \item[$\impliedby$:] Sei $\alpha^* = \alpha^{-1}, u,v,w\in V, v = \alpha(w)$ \[ \implies \inner uw = \inner{u}{\alpha^{-1}(v)} = \inner{u}{\alpha^*(v)} = \inner{\alpha(u)}{v} =\inner{\alpha(u)}{\alpha(w)} \checkmark \] \end{itemize} \end{proof} \begin{satz} Sei $B$ Orthonormalbasis, $\alpha \in \Hom(V, V), A= {}_B M(\alpha)_B$. Dann gilt: \begin{enumerate}[label=\alph*)] \item $\alpha$ orthogonal $\iff A$ orthogonal. \item $\alpha$ unitär $\iff A$ unitär. \end{enumerate} \end{satz} \begin{proof} Satz \ref{theo:3.2.4}: ${}_B M(\alpha^*)_B = A^*$ \\ ${}_B M(\alpha^{-1})_B = A^{-1}$ \end{proof} \begin{satz} \begin{enumerate}[label=\alph*)] \item $O(n, \R)$ ist Untergruppe von $\GL(n, \R)$. \item $U(n, \C)$ ist Untergruppe von $\GL(n, \C)$. \item $A \in O(n, \R) \implies \det(A) \in \{1, -1\}$. \item $A \in U(n, \C) \implies \abs{ \det(A) } = 1 \implies \det(A) = e^{i\alpha}, \alpha \in [ 0, 2\pi ]$. \end{enumerate} \end{satz} \begin{proof} \begin{enumerate}[label=\alph*)] \item $I \in O(n, \R), A,B \in O(n, \R)$ \\ $(AB)^* = B^*A^* = B^{-1}A^{-1} = (AB)^{-1} \implies AB \in =(n, \R)$ \\ $(A^{-1})^{{}^*} = (A^*)^{{}^*} = A \implies A^{-1} \in O(n, \R)$ \item Genauso \item $A^{-1} = A^T$. \begin{align*} 1 & = \det(A A^{-1}) = \det(A)\det(A^{-1}) = \det(A) \det(A^T) = \det(A) \det(A) \\ & = \det(A)^2 \implies \det(A) \in \{-1, 1\} \end{align*} \item $\det(\overline A) = \overline{\det(A)} \implies \det(A^*) = \overline{\det(A)}$. \begin{align*} & 1 = \det(A) \det(A^{-1})=\det(A) \det(A^*) = \det(A) \overline{\det(A)} \\ & \implies \abs{ \det(A) }=1 \end{align*} \end{enumerate} \end{proof} \subsubsection{Polarzerlegung} $z = \overbrace{e^{i\varphi}}^{\text{Betrag 1}} \underbrace{\abs{ z }}_{\mathclap{\text{positiv reell}}}$\\ Betrag 1 $\cong$ unitär, positiv $\cong$ selbstadjungiert mit positiven Eigenwerten. \begin{satz} [Polarzerlegung] \label{theo:3.3.9} Sei $V$ ein euklidischer/unitärer Vektorraum, $\dim(V)<\infty$ und sei $\alpha \in \Hom(V,V)$. Dann existiert eine orthogonale/unitäre Abbildung $\beta$ und eine selbstadjungierte Abbildung $\gamma$ mit lauter nichtnegativen reellen Eigenwerten, sodass $\alpha = \beta \circ \gamma$. Falls $\alpha$ Automorphismus ist, so sind alle Eigenwerte von $\gamma$ positiv, und $\gamma, \beta$ eindeutig bestimmt. \end{satz} \begin{proof} Zunächst: $\alpha$ Automorphismus. \begin{itemize} \item $\alpha^*$ ist auch Automorphismus. \begin{align*} \text{Sei }\alpha^*(v) = 0 & \implies \forall w \in V: \inner{w}{\alpha^*(v)} = 0 \\ & \implies \forall w \in V: \inner{\alpha(V)}{v} = 0 \\ & \implies v \in \underbrace{\im(\alpha)}_v{}^\bot = \{0\} \\ & \implies v = 0 \end{align*} \item $\alpha^*\circ \alpha$ ist Automorphismus und selbstadjungiert. \[ (\alpha^*\alpha)^{{}^*} = \alpha^*(\alpha^*)^{{}^*} = \alpha^* \alpha \] \item Satz \ref{theo:3.2.11} ist $\spec(\alpha^*\alpha) \subseteq \R\setminus\{0\}$. $\exists$ ONB $(e_1, \dots, e_n)$ von V aus Eigenvektoren, $(\lambda_1, \dots, \lambda_n) \subseteq \R\setminus\{0\}$ Eigenwerte. Behauptung: $\lambda_i > 0 \forall i \in [n]$ \[ \lambda_i = \lambda_i \inner{e_i}{e_i} = \inner{\lambda_ie_i}{e_i} = \inner{\alpha^*\alpha(e_i)}{e_i} = \inner{\alpha(e_i)}{\alpha(e_i)} > 0 \] \item $\gamma: \begin{cases} v & \to V \\ v = \sum_{i=1}^n \mu_i e_i & \mapsto \sum_{i=1}^n \mu_i \sqrt{\lambda_i} e_i\end{cases}$ ${}_B M(\alpha^* \alpha)_B = \begin{pmatrix} \lambda_1 \\ & \ddots \\ & & \lambda_n\end{pmatrix}$ ${}_B M(\gamma)_B = \begin{pmatrix} \sqrt{\lambda_1} \\ & \ddots \\ & & \sqrt{\lambda_n}\end{pmatrix}$ $\implies \gamma$ selbstadjungiert \\ $\implies \gamma^2 = \alpha^* \alpha$ \item $\alpha = \beta \circ \gamma \implies \beta = \alpha \circ \gamma^{-1}$ Behauptung: $\beta$ unitär, das heißt $\beta^{-1} = \beta^*$ \begin{align*} \beta^{-1} & = (\alpha \gamma^{-1})^{{}^{-1}} = \gamma \alpha^{-1} = \gamma^{-1} \gamma^2 \alpha^{-1} = \gamma^{-1} \alpha^* \alpha \alpha^{-1} = \gamma^{-1} \alpha^* \\ & = (\alpha(\gamma^{-1})^*)^{{}^*} = (\alpha \gamma^{-1})^{{}^*} = \beta^* \end{align*} $\implies \beta$ unitär. \end{itemize} \underline{Eindeutigkeit:} $\alpha = \beta' \circ \gamma'$ \[ \gamma^2 = \alpha^* \alpha = (\gamma')^* \underbrace{\beta'^* \beta'}_{\id}\gamma' = (\gamma')^* \gamma' = (\gamma')^2 \] $\implies \gamma, \gamma'$ haben dieselben Eigenwerte und Eigenvektoren. \\ $\implies \gamma = \gamma' \implies \beta = \beta'$ \\ \underline{$\alpha$ nicht injektiv}: \begin{itemize} \item $W := \ker(\alpha)^\bot \implies \alpha|_W$ ist injektiv. \\ Sei $v, w \in W, \alpha(v) = \alpha(w) \implies \alpha(v - w) = 0 \implies v - w \in \ker(\alpha) = W^\bot \cap W = \{0\} \implies v = w$ $\implies \alpha|_W = \beta_W \circ \gamma_W$ mit $\beta, \gamma \in \Hom(W, W); \beta_W$ unitär, $\gamma_W$ selbstadjungiert mit positiven Eigenwerten. \item Sei $( e_1, \dots, e_k )$ ONB von $W$, $(e_1, \dots, e_k, \dots, e_n)$ ONB von $V$. \item $\pi:\begin{cases} V & \to W \\ v = \sum_{i=1}^n \lambda_i e_i & \to \sum_{i=1}^k \lambda_i e_i\end{cases}$ orthogonale Projektion auf $W$. \\ $\pi$ ist selbstadjungiert: \begin{align*} & \inner{\pi(v)}w = \inner{\sum_{i=1}^k \lambda_i e_i}{\sum_{j=1}^n \mu_j e_j} = \sum_{i=1}^k \lambda_i \overline{\mu_i} \\ & \inner{v}{\pi(w)} = \inner{\sum_{i=1}^n \lambda_i e_i}{\sum_{j=1}^k \mu_j e_j} = \sum_{i=1}^k \lambda_i \overline{\mu_i} \end{align*} $\gamma := \pi^* \circ \gamma_W \circ \pi = \pi \circ \gamma_W \circ \pi$ \begin{align*} & v\in W & & \implies \gamma(v) = \gamma_W(v) \\ % Hier fehlt noch was aus der VO, nachschauen & v \in W^\bot = \ker(\alpha) & & \implies \gamma(v) = 0 \end{align*} $\beta := \underset{W}{\beta_W} \oplus \underset{W^\bot}{I}$ ist orthogonal/unitär. \\ $\implies \alpha = \beta \circ \gamma$ \end{itemize} \end{proof} \begin{defin} $\alpha \in \Hom(V, V)$ heißt $\genfrac{}{}{0pt}{0}{\text{\underline{positiv definit}}}{\text{\underline{positiv semi-definit}}}$, wenn $\forall v \in V\setminus \{0\}: \inner{\alpha(v)}{v}\genfrac{}{}{0pt}{0}{>}{\ge}0$ \end{defin} \begin{lemma} Sei $\alpha$ selbstadjungiert. Dann gilt \begin{align*} \alpha \text{ positiv definit} & \iff \text{ Alle Eigenwerte positiv} \\ \alpha \text{ positiv semi-definit} & \iff \text{ Alle Eigenwerte nicht-negativ} \end{align*} \end{lemma} \begin{proof} Sei $(e_1, \dots, e_n)$ ONB aus Eigenvektoren, $(\lambda_1, \dots, \lambda_n), \lambda_i \in \R$ Eigenwerte, $v = \sum_{i=1}^n \mu_i e_i$ \begin{align*} \inner{\alpha(v)}v = \inner{\sum_{i=1}^n \lambda_i \mu_i e_i}{\sum_{j=1}^n \mu_j e_j} = \sum_{i=1}^n \lambda_i \mu_i \overline{\mu_i} = \sum_{i=1}^n \lambda_i \underbrace{\abs{ \mu_i}}_{\ge 0}{}^2 \end{align*} Angenommen $\exists i \in [n]:\lambda_j \le 0 \implies \inner{\alpha(\lambda_j)}{e_j} = \lambda_j \le 0 \implies \alpha$ nicht positiv definit. \\ Angenommen $\forall i \in [n]: \lambda_i > 0 \implies \inner{\alpha(v)}v = \sum_{i=1}^n \lambda_i \abs{ \mu_i }^2 > 0$. \end{proof} \subsubsection{Bemerkung} In der Polarzerlegung ist $\beta$ orthogonal/unitär und $\gamma$ selbstadjungiert \& positiv (semi-)definit. \section[Hauptachsentheorem für symmetrische/hermitesche Matrizen]{Hauptachsentheorem für \\symmetrische/hermitesche Matrizen} \subsubsection{Ziel} Klassifizierung aller Skalarprodukte. \begin{defin} \begin{align*} A \in & \R^{\nxn} \text{heißt} & & \text{\underline{symmetrisch}, wenn} & & A = A^T \\ A \in & \C^{\nxn} \text{heißt} & & \text{\underline{hermitesch}, wenn} & & A = A^* \\ A \in & \R^{\nxn} \text{heißt} & & \text{\underline{schiefsymmetrisch}, wenn} & & A = -A^T \\ A \in & \C^{\nxn} \text{heißt} & & \text{\underline{schiefhermitesch}, wenn} & & A = A^* \end{align*} \end{defin} \begin{satz} $V$ euklidischer/unitärer Vektorraum, $\dim(V)< \infty$. Dann gilt: \begin{itemize} \item $\alpha$ selbstadjungiert $\iff \exists$ ONB $B$ mit ${}_B M(\alpha)_B$ \\ symmetrisch/hermitesch. \item $\alpha$ anti-selbstadjungiert $\iff \exists$ ONB $B$ mit ${}_B M(\alpha)_B$ \\ schiefsymmetrisch/schiefhermitesch. \end{itemize} \end{satz} \begin{satz} \label{theo:3.4.3} $A$ symmetrisch/hermitesch. \\ $\implies \exists$ orthogonale/unitäre Matrix $P$ mit $D = P^{-1} A P$ reelle Diagonalmatrix \end{satz} \begin{proof} Sei $E = \{e_1, \dots, e_n\} \subseteq \K^n$ kanonische Basis, \\ $\varphi_A : \begin{cases} \K^n & \to \K^n \\v & \mapsto Av\end{cases} \implies {}_E M(\varphi_A)_E = A \implies {}_E M(\varphi_A^*)_E = A^* = A$ \\ $\implies \varphi_A$ selbstadjungiert. $\implies \exists$ ONB $B=(b_1, \dots, b_n)$ von $\K^n$ aus ~Eigenvektoren von $\varphi_A$, Eigenwerte sind reell. $\implies {}_B M(\varphi_A)_B = D = \diag(\lambda_1, \dots, \lambda_n)$ \\ $\implies {}_B M(\varphi_A)_B = \underbrace{{}_B M(\id)_E}_{P^{-1}} \underbrace{{}_E M(\varphi_A)_E}_{A} \underbrace{{}_E M(\id)_B}_{\underbrace{(b_1, \dots, b_n)}_{P}} \implies D = P^{-1} A P$ \end{proof} \begin{korollar} $A$ symmetrisch/hermitesch $\implies$ Eigenvektoren zu verschiedenen Eigenwerten sind orthogonal. \end{korollar} \subsubsection{Berechnung der Hauptachsentransformation} $A \in \K^{\nxn}$ \begin{enumerate}[label=\arabic*)] \item $\chi_A(\lambda) = \prod_{j=1}^r (\lambda_j - \lambda)^{d_j} $ ($\sum d_j = n$; algebraische Vielfachheit = geometrische Vielfachheit) \item Für jedes $j \in [r]:$ Berechne Basis $B_j = (b_1^j, \dots, b_{d_j}^j)$ von $\ker(A - \lambda_j I)$ \item Orthogonalisiere $B_j$ zu ONS $E_j = (e_1^j, \dots, e_{d_j}^j)$ mittels Gram-Schmidt Verfahren. \item $B = \bigcup_{j=1}^r E_j$ ist die gesuchte Orthonormalbasis.\\ Insbesondere $B^{-1} A B = \begin{pmatrix} \lambda_1 \\ & \ddots \\ & & \lambda_n\end{pmatrix}$ \\ $B^{-1} = \overline B^T$ \end{enumerate} Polarzerlegung $A \in \K^{\nxn}, A^* A$ symmetrisch/hermitesch \[ \implies A^* A = P^* \begin{pmatrix} \lambda_1 \\ & \ddots \\ & & \lambda_n \end{pmatrix} P \] \[ (A^* A)^{\frac 12} = P^* \begin{pmatrix} \sqrt{\lambda_1} \\ & \ddots \\ & & \sqrt{\lambda_n} \end{pmatrix} P \underset{\mathclap{O=AS^{-1}}}{=} S \implies A = \underset{\mathclap{\substack{| \\ \text{orthogonal/unitär}}}}{O} S \] \begin{satz} Sei $V$ ein reeller/komplexer Vektorraum mit $\dim(V) = n, B=(b_1, \dots, b_n)$ Basis. Für $A \in K^{\nxn}$ ist \begin{equation} \label{eq:3.4.5.1} \inner vw := {}_B \Phi(v)^T A {}_B \overline{\Phi(w)} \end{equation} Genau dann ein Skalarprodukt, wenn $A$ symmetrisch/hermitesch und\\ $\underbrace{\text{positiv definit}}_{\substack{\forall \lambda \in \spec(A): \lambda > 0 \\ \rotatebox{90}{\tiny$\iff$} \\ \forall v \in V\setminus\{0\}: \inner{Av}{v} > 0}}$ ist. \\ Umgekehrt: Sei $\inner ..$ Skalarprodukt und $B$ Basis \[ \implies \inner vw = {}_B \Phi(v)^T \underbrace{(\inner{b_i}{b_j})_{i,j=1}^n}_A {}_B \overline{\Phi(w)} \] ($A = P^* \left( \begin{smallmatrix} \lambda_1 \\ & \ddots \\ & & \lambda_n\end{smallmatrix} \right) P \implies \inner vw = {}_B \Phi(v)^T P^* \left( \begin{smallmatrix} \lambda_1 \\ & \ddots \\ & & \lambda_n\end{smallmatrix} \right) P {}_B \overline{\Phi(w)}$ $\implies$ in geeigneter Basis ist jedes Skalarprodukt durch $\sum_{i=1}^n \mu_i \overline \lambda_i$ gegeben.) \end{satz} \begin{proof} Nur für $\K=\C$. Angenommen \ref{eq:3.4.5.1} ist Skalarprodukt. \begin{align*} \implies & \underset{\rotatebox{90}{$=$}}{\inner{b_i}{b_j}} = {}_B \Phi(b_i)^T A {}_B \overline{\Phi(b_j)} = e_i^T A e_j = a_{ij} \\ & \overline{\inner{b_j}{b_i}} = \overline{a_{ji}} \\ \implies & a_{ij} = \overline{a_{ji}} \implies A \text{ hermitesch} \end{align*} Weiters muss $A$ positiv definit sein: Angenommen $\exists x \in \C^n \setminus \{0\}: x^T Ax = 0 \implies v := \sum x_i b_i$, das heißt ${}_B \Phi(v) = x$ erfüllt $\inner vv = {}_B \Phi(v)^T A {}_B \overline{\Phi(v)} = x^T A \overline x = 0$ \\ Sei $A$ hermitesch \& positiv definit. Klarerweise gilt dann für \\ $\inner uv := {}_B \Phi(u)^T A {}_B \overline{\Phi(v)}$: \begin{align*} \inner{u+v}{w} & = \inner uw + \inner vw \\ \inner uv & = \overline{\inner vu} \\ \inner{\lambda u}{v} & = \lambda \inner uv \end{align*} Bleibt zu zeigen, dass $\forall v \in V\setminus\{0\}: \inner vv > 0$ \\ Satz \ref{theo:3.4.3} $\implies \exists$ unitäre Matrix $U$ mit $A = \underset{\substack{\rotatebox{90}{$=$}\\U^{-1}}}{U^*} \underbrace{\diag(\lambda_1, \dots, \lambda_n)}_\Sigma U, \lambda_i \in (0, \infty)\forall i \in [n]$ \begin{align*} & \inner vv={}_B \Phi(v)^T A {}_B \overline{\Phi(v)} = {}_B \Phi(v)^T U^* \Sigma U {}_B \overline{\Phi(v)} = \underset{\substack{\rotatebox{90}{$=$} \\\sum \lambda_i \abs{ x_i }^2 > 0}} {(\overline U {}_B\Phi(v))^T \Sigma \overline{\overline U {}_B (v)}} \\ & v \neq0 \implies \overline U {}_B \Phi(v) = \left( \begin{smallmatrix} x_1 \\ \vdots \\ x_n \end{smallmatrix} \right) \neq 0 \end{align*} \end{proof} \begin{defin} Sei $A \in \K^{\nxn}$ eine symmetrische/hermitesche Matrix. \begin{itemize} \item \[ t(A) := \abs{ \{\lambda \in \spec(A): \lambda > 0 \} } \] heißt \underline{Trägheitsindex} von $A$. \item $A, B$ heißen \underline{kongruent} wenn eine invertierbare Matrix $Q \in \K^{\nxn}$ existiert mit $B = Q^* A Q$ \end{itemize} \end{defin} \subsubsection{Bemerkung} $M_B(\sigma), M_{B'}(\sigma)$ sind kongruent (\& umgekehrt) \begin{satz} [Trägheitssatz von Sylvester] Sei $A \in \K^{\nxn}$ symmetrisch/hermitesch mit $\rg(A) = r, t(A)=t$. Dann gilt: \begin{enumerate}[label=\alph*)] \item Es gibt $S \in \K^{\nxn}$ invertierbar mit \[ S^* A S = \diag(\underbrace{1, \dots, 1}_t, \underbrace{-1, \dots, -1}_{r-t}, \underbrace{0, \dots, 0}_{n-r}) \] $A$ ist zu dieser Matrix kongruent. \item $A, B$ kongruent $\iff t(A) = t(B) \land \rg(A) = \rg(B)$ \end{enumerate} \subsubsection{Bemerkung} Trägheitsindex und Rang charakterisieren symmetrische Bilinearformen / hermitesche Sesquilinearformen komplett. \end{satz} \begin{proof} Nur $\K = \C$ \begin{enumerate}[label=\alph*)] \item Satz \ref{theo:3.4.3} $\implies \exists P$ unitär mit $P^* AP = \diag(\lambda_1, \dots, \lambda_n)$ o.B.d.A. $\lambda_1, \dots, \lambda_t > 0, \lambda_{t+1}, \dots, \lambda_{r} < 0, \lambda_{r+1}, \dots, \lambda_n = 0$. \\ Setze $T = \begin{pmatrix} \frac{1}{\sqrt{\abs{ \lambda_1 }}} \\ & \ddots \\ & & \frac{1}{\sqrt{\abs{ \lambda_r }}} \\ & & & 1 \\ & & & & \ddots \\ & & & & & 1 \end{pmatrix}$ $\implies S := PT$ ist invertierbar. \begin{align*} S^* A S & = T \underbrace{P^* A P}_{\mathclap{\diag{\lambda_1, \dots, \lambda_n}}} T = T \begin{pmatrix} \lambda_1 \\ & \ddots \\ & & \lambda_n \end{pmatrix} T \\ & = \begin{pmatrix} 1 \\ & \ddots \\ & & 1 \\ & & & -1 \\ & & & & \ddots \\ & & & & & -1 \\ & & & & & & 0 \\ & & & & & & & \ddots \\ & & & & & & & & 0 \end{pmatrix} \end{align*} \item $A, B$ kongruent, das heißt $\exists Q$ invertierbar mit $B = Q^* A Q$\\ $ \implies \rg(B) = \rg(A)$. Satz \ref{theo:3.4.3} $\implies \exists P_1, P_2$ unitär mit \begin{align*} P_1^* A P_1 & = \begin{pmatrix} \lambda_1 \\ & \ddots \\ & & \lambda_n \end{pmatrix} = D \\ P_2^* A P_2 & = \begin{pmatrix} \mu_1 \\ & \ddots \\ & & \mu_n \end{pmatrix} = G \end{align*} $t(A) := t, t(B) := s$ \\ Ordne so, dass $\ontop{\lambda_1, \dots, \lambda_t > 0, \lambda_{t+1}, \dots, \lambda_r < 0, \lambda_{r+1}, \dots, \lambda_n = 0} {\mu_1, \dots, \mu_s > 0, \mu_{s+1}, \dots, \mu_r < 0, \mu_{r+1}, \dots, \mu_n =0}$ \\ Setze $a_i := \sqrt{\abs{\lambda_i}}, b_i := \sqrt{\abs{\mu_i}}$ \begin{equation} \label{eq:3.4.6.1} x^* D x = \sum_{j=1}^t a_j^2 \abs{ x_j }^2 - \sum_{j=t+1}^r a_j^2 \abs{x_j}^2 \end{equation} $C := P_2 Q^{-1} P_1, y := Cx$ \begin{equation} \label{eq:3.4.6.2} x^* D x = x^* C^* G C x = y^* G y = \sum_{j=1}^s b_j^2 \abs{ y_j }^2 - \sum_{j=s+1}^r b_j^2 \abs{ y_j }^2 \end{equation} Angenommen $t 0 & & \text{\Lightning} \end{align*} $\implies s = t$ \end{enumerate} \end{proof} \subsubsection{Berechnung des Trägheitsindex:} Sei $A$ symmetrisch hermitesch, $\det(A) \neq 0 \implies \chi_A(\lambda) = a_1 \lambda^n + \dots + \underset{\mathrlap{\rotatebox{325}{\scriptsize$\neq 0$}}}{a_0}$ mit $a_i \in \R$ ist Polynom mit lauter reellen Nullstellen. \begin{satz} [Vorzeichenregel von Descartes] Sei $p$ Polynom mit $p(0) \neq 0$, reellen Koeffizienten und lauter reellen Nullstellen. Dann gilt \[ \abs{ \{ \lambda: p(\lambda) = 0 \land \lambda > 0 \} } = \abs{ i \in \{0, \dots n-1 \}: a_i a_{i+1} < 0 } \] \end{satz} \section{Bilinearformen und Sesquilinearformen} \begin{defin} \begin{itemize} \item $\sigma: V\times V \to \K$ mit $\forall u,v,w \in V, \lambda \in \K$: \begin{align} \sigma(v+w, u) & = \sigma(v, u) + \sigma(w, u) \nonumber \\ \sigma(v, w+u) & = \sigma(v, w) + \sigma(v, u) \nonumber \\ \sigma(\lambda v, w) & = \lambda \sigma(v, w) \nonumber \\ \sigma(v, \lambda w) & = \lambda \sigma(v, w) \label{eq:3.5.1.1} \end{align} heißt \underline{Bilinearform}. \item Falls $\K = \C$ und anstelle von \ref{eq:3.5.1.1} gilt, dass $\sigma(v, \lambda w) = \overline \lambda \sigma(v,w)$, so heißt $\sigma$ \underline{Sesquilinearform}. \item Eine Bilinearform heißt \underline{symmetrisch}, wenn $\sigma(u, v) = \sigma(v, u)$ und \\ \underline{alternierend}, wenn $\sigma(u, v) = -\sigma(v, u)$. \item Eine Sesquilinearform heißt \underline{hermitesch}, wenn $\sigma(u, v) = \overline{\sigma(v, u)}$ \end{itemize} \end{defin} \subsubsection{Beispiel} \begin{itemize} \item Euklidisches Skalarprodukt ist symmetrische Bilinearform. \item Unitäres Skalarprodukt ist hermitesche Bilinearform. \item \begin{equation} \label{eq:3.5.1.2} \sigma(x, y) = x_1 y_1 + x_1 y_2 + x_2 y_1 - 5 x_2 y_2 = \begin{pmatrix} x_1 & x_2 \end{pmatrix} \begin{pmatrix} 1 & 1 \\ 1 & -5 \end{pmatrix} \begin{pmatrix} y_1 \\ y_2 \end{pmatrix} \end{equation} $V=\R^2$ $ q(x) := \sigma(x, x) = x_1^2 + x_1 x_2 + x_2 x_1 - 5x_2^2, \R^2 \to \R$ \end{itemize} Sei $ B= (b_1, \dots, b_n)$ Basis, so ist $M_B(\sigma) := (\sigma(b_i, b_j))_{i,j=1}^n$ \begin{lemma} \label{theo:3.5.2} \begin{enumerate}[label=\alph*)] \item Es gilt für $\sigma$ Bilinearform und $B$ Basis, dass \[ \sigma(u, v) = {}_B \Phi(u)^T M_B(\sigma) {}_B \Phi(v) \; \forall u, v \] \item Es gilt für $\sigma$ Sequilinearform, dass \[ \sigma(u, v) = {}_B \Phi(u)^T M_B(\sigma) {}_B \overline{\Phi(v)} \; \forall u, v \] \item Sei $B'$ eine weitere Basis und $\K = \R$ \[ M_{B'}(\sigma) = {{}_B M(\id)_{B'}}^T \, M_B(\sigma) \, {}_B M(\id)_{B'} \] \item Sei $B'$ eine weitere Basis und $\K = \C$ \[ M_{B'}(\sigma) = {{}_B M(\id)_{B'}}^T \, M_B(\sigma) \, {}_B \overline{M(\id)}_{B'} \] \item $\sigma$ symmetrisch/hermitesch $\iff M_B(\sigma)^* = M_B(\sigma)$ \end{enumerate} \end{lemma} \begin{proof} \begin{enumerate}[label=\alph*)] \item Analog wie b) \item $u = \sum \lambda_i b_i, v=\sum \mu_j b_j, A = M_B(\sigma)$ \begin{align*} \sigma(u, v) & = \sigma\left(\sum \lambda_i b_i, \sum \mu_j b_j\right) = \sum_{i=1}^n \lambda_i \underbrace{\sum_{j=1}^n \underbrace{\sigma(b_i, b_j)}_{a_{ij}} \overline \mu_j} _{A {}_B\overline{\Phi(v)}} \\ & = {}_B \Phi(u)^T M_B(\sigma) {}_B \overline{\Phi(v)} \end{align*} \item Analog wie d) \item $b'_i = \sum_k a_{ki} b_k, M_{B'} = (\sigma(b'_i, b'_j))_{i,j}$ \begin{align*} \sigma(b'_i, b'_j) & = \sigma\left(\sum_k a_{ki} b_k, \sum_l a_{lj} b_l\right) = \sum_k a_{ki} \sum_l \underbrace{\sigma(b_i,b_j)}_{M_B(\sigma)} \overline{a_{lj}} \\ & = \left(A^T M_B(\sigma)\overline A\right)_{ij} \end{align*} \item \begin{align*} \underset{\rotatebox{70}{$=$}}{\sigma(v, w)} & = \underset{\rotatebox{110}{$=$}}{\overline{\sigma(w, v)}} \\ {}_B \Phi(v)^T M_B(\sigma) {}_B \overline{\Phi(w)} & = \left({}_B\overline{\Phi(w)}^T \overline{M_B(\sigma)} {}_B \Phi(v)\right)^T \\ & = {}_B \Phi(v)^T \overline{M_B(\sigma)}^T {}_B \overline{\Phi(w)} \\ \implies M_B(\sigma) = M_B(\sigma)^* \end{align*} \end{enumerate} \end{proof} \begin{satz} Sei $V$ euklidischer/unitärer Vektorraum und $\sigma$ symmetrische/hermitesche Bilinear-/Sesquilinearform. Dann existiert eine Orthonormalbasis $B$ mit \\ $M_B(\sigma)$ reelle Diagonalmatrix. \end{satz} \begin{proof} Nach Lemma \ref{theo:3.5.2} e) gilt $M_B(\sigma)^* = M_B(\sigma)$ und daher nach Satz \ref{theo:3.4.3} gibt es orthogonale/unitäre Matrix $U$ mit \[ U^* M_B(\sigma) U = \left(\begin{smallmatrix} \lambda_1 \\ & \ddots \\ & & \lambda_n \end{smallmatrix}\right), \lambda_i \in \R \forall i \in [n] \] Behauptung folgt dann aus Lemma \ref{theo:3.5.2} d). \end{proof} \subsubsection{Beispiel} $\sigma$ wie in \ref{eq:3.5.1.2}, $A = \begin{pmatrix} 1 & 1 \\ 1 & -5 \end{pmatrix}$ \\ $\chi_a(\lambda) = \det\begin{pmatrix} 1-\lambda & 1 \\ 1 & -5-\lambda \end{pmatrix} = (\lambda -1)(5 + \lambda) - 1$ \\ Nullstellen: $\lambda_1, \lambda_2 = -2 \pm \sqrt{10}$\\ $b_1 = \frac{1}{\sqrt{20+6\sqrt{10}}} \begin{pmatrix} 3 + \sqrt{10} \\ 1 \end{pmatrix}, b_2 = \frac{1}{\sqrt{20-6\sqrt{10}}} \begin{pmatrix}3 - \sqrt{10} \\ 1 \end{pmatrix}$ \\ $M_B(\sigma) = \begin{pmatrix} -2 + \sqrt{10} & 0 \\ 0 & -2 -\sqrt{10} \end{pmatrix}$ \\ $ \implies q(\tilde x_1, \tilde x_2) = \lambda_1 \tilde x_1^2 + \lambda_2 \tilde x_2^2 \equiv c$ \begin{tikzpicture}[scale=1.8] \tikzmath{ \bfactor1 = 1 / sqrt(20 + 6* sqrt(10)); \bfactor2 = 1 / sqrt(20 - 6* sqrt(10)); \bx1 = \bfactor1 * (3 + sqrt(10)); \by1 = \bfactor1; \bx2 = \bfactor2 * (3 - sqrt(10)); \by2 = \bfactor2; \brichtung1 = \by1 / \bx1; \brichtung2 = \by2 / \bx2; } \begin{axis}[ title=\scriptsize{Niveaulinien von $q(x)$}, xlabel={$x_1$}, ylabel={$x_2$}, ymin=-2.7,ymax=2.7, xmin=-3.3,xmax=3.3, view={0}{90}, axis lines=middle, tick label style={font=\tiny}, label style={font=\scriptsize}, ] \addplot3 [ contour gnuplot={ levels={0,-1,-4,-9,-16,-25,-36,1,4,9}, contour label style={every node/.append style={text=ForestGreen}}, label distance = 90pt, }, samples=80, domain=-3.3:3.3, domain y=-2.7:2.7, contour/draw color={ForestGreen}, ] {x^2 + 2*x*y - 5*y^2}; \draw [->, red, thick] (0,0) -- (\bx1, \by1) node[above]{\footnotesize$b_1$}; \draw [->, red, thick] (0,0) -- (\bx2, \by2) node[above right]{\footnotesize$b_2$}; \addplot [ domain=-3.3:3.3, color=red, style={dash pattern=on 3pt off 2pt on 15pt off 2pt}, ] {x * \brichtung1}; \addplot [ domain=-3.3:3.3, color=red, style={dash pattern=on 3pt off 2pt on 15pt off 2pt}, ] {x * \brichtung2}; \end{axis} \end{tikzpicture} \begin{defin} Sei $\rho: V \to \K$ heißt \underline{quadratische Form} wenn $\forall u, v \in V, \lambda \in \K:$ \begin{enumerate}[label=\alph*)] \item $\rho(\lambda v) = \lambda^2 \rho(v)$ \item $ \sigma(u, v) := \rho(u + v) - \rho(u) - \rho (v)$ ist eine (symmetrische) Bilinearform \end{enumerate} \end{defin} \begin{lemma} Sei $\operatorname{char}(\K) \neq 2$. Dann entsprechen die quadratischen Formen und symmetrischen Bilinearformen einander eineindeutig. \end{lemma} \begin{proof} $\rho$ quadratische Form $\implies \sigma(v, w) = \rho(u + v) - \rho(u) - \rho(v)$ ist symmetrische Bilinearform. \\ Sei umgekehrt $\sigma$ symmetrische Bilinearform, $\rho(v) := \underset{\mathclap{\substack{\rotatebox{90}{$\to$}\\\operatorname{char}(\K) \neq 2}}} {\frac 12} \sigma(v, v)$. \begin{align*} \rho(\lambda v) = \frac 12 \sigma(\lambda v, \lambda v) & = \lambda^2 \frac 12 \sigma(v, v) = \lambda^2 \rho(v) \implies \text{a)} \\ \rho(u+v) - \rho(u) - \rho(v) & = \frac 12(\sigma(u + v, u + v) - \sigma(u, u) - \sigma(v, v)) \\ & \begin{multlined} = \frac 12(\cancel{\sigma(u, u)} + \sigma(u, v) + \sigma(v, u) + \cancel{\sigma(v, v)} \\ - \cancel{\sigma(u, u)} - \cancel{\sigma(v, v)})\end{multlined} \\ & = \sigma(u, v) \text{ ist symmetrische Bilinearform.} \end{align*} \end{proof} \begin{defin} \label{theo:3.5.6} Sei $V\, \C$-VR. $\rho: V \to \R$ heißt \underline{hermitesche Form} wenn $\forall u, v \in V, \lambda \in \C$: \begin{enumerate}[label=\alph*)] \item $\rho(\lambda v) = \abs{\lambda}^2 \rho(v)$ \item $\rho(u+v) + \rho(u -v) = 2(\rho(u) + \rho(v))$ \item $\sigma(u,v) := \frac 12 (\rho(u+v) + i\rho(u +iv) - (1+i)(\rho(u) + \rho(v)))$ ist hermitesche Sesquilinearform. \end{enumerate} \end{defin} \begin{lemma} Hermitesche Formen und hermitesche Sesquilinearformen entsprechen einander eineindeutig \end{lemma} \begin{proof} Für hermitesche Form ist durch Definition \ref{theo:3.5.6} c) eine hermitesche Sesquilinearform definiert. \\ Sei umgekehrt $\sigma$ hermitesche Sesquilinearform. Dann ist $\rho(v) := \frac12 \sigma(v, v)$ hermitesche Form: \begin{enumerate}[label=\alph*)] \item \checkmark \item \begin{align*} \rho(u+v) + \rho(u - v) &= \sigma(u+v, u+v) + \sigma(u-v, u-v) \\ &\begin{multlined}= \sigma(u, u) + \sigma(v, v) + \sigma(u, v) + \sigma(v, u) + \sigma(u, u)\\ + \sigma(v, v) - \sigma(u, v) - \sigma(v, u) \end{multlined} \\ &= 2\sigma(u, u) + 2\sigma(v, v) \\ &= 2(\rho(u) + \rho(v)) \end {align*} \item \begin{align*} \frac12 (\rho(u+v) + i\rho(u+iv) & - (1+i)(\rho(u)+\rho(v))) = \\ & \begin{multlined} = \sigma(u+v, u+v) + i \sigma(u+iv,u+iv) \\- \sigma(u, u) - \sigma(v, v) - i\sigma(u, u) - i \sigma(v, v) \end{multlined} \\ & = \sigma(u, v) + \sigma(v, u) + i \sigma(iv, u) + i \sigma(u, iv) \\ & = \sigma(u, v) + \overline{\sigma(u, v)} + i \overline{\sigma(u, iv)} + \sigma(u, v) \\ & = \sigma(u, v) + \overline{\sigma(u, v)} + i \cdot \overline{\overline{i}} \cdot \overline{\sigma(u, v)} + \sigma(u, v) \\ & = 2 \sigma(u, v) \end{align*} \end{enumerate} \end{proof} \subsubsection{Bemerkung} $\sigma$ heißt Polarform von $\rho$ \section[Die Singulärwertzerlegung und die Pseudoinverse]{Die Singulärwertzerlegung und die \\Pseudoinverse} Wir wollen nun für zwei euklidische Vektorräume $V, W$ eine geeignete Normalform \\ bezüglich Orthonormalbasen herleiten. Polarzerlegung besagt für $\alpha \in \Hom(V, V)$, dass Orthonormalbasen $B, B'$ von $V$ existieren mit \[ {}_B M(\alpha)_B = \begin{pmatrix} s_1 \\ & \ddots \\ & & s_r \\ & & & 0 \\ & & & & \ddots \\ & & & & & 0 \end{pmatrix}, s_1, \dots, s_n > 0 \] Das heißt $\alpha$ lässt sich aus orthogonalen Endomorphismen und Skalierung zusammensetzen. \begin{satz} [Singulärwertzerlegung] Sei $A \in \R^{m \times n} / \C^{m \times n}$. Dann gibt es orthogonale/unitäre Matrizen $U, V$ sowie $s_1, \dots, s_r \in (0, \infty)$ mit \[ A = \underbrace{U}_{\K^{m\times m}} \underbrace{\begin{pmatrix}s_1 \\ & \ddots \\ & & s_r \\ & & & 0 \\ & & & & \ddots \\ & & & & & 0\end{pmatrix}}_{\K^{m \times n}} \underbrace{V}_{\K^{n \times n}} \] $s_1, \dots, s_r$ heißen \underline{Singulärwerte} von $A$. \end{satz} \begin{proof} \begin{itemize} \item $A^* A \in \K^{\nxn}$ selbstadjungiert und positiv semi-definit. \\ Eigenwerte $\lambda_1, \dots, \lambda_n \in [0, \infty)$, ONB $b_1, \dots, b_n$ aus Eigenvektoren. Sei $\lambda_1, \dots, \lambda_r \in (0, \infty), \lambda_{r+1} = \dots = \lambda_n = 0$ $s_i := \sqrt{\lambda_i}, i\in [n]$ \item Es gilt, dass $\overbrace{\frac 1{s_1} A b_1}^{b_1'}, \dots, \overbrace{\frac 1{s_r} A b_r}^{b_r'}$ Orthonormalsystem in $\K^m$ ist. \begin{align*} \overline{\inner{Ab_i}{Ab_j}}_{\K^m} & = \overline{b_i^T A^T \overline A \,\overline{b_j}} = \overline{b_i}^T A^* A b_j = \lambda_j \overline{b_i}^T b_j \\ & = \lambda_j \overline{\inner{b_i}{b_j}}_{\K^n} = \lambda_j \delta_{ij} \in \R \end{align*} \item Ergänze $b_1', \dots, b_r'$ zu Orthonormalbasis $b_1', \dots, b_r', \dots, b_m'$ von $\K^m$. \\ Sei $\varphi_A: x \mapsto A\cdot x \implies {}_{B'} M(\varphi_A)_B = \left( \begin{smallmatrix} s_1 \\ & \ddots \\ & & s_r \\ & & & 0 \\ & & & & \ddots \\ & & & & & 0 \end{smallmatrix} \right)$ \\ $v = \sum \mu_i b_i \implies Av = \sum \mu_i \underbrace{A b_i}_{s_i b_i'} = \sum \mu_i s_i b_i'$ \end{itemize} \end{proof} Mittels der Singulärwertzerlegung können wir für jede Matrix (bzw. lineare Abbildung) eine verallgemeinerte Inverse berechnen. Sei ${}_B M(\alpha)_B = \begin{pmatrix}s_1 \\ & \ddots \\ & & s_r \\ & & & 0 \\ & & & & \ddots \\ & & & & & 0\end{pmatrix}$ \\ $\implies \ker(\alpha) = \linspan{ b_{r+1}, \dots, b_n }_V, \im(\alpha) = \linspan{b'_1, \dots b_r'}, \ker(\alpha)^\bot = \linspan{ b_1, \dots, b_r }_V$ \begin{align*} \alpha: V & \to \ker(\alpha)^{\bot} & & \overset{\beta}{\to} \im(\alpha) & & \to W \\ \sum_{i=1}^n \lambda_i b_i & \mapsto \sum_{i=1}^r \lambda_i b_i & & \mapsto \sum_{i=1}^r s_i \lambda_i b_i' & & \mapsto \sum_{i=1}^r s_i \lambda_i b_i' \\ \left(\begin{smallmatrix}x_1 \\ \vdots \\ x_n\end{smallmatrix}\right) & \mapsto \left(\begin{smallmatrix}x_1 \\ \vdots \\ x_r\end{smallmatrix}\right) & & \mapsto \left(\begin{smallmatrix}s_1 x_1 \\ \vdots \\ s_r x_r\end{smallmatrix}\right) & & \mapsto \left.\left(\begin{smallmatrix} s_1 x_1 \\ \vdots \\ s_r x_r \\ 0 \\ \vdots \\ 0 \end{smallmatrix}\right)\right\} m \\ \sum_{i=1}^r \frac{\mu_i}{s_i} b_i & \mapsfrom \sum_{i=1}^r \frac{1}{s_i} \mu_i b_i & & \underset{\beta^{-1}}{\mapsfrom} \sum_{i=1}^r \mu_i b_i' & & \mapsfrom \sum_{i=1}^m \mu_i b_i' = w \cdot \alpha^{+} \end{align*} \subsubsection{Bemerkung:} $\alpha$ invertierbar, $V=W \implies n = m = r \implies \alpha^\dagger = \alpha^{-1}$ \\ Wir haben eine echte Verallgemeinerung. \begin{defin} \leavevmode \begin{itemize} \item Sei $A \in \K^{m \times n}, \K \in \{\R,\C\}$ mit \[ A = U^* \Sigma V, \Sigma = \left( \begin{smallmatrix} s_1 \\ & \ddots \\ & & s_r \\ & & & 0 \\ & & & & \ddots \\ & & & & & 0 \end{smallmatrix} \right) \] Dann heißt die Matrix \[ A^\dagger = V^* \Sigma^\dagger U \in \K^{n \times m}, \Sigma^\dagger = \left( \begin{smallmatrix} \frac1{s_1} \\ & \ddots \\ & & \frac1{s_r} \\ & & & 0 \\ & & & & \ddots \\ & & & & & 0 \end{smallmatrix} \right) \] (Moore-Penrose) \underline{Pseudoinverse} von $A$. \item Sei $\alpha \in \homk(V, W), \dim(V), \dim(W) < \infty$ und $B, B'$ Orthonormalbasen mit ${}_{B'} M(\alpha)_B = \Sigma$ und $\alpha^\dagger$ so, dass ${}_B M(\alpha^\dagger)_B = \Sigma^\dagger$. Dann heißt $\alpha^\dagger$ (Moore-Penrose) \underline{Pseudoinverse} von $\alpha$. \end{itemize} \end{defin} \begin{satz} \label{theo:3.6.3} Seien $V, W$ endlich dimensional euklidische/unitäre Vektorräume, \\ $\alpha \in \Hom(V, W)$. Dann gilt: \[ \alpha^\dagger \text{ ist pseudoinverse} \iff \begin{aligned} & \alpha \circ \alpha^\dagger \circ \alpha = \alpha \\ & \alpha^\dagger \circ \alpha \circ \alpha^\dagger = \alpha^\dagger \\ & \alpha \circ \alpha^\dagger \text{ selbstadjungiert} \\ & \alpha^\dagger \circ \alpha \text{ selbstadjungiert} \\ \end{aligned} \] \end{satz} \begin{proof} Beweis über Matrizen, da äquivalent. Weiters die $\implies$ Richtung nur für \R. \begin{itemize} \item[$\implies$:] $A = U^T \Sigma V, A^\dagger = V^T \Sigma^\dagger U$ \[ A A^\dagger = U^T \Sigma \underbrace{V V^T}_{=I} \Sigma^\dagger U = U^T \Sigma \Sigma^\dagger U = U^T \left( \begin{smallmatrix} 1 \\ & \ddots \\ & & 1 \\ & & & 0 \\ & & & & \ddots \\ & & & & & 0 \end{smallmatrix} \right) U \] $A^\dagger A$. \[ A A^\dagger A = U^T \Sigma \underbrace{V V^T}_I \Sigma^\dagger \underbrace{U U^T}_I \Sigma V = U^T \underbrace{\Sigma \Sigma^\dagger \Sigma}_\Sigma V = U^T \Sigma V = A \] \item[$\impliedby$:] \begin{itemize} \item Sei $\alpha \in \Hom(V, W), \alpha^\dagger \in \Hom(W, V)$ \begin{equation} \label{eq:3.6.3.1} \begin{aligned} \ker(\alpha) = \ker(\alpha^\dagger \circ \alpha) & & \im(\alpha) = \im(\alpha \circ \alpha^\dagger) \\ \ker(\alpha^\dagger) = \ker(\alpha \circ \alpha^\dagger) & & \im(\alpha^\dagger) = \im(\alpha^\dagger \circ \alpha) \end{aligned} \end{equation} $\ker(\alpha) \subseteq \ker(\alpha^\dagger \circ \alpha) \subseteq \ker(\alpha \circ \alpha^\dagger \circ \alpha) = \ker(\alpha) \implies \ker(\alpha) = \ker(\alpha^\dagger \circ \alpha)$ $\im(\alpha) \subseteq \im(\alpha \circ \alpha^\dagger) \subseteq \im(\alpha \circ \alpha^\dagger \circ \alpha) = \im(\alpha) \implies \im(\alpha) = \im(\alpha \circ \alpha^\dagger)$ \item $\nu := \alpha^\dagger \circ \alpha$ erfüllt $\nu \circ \nu$ und ist selbstadjungiert ? für $\nu' := \alpha \circ \alpha^\dagger$ \\ $\implies \underbrace{\ker(\nu)}_{=\ker(\alpha)} \bot \im(\nu)$ [Sei $v\in \ker(\nu), w = \nu(v) \in \im(\nu) \implies \inner vw = \inner{\nu(v)}{w} = 0$] \\ $\implies$ \begin{enumerate}[label=\alph*)] \item $\nu(v) \in \im(\nu)$ \item $\forall u \in \im(\nu), v \in V:$ \begin{align*} \inner{\nu(v) - v}{u} & = \inner{\nu(v) - v}{\nu(w)} = \inner{\nu^2(v) - \nu(v)}{w} \\ & = \inner{\nu(v) - \nu(v)}{w} = 0 \end{align*} \end{enumerate} $\implies (b_1, \dots, b_n)$ ONB mit $\linspan{ b_{r+1}, \dots, b_n }_V = \ker(\nu) = \ker(\alpha)$ \begin{align*} & \sum_{i=1}^n \lambda_i b_i & & \overset{\nu}{\mapsto} \sum_{i=1}^r \lambda_i b_i \text{[$\nu$ ist orthogonale Projektion auf $\ker(\alpha)^\bot$]} \\ \text{Analog:} & \sum_{i=1}^m \mu_i b_i' & & \overset{\nu'}{\mapsto} \sum_{i=1}^r \mu_i b_i' \text{[$\nu'$ ist orthogonale Projektion auf $\im(\alpha)$]} \\ \end{align*} \[ \implies {}_B M(\alpha^\dagger \circ \alpha)_B = \left(\begin{smallmatrix} 1 \\ & \ddots \\ & & 1 \\ & & & 0 \\ & & & & \ddots \\ & & & & & 0 \end{smallmatrix}\right),\; {}_{B'} M(\alpha \circ \alpha^\dagger)_{B'} = \left(\begin{smallmatrix} 1 \\ & \ddots \\ & & 1 \\ & & & 0 \\ & & & & \ddots \\ & & & & & 0 \end{smallmatrix}\right) \] \begin{equation} \label{eq:3.6.3.2} \begin{split} \implies \underbrace{\left(\begin{smallmatrix} 1 \\ & \ddots \\ & & 1 \\ & & & 0 \\ & & & & \ddots \\ & & & & & 0 \end{smallmatrix}\right)}_n &= {}_B M(\alpha^\dagger)_{B'} \cdot {}_{B'} M(\alpha)_B = \\ &= \begin{pmatrix} a_{11}^\dagger & \dots & a_{1m}^\dagger \\ \vdots & & \vdots \\ a_{n1}^\dagger & \dots & a_{nm}^\dagger \end{pmatrix} \underbrace{\left.\left(\begin{smallmatrix} s_1 \\ & \ddots \\ & & s_r \\ & & & 0 \\ & & & & \ddots \\ & & & & & 0 \end{smallmatrix}\right)\right\}}_n \scriptstyle{m} \end{split} \end{equation} \begin{align*} \underbrace{ \left(\begin{smallmatrix} 1 \\ & \ddots \\ & & 1 \\ & & & 0 \\ & & & & \ddots \\ & & & & & 0 \end{smallmatrix}\right) }_m & = {}_{B'} M(\alpha)_B \cdot {}_B M(\alpha^\dagger)_{B'} = \\ & =\scriptstyle{m}\underbrace{\left\{\left(\begin{smallmatrix} s_1 \\ & \ddots \\ & & s_r \\ & & & 0 \\ & & & & \ddots \\ & & & & & 0 \end{smallmatrix}\right)\right.}_n \begin{pmatrix} a_{11}^\dagger & \dots & a_{1m}^\dagger \\ \vdots & & \vdots \\ a_{n1}^\dagger & \dots & a_{nm}^\dagger \end{pmatrix} \end{align*} Es gilt $\ker(\alpha^\dagger) = \ker(\nu') = \im(\alpha)^\bot = \linspan{ b_{r+1}', \dots, b_r' } \implies a^\dagger_{i\_} = 0 \forall i > r$ \[ \implies {}_B M(\alpha^\dagger)_{B'} = \begin{pmatrix} \begin{smallmatrix} a_{11}^\dagger & \dots & a_{1r}^\dagger \\ \vdots & & \vdots \\ a_{r1}^\dagger & \dots & a_{rr}^\dagger \end{smallmatrix} & 0 \\ 0 & 0 \end{pmatrix} \text{ und } a_{ij}^\dagger = \delta_{ij} \frac{1}{s_i} \text{ wegen \ref{eq:3.6.3.2}} \] \end{itemize} \end{itemize} \end{proof} \begin{satz} Sei $\alpha \in \Hom(V, W)$. \begin{itemize} \item $\alpha$ injektiv $\implies \alpha^\dagger = (\alpha^* \circ \alpha)^{{}^{-1}} \circ \alpha^*$ \item $\alpha$ surjektiv $\implies \alpha^\dagger = \alpha^* \circ (\alpha \circ \alpha^*)^{{}^{-1}}$ \end{itemize} \end{satz} \begin{proof} Sei $\alpha$ injektiv $\implies \alpha^* \circ \alpha$ bijektiv. Angenommen $\alpha^* \circ \alpha$ nicht surjektiv $\implies$ \begin{align*} \exists w \in V \setminus \{0\}: \forall v \in V: \inner{\alpha^* \circ \alpha(v)}{w} = 0 \\ \implies \forall v: \inner{\alpha(v)}{\alpha(w)}_W = 0 \\ \implies \alpha(w) \in \im(\alpha)^\bot \cap \im(\alpha) \implies \alpha(w) = 0 \\ \overset{\alpha \text{ injektiv}}{\implies} w = 0 & \text{\Lightning} \end{align*} $\implies \beta:= (\alpha^* \circ \alpha)^{{}^{-1}} \circ \alpha^*$ ist wohldefiniert. Nun gilt: \begin{itemize} \item $\alpha \circ \beta \circ \alpha = \alpha \circ (\alpha^* \circ \alpha)^{{}^{-1}} \circ \alpha^* \circ \alpha = \alpha$ \item $\beta \circ \alpha \circ \beta = (\alpha^* \circ \alpha)^{{}^{-1}} \circ \alpha^* \circ \alpha \circ \beta = \beta$ \item $\beta \circ \alpha, \alpha \circ \beta$ sind selbstadjungiert. \end{itemize} $\underset{\text{Satz \ref{theo:3.6.3}}}{\implies} \beta = \alpha^\dagger$ \end{proof} \subsubsection{Anwendung: Methode der kleinsten Quadrate} Sei $Ax = b$ Lineares Gleichungssystem mit $L(A,b) = \emptyset$. Versuche ein $x$ zu finden mit $\norm{Ax-b}_{\K^m}$ minimal, $\norm{\alpha(v) - w}$ minimal. Sei $b_1, \dots, b_n$ Orthonormalbasis von $V$, \\ $b_1', \dots, b_m'$ ONB von $W$. $\linspan{ b_1, \dots, b_r } = \ker(\alpha)^\bot, \linspan {b_1', \dots b_r'} = \im(\alpha)$ $v = \sum_{i=1}^n \lambda_i b_i \implies \alpha(v) = \sum_{i=1}^r s_i \lambda_i b_i'$ $w = \sum_{i=1}^n \mu_i b_i'$ \[ \norm{\alpha(v) - w}^2 = \norm{\sum_{i=1}^r s_i \lambda_i b_i' - \sum_{i=1}^m \mu_i b_i'}^2 = \sum_{i=1}^r (s_i \lambda_i - \mu_i)^2 + \sum_{i = r+1}^m \mu_i^2 \] Wird minimal wenn $\lambda_i = \frac{\mu_i}{s_i}, i \in [r]$, das heißt das optimale $v$ ist durch $v^\dagger = \alpha^\dagger(w)$ gegeben. $\left[\text{Alle optimalen durch }v^\dagger + \sum_{j=r+1}^m \mu_j^2 = L(\alpha^* \alpha, \alpha^*(w))\right]$ \begin{satz} Sei $\alpha \in \homk(V, W), \K \in \{\R,\C\}, V, W$ endlich dimensional. Sei $w \in W$. Dann gilt mit $v^\dagger = \alpha^\dagger(w)$ dass \[ \norm{\alpha(v^\dagger)-w} = \min_{v\in V} \norm{\alpha(v) - w} \] Alle Vektoren mit dieser Eigenschaft erfüllen die\\ \underline{Normalgleichungen} $\ontop{\alpha^* \alpha(v) = \alpha^*(w)}{A^* A x = A^* b}$ \end{satz} \begin{proof} Angenommen $L(A, b) \neq \emptyset$. $\to$ Suche $v \in L(A, b)$ mit minimaler Norm: Sei $w \in \im(\alpha) \implies w = \sum_{j=1}^r \mu_j b_j'$ \[ \implies L(\alpha, w) = \left\{ \sum_{j=1}^r \frac{\mu_j}{s_j} b_j + \sum_{j=r+1}^n \lambda_j b_j: \lambda_{r+1}, \dots, \lambda_n \in \K\right\} \] Minimale Norm, wenn $\lambda_{r+1}, \dots, \lambda_n = 0$ das heißt für $v = \sum_{i=1}^r \frac{\mu_i}{s_i} b_i = \alpha^\dagger(w)$ \end{proof} \begin{satz} Sei $\alpha \in \Hom(V, W), w \in \im(\alpha)$. Dann gilt mit $v^\dagger = \alpha^\dagger (w)$: \[ \norm{v^\dagger} = \min\{\norm v: \alpha(v) = w \} \] \end{satz} \subsubsection{Beispiel (lineare Regression)} $(t_i, y_i)_{i=1}^m$ gegeben. Wollen Polynome finden, die gut auf die Messungen passen. Suchen also $p: p(t_i) \sim y_i, \forall i \in [m]$ \\ \begin{tikzpicture}[declare function={approx(\x) = 0.17 * \x * \x + -1.1 * \x + 4.1;}] \draw [red, thick] plot [domain=0:10,samples=144,smooth] (\x, {approx(\x)}) node[right,color=black] {$p(t) = a_0 + a_1 t + a_2 t^2$}; \foreach[count=\i] \diff in {0.93,-0.56,-2.12,1.35,-0.83,0.87,-0.04,-1.16,-0.02,1.25,0.13,-0.62,2.71,-1.84,-0.24,1.64,-0.06,-0.52,-0.21,-1.24} \filldraw[ForestGreen] (\i * .5, {approx(\i * .5) + \diff * .6}) circle (.5mm); \draw[->] (0, 0) -- (10, 0); \draw[->] (0, 0) -- (0, 10); \end{tikzpicture} \[ \text{minimiere } \sum_{i=1}^m (f(t_i) - y_i)^2 = \sum_{i=1}^m (a_0 + a_1 t_i + a_2 t_i^2 - y_i)^2 = \norm{A x -b}^2_{\K^m} \text{ wobei} \] \[ A = \begin{pmatrix} 1 & t_1 & t_1^2 \\ \vdots & \vdots & \vdots \\ 1 & t_m & t_m^2 \end{pmatrix}, x = \begin{pmatrix} a_0 \\ a_1 \\ a_2 \end{pmatrix}, b = \begin{pmatrix} y_1 \\ \vdots \\ y_m \end{pmatrix} \] \subsubsection{Anwendung: Ausgleichsquadrik} Problem: homogenes LGS $Ax=0$. Finde $x$ mit $\norm x = 1$ und $\norm{Ax}$ minimal. \\ $b_1, \dots, b_n$ ONB aus EVen von $A^* A$ mit nichtnegativen EWen. \begin{align*} X = \sum \lambda_i b_i \implies \norm{Ax}^2 & = \inner{Ax}{Ax} \\ & = \inner{A^* A x}{x} = \inner{\sum s_i \lambda_i b_i}{\sum \lambda_j b_j} = \sum_{i=1}^n s_i \abs{\lambda_i}^2 \end{align*} $s_1 \le s_2 \le \dots \le s_n, \norm x = \sum \abs{\lambda_i}^2$ \[ \frac{\norm{Ax}}{\norm x} = \frac{\sum s_i \abs{\lambda_i}^2}{\sum \abs{\lambda_i}^2} \ge \frac{s_1 \sum \abs{\lambda_i}^2}{\sum \abs{\lambda_i}^2} s_1 \] $\norm x = 1 \implies \norm{Ax} \ge s_1$ $\norm{b_i} \implies \lambda_1, \lambda_2 = \dots = \lambda_n = 0 \implies \norm{Ab_1} = s_1 \implies b_1$ löst unser Minimierungsproblem. \\ $Q = \{(x,y) \in \R^2: \psi(x, y) = 0\}$ \\ $\psi(x, y):= a_1 x^2 + a_2 xy + a_3 y^2 a_4 x + a_5 y + a_6$ \\ Gegeben: $(x_i,y_i)^m_{i=1}$ Suche $x = (a_1, \dots, a_6)^T$ mit $\norm x = 1$ sodass \[ \sum_{i=1}^m \left(a_1 x_i^2 + a_2 x_i y_i + a_3 y_i^2 + a_4 x_i + a_5 y_i + a_6\right)^2=\norm{Ax}^2 \] minimal. $A = \begin{pmatrix} x_1^2 & x_1 y_1 & y_1^2 & x_1 & y_1 & 1 \\ \vdots & \vdots & \vdots & \vdots & \vdots & \vdots \\ x_m^2 & x_m y_m & y_m^2 & x_m & y_m & 1 \\ \end{pmatrix}$ \\ Suche $x \in \R^6$ mit $\norm x = 1$ und $\norm{Ax}$ minimal. $\implies x$ ist Eigenvektor von $A^* A$ zum kleinsten Eigenwert. \begin{nonumbersatz} Sei $A \in \K^{m \times n}$ und $b \in \K^n$ Eigenvektor von $A^* A$ zum kleinsten Eigenwert $r_1$. Dann gilt \[ \frac{\norm{Ab}}{\norm b} = \min\left\{\frac{\norm{Ax}}{\norm x}: x\in\R^n\right\} = \sqrt{r_1} \] \end{nonumbersatz} \end{document}